HESI exit exam 799 questions

Réussis tes devoirs et examens dès maintenant avec Quizwiz!

An elderly male client is admitted to the urology unit with acute renal failure due to a postrenal obstruction. Which questions best assists the nurse in obtaining relevant historical data?

"Have you had any difficulty in starting your urinary stream"

The nurse applies a blood pressure cuff around a client's left thigh. To measure the client's blood pressure, where should the diaphragm of the stethoscope be placed? (Mark the location on one of the images.)

"On left thigh with arrow pointing to inner thigh"

The nurse administers an antibiotic to a client with respiratory tract infection. To evaluate the medication's effectiveness, which laboratory values should the nurse monitor? Select all that apply

-White blood cell (WBC) count -Sputum culture and sensitivity

An adolescent receives a prescription for an injection of s-matriptan succinate 4 mg subcutaneously for a migraine headache. Using a vial labeled, 6 mg/ 0.5 ml, how many ml should the nurse administer? (Enter the numerical value only. If rounding is required, round to the nearest hundredth.

0.33 mL

A school-age child who weighs 42 pounds receives a post-tonsillectomy prescription for promethazine (Phenergan) 0.5 mg/kg IM to prevent postoperative nausea. The medication is available in 25 mg/ml ampules. How many ml should the nurse administer? (Enter numeric value only. If rounding is required, round to the nearest tenth).

0.4

An infant is receiving penicillin G procaine 220,000 units IM. The drug is supplied as 600,000 units/ml. How many ml should the nurse administer? (Enter numeric value only. If rounding is required, round to the nearest tenth)

0.4

The healthcare provider prescribes celtazidime (Fortax) 35 mg every 8 hours IM for an infant. The 500 mg vial is labeled with the instruction to add 5.3 ml diluent to provide a concentration of 100 mg/ml. How many ml should the nurse administered for each dose? (Enter numeric value only. If rounding is required, round to the nearest tenth

0.4

To reduce staff nurse role ambiguity, which strategy should the nurse-manager implement?

Review the staff nurse job description to ensure that it is clear, accurate, and current

The first paddle has been placed on the chest of a client who needs defibrillation. Where should the nurse place the second paddle? (Mark the location where the second paddle should be placed on the image).

Right upper chest left midaxillary.

A client with multiple sclerosis (MS) is admitted to the medical unit. The client reports... which action should the nurse implement to reduce the client's risk for falls?

Schedule frequent rest periods Provide assistance to bedside commode Teach to patch one eye when ambulating

A client is admitted to a mental health unit after attempting suicide by taking a handful of medications. In developing a plan of care for this client, which goal has the highest priority?

Signs a no-self-harm contract.

A school nurse is called to the soccer field because a child has a nose bleed (epistaxis). In what position should the nurse place the child

Sitting up and leaning forward

An infant is receiving gavage feedings via nasogastric tube. At the beginning of the feeding, the infant's heart rate drops to 80 beats / minute. What action should the nurse take

Slow the feeding and monitor the infant's response.

To reduce the risk of symptoms exacerbation for a client with multiple sclerosis (MS), which instructions should the nurse include in the client's discharge plan? (Select all that apply).

Space activities to allow for rest periods Take warm baths before starting exercise

A male client with cancer who has lost 10 pounds during the last months tells the nurse that beef, chicken, and eggs, which used to be his favorite foods, now they taste "bitter". He complains that he simply has no appetite. What action should the nurse implement?

Suggest the use of alternative sources of protein such as dairy products and nuts

A female client receives a prescription for alendronate sodium (Fosamax) to treat her newly diagnose osteoporosis. What instruction should the nurse include in the client's teaching plan?

Take on an empty stomach with a full glass of water

Nurses working on a surgical unit are concerned about the physicians treatment of clients during invasive procedures, such as dressing changes and insertion of IV lines. Clients are often crying during the procedures, and the physician is usually unconcerned or annoyed by the client's response. To resolve this problem, what actions should the nurses take? (Arrange from the first action on the top of the list on the bottom) Talk to the physician as a group in a non-confrontational manner. File a formal complaint with the state medical board. Submit a written report to the director of nursing. Contact the hospital's chief of medical services. File a formal complaint with the state medical board. Document concerns and report them to the charge nurse.

Talk to the physician as a group in a non-confrontational manner. Document concerns and report them to the charge nurse. Submit a written report to the director of nursing. Contact the hospital's chief of medical services. File a formal complaint with the state medical board.

For the past 24 hours, an antidiarrheal agent, diphenoxylate, has been administered to a bedridden, older client with infectious gastroenteritis. Which finding requires the nurse to take further action?

Tented skin turgor

During a visit to the planned parenthood clinic, a young woman tells the nurse that she is going to discontinue taking the oral contraceptives she has taken for three years because she wants to get pregnant. History indicates that her grandfather has adult onset diabetes and that she was treated for chlamydia six months ago, which factor in this client's history poses the greatest risk for this woman's pregnancy?

Three year history of taking oral contraceptives

A client with hyperthyroidism is admitted to the postoperative after subtotal thyroidectomy. Which of the client's serum laboratory values requires intervention by the nurse

Total calcium 5.0 mg/dl

A client who is at 36 weeks gestations is admitted with severe preclampsia. After a 6 gram loading dose of magnesium sulfate is administered, an intravenous infusion of magnesium sulfate at a rate of 2 grams/hour is initiated. Which assessment finding warrants immediate intervention by the nurse?

Urine output 20 ml/hour

A 4-year-old with acute lymphocytic leukemia (ALL) is receiving a chemotherapy (CT) protocol that includes methotrexate (Mexate, Trexal, MIX), an antimetabolite. Which information should the nurse provide the parents about caring for their child?

Use sunblock or protective clothing when outdoors

The nurse is preparing to mix two medications from two different multidose vials, A and B. In which order should these actions be implemented when drawing the solutions from the vials? (Arrange from first on top to last on the bottom) Verify the drug and dose with the label on the vial Inject the volume of air to be aspirated from each vial Aspirate the desired volume from vial A Aspirate the desired volume from vial B

Verify the drug and dose with the label on the vial Inject the volume of air to be aspirated from each vial Aspirate the desired volume from vial A Aspirate the desired volume from vial B

The practical nurse (PN) is assigned to work with three registered nurses (RN) who are caring for neurologically compromised clients. The client with which change in status is best to assign to the PN?

Viral meningitis whose temperature changed from 101 F to 102 F.

A child with heart failure (HF) is taking digitalis. Which signs indicates to the nurse that the child may be experiencing digitalis toxicity?

Vomiting

The nurse is assessing a female client's blood pressure because she reported feeling dizzy. The blood pressure cuff is inflated to 140 mm hg and as soon as the cuff is deflated a korotkoff sound is heard. Which intervention should the nurse implement next

Wait 1 minute and palpate the systolic pressure before auscultating again

When assessing a male client, the nurse notes that he has unequal lung expansion. What conclusion regarding this finding is most likely to be accurate? The client has: a- A collapsed lung b- A history of COPD c- A chronic lung infection d- Normally functioning lungs

a- A collapsed lung

After teaching a male client with chronic kidney disease (CKD) about therapeutic diet... which menu of foods indicates that the teaching was effective? Select all that apply a- A slice of whole grain toast b- Half cup of black beans c- A ham and cheese sandwich d- A bowl of cream of wheat e- Two bananas.

a- A slice of whole grain toast d- A bowl of cream of wheat

The nurse requests a meals tray for a client follows Mormon beliefs and who is on clear liquid diet following abdominal surgery. Which meal item should the nurse request for this client? (Select all that apply) a- Apple juice b- Chicken broth. c- Hot chocolate d- Orange juice e- Black coffee

a- Apple juice b- Chicken broth.

The healthcare provider prescribes oxycodone/ aspirin 1 tab PO every 4h as needed for pain, for a client with polycystic kidney disease. Before administering this medication, which component of the prescription should the nurse question? a- Aspirin content. b- Dose c- Route d- Risk for addiction

a- Aspirin content.

Which information is more important for the nurse to obtain when determining a client's risk for (OSAS)? a- Body mass index b- Level of consciousness c- Self-description of pain d- Breath sounds

a- Body mass index

A client with end-stage liver failure is declared brain dead. The family wants to discontinue feeding and donate any viable organs. Which action should the nurse take? a- Contact the regional organ procurement agency b- Convene a multidisciplinary care conference c- Explain that client may not be an organ donor candidate d- Discontinue feeding and fluids per the family's request.

a- Contact the regional organ procurement agency

The nurse is assessing the thorax and lungs of a client who is having respiratory difficulty. Which finding is most indicative of respiratory distress? a- Contractions of the sternocleidomastoid muscle. b- Respiratory rate of 20 breath/mints c- Downward movement of diaphragm with inspiration d- A pulse oximetry reading of SpO2 95%

a- Contractions of the sternocleidomastoid muscle.

Following insertion of a LeVeen shunt in a client with cirrhosis of the liver, which assessment finding indicates to the nurse that the shunt is effective? a- Decrease abdominal girth b- Increased blood pressure c- Clear breath sounds d- Decrease serum albumin.

a- Decrease abdominal girth

The nurse assesses a 78-year-old male client who has left sides heart failure. Which symptoms would the nurse expect this client to exhibit? a- Dyspnea, cough, and fatigue. b- Hepatomegaly and distended neck veins c- Pain over the pericardium and friction rub. d- Narrowing pulse pressure and distant heart sounds.

a- Dyspnea, cough, and fatigue.

One day following a total knee replacement, a male client tells the nurse that he is unable to transfer because it is too painful. What action should the nurse implement? a- Encourage use of analgesics before position change b- Assess anxiety about transferring to commode chair c- Assist client during transfer on the first two days d- Review use of assistive devices for weight bearing.

a- Encourage use of analgesics before position change

A young adult male was admitted 36 hours ago for a head injury that occurred as the result of a motorcycle accident. In the last 4 hours, his urine output has increased to over 200 ml/H. Before reporting the finding to the healthcare provider, which intervention should the nurse implement? a- Evaluate the urine osmolality and the serum osmolality values. b- Obtain blood pressure and assess for dependent edema c- Measure oral secretions suctioned during last hours d- Obtain capillary blood samples q2 hours for glucose monitoring.

a- Evaluate the urine osmolality and the serum osmolality values.

The nurse is developing a plan of care for a middle-aged woman who is diagnosed with type 2 diabetes mellitus (DM). To lower her blood glucose and increase her serum high-density lipoprotein (HDL) levels, which instruction is most important for the nurse to provide? a- Exercise at least three times weekly b- Monitor blood glucose levels daily c- Limit intake of foods high in saturated fat d- Learn to read all food product labels

a- Exercise at least three times weekly

Which assessment finding indicates to the nurse a client's readiness for pulmonary function tests? a- Expresses an understanding of the procedure. b- NPO for 6 hrs. c- No known drug allergies d- Intravenous access intact.

a- Expresses an understanding of the procedure.

An adult man reports that he recently experienced an episode of chest pressure and breathlessness when he was jogging in the neighborhood. He expresses concern because both of his deceased parents had heart disease and his father was a diabetic. He lives with his male partner, is a vegetarian, and takes atenolol which maintain his blood pressure at 138/74. Which risk factors should the nurse explore further with the client? Select all that apply a- History of hypertension. b- Homosexual lifestyle c- Vegetarian diet d- Excessive aerobic exercise e- Family heath history.

a- History of hypertension. e- Family heath history.

A nurse is caring for a client with Diabetes Insipidus. Which assessment finding warrants immediate intervention by the nurse? a- Hypernatremia b- Excessive thirst c- Elevated heart rate d- Poor skin turgor

a- Hypernatremia

The nurse is assessing a 4-year-old boy admitted to the hospital with the diagnosis of possible nephrotic syndrome. Which statement by the parents indicates a likely correlation to the child's diagnosis? a- I couldn't get my son's socks and shoes on this morning" b- My son has been on amoxicillin/clavulanate for 2 days for an ear infection c- My son has had a red rash over his entire body for the past 4 days. d- I couldn't get my son calm down and sleep last night.

a- I couldn't get my son's socks and shoes on this morning"

The nurse is caring for a client with hypovolemic shock who is receiving two units of packed red blood cells (RBCs) through a large bore peripheral IV. What action promotes maintenance of the client's cardiopulmonary stability during the blood transfusion? a- Increase the oxygen flow via nasal cannula if dyspnea is present. b- Place in a Trendelenburg position to increase cerebral blood flow c- Monitor capillary glucose measurements hourly during transfusion. d- Encourage increased intake of oral fluid to improve skin turgor.

a- Increase the oxygen flow via nasal cannula if dyspnea is present.

A young adult male is admitted to the emergency department with diabetic ketoacidosis (DKA). His pH is 7.25, HCO3 is 12 mEq/L or 12 mmol/L (SI), and blood glucose is 310 mg/dl or 17.2 mmol/L (SI). Which action should the nurse implement? a- Infuse sodium chloride 0.9% (normal saline) b- Prepare an emergency dose of glucagon c- Determine the last time the client ate d- Check urine for ketone bodies with a dipstick

a- Infuse sodium chloride 0.9% (normal saline)

During a left femoral artery aortogram, the healthcare provider inserts an arterial sheath and initiate. Through the sheath to dissolve an occluded artery. Which interventions should the nurse implement? a- Instruct the client to keep the left leg straight b- Keep the head of bed at 60-degree angle. c- Observe the insertion site for a hematoma d- Manually flush the arterial sheath hourly e- Circle first noted drainage on the dressing

a- Instruct the client to keep the left leg straight c- Observe the insertion site for a hematoma e- Circle first noted drainage on the dressing

The nurse is teaching a client about the antiulcer medications ranitidine which was... statement best describes the action of this drug? a- It blocks the effects of histamine, causing decreased secretion of acid b- Ranitidine will neutralize gastric acid and decrease gastric pH c- This drug provides a protective coating over the gastric mucosa d- It effectively blocks 97% of the gastric acid secreted in the stomach

a- It blocks the effects of histamine, causing decreased secretion of acid

The nurse is assessing a middle-aged adult who is diagnosed with osteoarthritis. Which factor in this client's history is a contributor to the osteoarthritis? a- Long distance runner since high school. b- Lactose intolerant since childhood c- Photosensitive to a drug currently taking d- Recently treated for deep vein thrombosis.

a- Long distance runner since high school.

A client with acute pancreatitis is complaining of pain and nausea. Which interventions should the nurse implement (Select all that apply) a- Monitor heart, lung, and kidney function. b- Notify healthcare provider of serum amylase and lipase levels. c- Position client on abdomen to provide organ stability d- Encourage an increased intake of clear oral fluids e- Review client's abdominal ultrasound findings.

a- Monitor heart, lung, and kidney function. b- Notify healthcare provider of serum amylase and lipase levels. e- Review client's abdominal ultrasound findings.

A male client is returned to the surgical unit following a left nephrectomy and is medicated with morphine. His dressing has a small amount of bloody drainage, and a JacksonPratt bulb surgical drainage device is in place. Which interventions is most important for the nurse to include in this clients plan of care? a- Monitor urine output hourly. b- Assess for back muscle aches c- Record drainage from drain d- Obtain body weight daily

a- Monitor urine output hourly.

The nurse is caring for a toddler with a severe birth anomaly that is dying. The parents... holding the child as death approaches. Which intervention is most important for the nurse? a- Notify nursing supervisor and hospital chaplain of the child's impending death. b- Verify that the no resuscitate forms are in the child's medical record c- Ask the parents if they have made arrangements with a funeral home d- Provide staff coverage to sit with them as the child's death approaches.

a- Notify nursing supervisor and hospital chaplain of the child's impending death.

When changing a diaper on a 2-day-old infant, the nurse observes that the baby's legs are... this finding, what action should the nurse take next? a- Notify the healthcare provider b- Continue care since this is a normal finding c- Document the finding in the record d- Perform range of motion to the joint.

a- Notify the healthcare provider

The nurse is preparing dose # 7 of an IV piggyback infusion of tobramycin for a 73-yearol client with... Infected pseudomonas aeruginosa. Which assessment data warrants further intervention by the nurse? a- Peak and through levels has not been drawn since the tobramycin was started b- Today labs report indicates a white blood cell count of 13,000 cell/mm3 or 13 x 10777/L (S1) c- A serum creatinine level of 1.0 mg/dl or 88 mcmol/L (S1) is documented on yesterday flowsheet. d- The culture growth form the burn areas is sensitive to aminoglycosides.

a- Peak and through levels has not been drawn since the tobramycin was started

Which interventions should the nurse include in a long-term plan of care for a client with COPD? a- Reduce risk factors for infection b- Administer high flow oxygen during sleep c- Limit fluid intake to reduce secretions d- Use diaphragmatic breathing to achieve better exhalation

a- Reduce risk factors for infection

The nurse is preparing to discharge an older adult female client who is at risk for hyPOCALCEMIA nurse include with this client's discharge teaching? a- Report any muscle twitching or seizures b- Take vitamin D with calcium daily c- Avoid seafood, particularly selfish d- Low fat yogurt is a good source of calcium e- Keep a diet record to monitor calcium intake

a- Report any muscle twitching or seizures b- Take vitamin D with calcium daily d- Low fat yogurt is a good source of calcium e- Keep a diet record to monitor calcium intake

A client whose wrists are sutured from a recent suicide attempt is been transferred from a medical unit. Which nursing diagnosis is of the highest priority? a- Risk for self-directed violence related to impulsive actions b- Risk for violence related to feeling of guilt and failure c- Low self-esteem related to feeling of loss of control d- Ineffective coping related to violent actions towards self.

a- Risk for self-directed violence related to impulsive actions

A school-aged child was recently diagnosed with celiac disease. Which instruction should the nurse give the classroom teacher? a- The child should avoid eating homemade cookies and cupcakes during parties. b- No products containing any form of peanuts should be allowed in the classroom c- Report a runny nose or head cold to the nurse immediately for further revaluation. d- Avoiding direct contact sports and games will reduce the child's risk of bruising.

a- The child should avoid eating homemade cookies and cupcakes during parties.

A 6 -years-old who has asthma is demonstrating a prolonged expiratory phase and wheezing, and has 35% personal best peak expiratory flow rate (PEFR). Based on these finding, which action should the nurse implement first? a. Administer a prescribed bronchodilator. b. Report finding to the healthcare provider. c. Encourage the child to cough and deep breath d. Determine what trigger precipitated this attack.

a. Administer a prescribed bronchodilator.

A newly admitted client vomits into an emesis basin as seen in the picture. The nurse should consult with the healthcare provider before administering which of the client's prescribes medications? a. Clopidogrel (Plavix), an antiplatelet agent, given orally b. Methylprednisolone (solu-medrol), a corticosteroid, to be given IV c. Nitroglycerin (Nitro-Dur) an antianginal, to be given transdermally. d. Enoxaparin (lovenox), a low-molecular weight heparin to be given subcutaneous. e. Furosemide (Lasix), a loop diuretic, to be given intravenously.

a. Clopidogrel (Plavix), an antiplatelet agent, given orally

A young couple who has been unsuccessful in conceiving a child for over a year is seen in the family planning clinic. During an initial visit, which intervention is most important for the nurse to implement? a. Determine current sexual practice b. Prepare a female client for an ultrasound c. Request a sperm sample for ovulation d. Evaluate hormone levels on both clients

a. Determine current sexual practice

A newly hired home health care nurse is planning the initial visit to an adult client who has had multiple sclerosis (MS) for the past 20 years and is currently bed-bound and is lifted by a hoist. And unlicensed caregiver provides care 8 hours/ daily, 5 days/week. During the initial visit to this client, which intervention is most important to the nurse to implement? a. Determine how the client is cared for when caregiver is not present. b. Develop a client needs assessment and review with the caregiver c. Evaluate the caregiver's ability to care for the client's needs. d. Review with the care giver the interventions provided each day.

a. Determine how the client is cared for when caregiver is not present.

A client with persistent low back pain has received a prescription for electronic stimulator (TENS) unit. After the nurse applies the electrodes and turns on the power, the client reports feeling a tingling sensation. How should the nurse respond? a. Determine if the sensation feels uncomfortable. b. Decrease the strength of the electrical signals. c. Remove electrodes and observe for skin redness. d. Check the amount of gel coating on the electrodes.

a. Determine if the sensation feels uncomfortable.

A male client was transferred yesterday from the emergency department to the telemetry unit because he had ST depression and resolved chest pain. When his EKG monitor alarms for ventricular tachycardia (VT), what action should the nurse take first? a. Determine the client's responsiveness and respirations b. Bring the crash cart to the room to defibrillate the client. c. Immediately initiate chest compressions. d. Notify the emergency response team

a. Determine the client's responsiveness and respirations

The charge nurse of the Intensive Care Unit is making assignments for the permanent staff and one RN who was floated from a medical unit. The client with which condition is the best to assign to the float nurse? a. Diabetic ketoacidosis and titrated IV insulin infusion b. Emphysema extubated 3 hours ago receiving heated mist c. Subdural hematoma with an intracranial monitoring device d. Acute coronary syndrome treated with vasopressors

a. Diabetic ketoacidosis and titrated IV insulin infusion

After receiving the first dose of penicillin, the client begins wheezing and has trouble breathing. The nurse notifies the healthcare provider immediately and received several prescriptions. Which medication prescription should the nurse administer first? a. Epinephrine Injection, USP IV b. Diphenhydramine IV c. Albuterol (Ventolin) inhaler d. Methylprednisolone IV

a. Epinephrine Injection, USP IV

In monitoring tissue perfusion in a client following an above the knee amputation (aka), which action should the nurse include in the plan of care? a. Evaluate closet proximal pulse. b. Asses skin elasticity of the stump. c. Observe for swelling around the stump. d. Note amount color of wound drainage.

a. Evaluate closet proximal pulse.

The nurse ask the parent to stay during the examination of a male toddler's genital area. Which intervention should the nurse implement? a. Examine the genitalia as the last part of the total exam. b. Use soothing statements to facilitate cooperation c. Allow the child to keep underpants on to examine genitalia d. Work slowly and methodically so not to stress the child

a. Examine the genitalia as the last part of the total exam.

A young adult woman visits the clinic and learns that she is positive for BRCA1 gene mutation and asks the nurse what to expect next. How should the nurse respond? a. Explain that counseling will be provided to give her information about her cancer risk. b. Gather additional information about the client's family history for all types of cancer. c. Offer assurance that there are a variety of effective treatments for breast cancer. d. Provide information about survival rates for women who have this genetic mutation.

a. Explain that counseling will be provided to give her information about her cancer risk.

An adult male was diagnosed with stage IV lung cancer three weeks ago. His wife approaches the nurse and asks how she will know that her husband's death is imminent because their two adult children want to be there when he dies. What is the best response by the nurse? a. Explain that the client will start to lose consciousness and his body system will slow down b. Reassure the spouse that the healthcare provider will let her know when to call the children c. Offer to discuss the client's health status with each of the adult children d. Gather information regarding how long it will take for the children to arrive

a. Explain that the client will start to lose consciousness and his body system will slow down

When obtaining a rectal temperature with an electronic thermometer, which action is most important for the nurse to perform? a. Hold the thermometer in place. b. Place the disposable pad under buttocks c. Instruct the client to breathe deeply d. Return the probe to the charger.

a. Hold the thermometer in place.

A group of nurse-managers is asked to engage in a needs assessment for a piece of equipment that will be expensed to the organization's budget. Which question is most important to consider when analyzing the cost-benefit for this piece of equipment? a. How many departments can use this equipment? b. Will the equipment require annual repair? c. Is the cost of the equipment reasonable? d. Can the equipment be updated each year?

a. How many departments can use this equipment?

A client is admitted to the intensive care unit with diabetes insipidus due to a pituitary gland tumor. Which potential complication should the nurse monitor closely? a. Hypokalemia b. Ketonuria. c. Peripheral edema d. Elevated blood pressure

a. Hypokalemia

A male client arrives at the clinic with a severe sunburn and explains that he did not use sun screen because it was an overcast day. Large blisters are noted over his back and chest and his shirt is soaked with serosanguinous fluid. Which assessment finding warrants immediate intervention by the nurse? a. Hypotension. b. Fever and chills c. Dizziness d. Headache

a. Hypotension.

The mother of a 7-month-old brings the infant to the clinic because the skin in the diaper area is excoriated and red, but there are no blisters or bleeding. The mother reports no evidence of watery stools. Which nursing intervention should the nurse implement? a. Instruct the mother to change the child's diaper more often. b. Encourage the mother to apply lotion with each diaper charge c. Tell the mother to cleanse with soap and water at each diaper change d. Ask the mother to decrease the infant's intake of fruits for 24 hours.

a. Instruct the mother to change the child's diaper more often.

A child newly diagnosed with sickle cell anemia (SCA) is being discharged from the hospital. Which information is most important for the nurse to provide the parents prior to discharge? a. Instructions about how much fluid the child should drink daily b. information about non-pharmaceutical pain reliever measures c. Referral for social services for the child and family d. Signs of addiction to opioid and medications

a. Instructions about how much fluid the child should drink daily

The nurse is caring for a client who is taking a macrolide to treat a bacterial infection. Which finding should the nurse report to the healthcare provider before administering the next dose? a. Jaundice b. Nausea c. Fever d. Fatigue

a. Jaundice

The client with which type of wound is most likely to need immediate intervention by the nurse? a. Laceration b. Abrasion c. Contusion d. Ulceration

a. Laceration

An adult male client is admitted to the emergency room following an automobile collision in which he sustained a head injury. What assessment data would provide the earliest that the client is experiencing increased intracranial pressure (ICP)? a. Lethargy b. Decorticate posturing c. Fixed dilated pupil d. Clear drainage from the ear.

a. Lethargy

A client is receiving lactulose (Portalac) for signs of hepatic encephalopathy. To evaluate the client's therapeutic response to this medication, which assessment should the nurse obtain? a. Level of consciousness b. Percussion of abdomen c. Serum electrolytes d. Blood glucose.

a. Level of consciousness

The nurse is developing a teaching program for the community. What population characteristic is most influential when choosing strategies for implementing a teaching plan? a. Literacy level b. Prevalent learning style c. Median age d. Percent with internet access.

a. Literacy level

An adult female client with chronic kidney disease (CKD) asks the nurse if she can continue...Medications. Which medication provides the greatest threat to this client? a. Magnesium hydroxide (Maalox). b. Birth control pills c. Cough syrup containing codeine d. Cold medication containing alcohol

a. Magnesium hydroxide (Maalox).

The nurse is preparing a client for discharge from the hospital following a liver transplant. Which instruction is most important for the nurse to include in this client's discharge teaching plan? a. Monitor for an elevated temperature b. Measure the abdominal girth daily c. Report the onset of sclera jaundice d. Keep a record of daily urinary output

a. Monitor for an elevated temperature

A client with acute renal failure (ARF) is admitted for uncontrolled type 1 diabetes Mellitus and hyperkalemia. The nurse administers an IV dose of regular insulin per sliding scale. Which intervention is the most important for the nurse to include in this client's plan of care? a. Monitor the client's cardiac activity via telemetry. b. Maintain venous access with an infusion of normal saline. c. Assess glucose via fingerstick q4 to 6 hours. d. Evaluate hourly urine output for return of normal renal function

a. Monitor the client's cardiac activity via telemetry.

While monitoring a client during a seizure, which interventions should the nurse implement? (Select all that apply a. Move obstacle away from client b. Monitor physical movements c. Insert an oral padded tongue blade d. Observe for a patent airway e. Record the duration of the seizure f. Restrain extremity to avoid seizures

a. Move obstacle away from client b. Monitor physical movements d. Observe for a patent airway e. Record the duration of the seizure

An older female who ambulate with a quad-cane prefer to use a wheel chair because she has a halting and unsteady gait at times. Which interventions should the nurse implement? (Select all that apply) a. Move personal items within client's reach b. Lower bed to the lower possible position c. Raise all bed rails when the client is resting d. Give directions to call for assistance e. Assist client to the bathroom in 2 hours. f. Encourage the use of the wheelchair

a. Move personal items within client's reach b. Lower bed to the lower possible position d. Give directions to call for assistance e. Assist client to the bathroom in 2 hours.

An older client is admitted to the intensive care unit with severe abdominal pain, abdominal distention, and absent bowel sound. The client has a history of smoking 2 packs of cigarettes daily for 50 years and is currently restless and confused. Vital signs are: temperature 96`F, heart rate 122 beats/minute, respiratory rate 36 breaths/minute, mean arterial pressure(MAP) 64 mmHg and central venous pressure (CVP) 7 mmHg. Serum laboratory findings include: hemoglobin 6.5 grams/dl, platelets 6o, 000, and white blood cell count (WBC) 3,000/mm3. Based on these findings this client is at greatest risk for which pathophysiological condition? a. Multiple organ dysfunction syndrome (MODS) b. Disseminated intravascular coagulation (DIC) c. Chronic obstructive disease. d. Acquired immunodeficiency syndrome (AIDS)

a. Multiple organ dysfunction syndrome (MODS)

A client with urticaria due to an environmental allergies is taking diphenhydramine... Which complaint should the nurse identify to the client as a side effect of the OTC medication? a. Nausea and indigestion. b. Hyper salivation c. Eyelid and facial twitching d. Increased appetite

a. Nausea and indigestion.

A client is admitted to isolation with the diagnosis of active tuberculosis (TB). Which infection control measures should the nurse implement? a. Negative pressure environment b. Contact precautions c. Droplet precautions d. Protective environment

a. Negative pressure environment

The nurse notes an increase in serosanguinous drainage from the abdominal surgical wound from an obese client. What action should the nurse implement? a. Observe the wound for dehiscence b. Teach the client to splint the incision while coughing c. Assess the skin surrounding the wound for maceration d. Obtain a culture of the wound drainage.

a. Observe the wound for dehiscence

An adult client with schizophrenia begin treatment three days ago with the Antipsychotic risperidone. The client also received prescription for trazodone as needed for sleep and clonazepam as needed for severe anxiety. When the client reports difficulty with swallowing, what action should the nurse take? a. Obtain a prescription for an anticholinergic medication b. Determine how many hours declined slept last night c. Administer the PRN prescription for severe anxiety d. Watch the thyroid cartilage move while the client swallows

a. Obtain a prescription for an anticholinergic medication

A toddler with a history of an acyanotic heart defect is admitted to the pediatric intensive...rate of 60 breaths/ minute, and a heart rate of 150 beats/minute. What action should the nurse take? a. Obtain a pulse oximeter reading b. Assess the child blood pressure c. Perform a neurological assessment d. Initiate peripheral intravenous access.

a. Obtain a pulse oximeter reading

A young adult female college student visits the health clinic in early winter to obtain birth control pills. The clinic nurse asks if the student has received an influenza vaccination. The student stated she did not receive vaccination because she has asthma. How should the nurse respond? a. Offer to provide the influenza vaccination to the student while she is at the clinic b. Encourage the student to obtain a vaccination prior to the next influenza season. c. Confirm that a history of asthma can increase risks associated with the vaccine. d. Advise the student that the nasal spray vaccine reduces side effects for people with asthma.

a. Offer to provide the influenza vaccination to the student while she is at the clinic

A client on a long-term mental health unit repeatedly takes own pulse regardless of the circumstance. What action should the nurse implement? a. Overlook the client's behavior. b. Distract client to interfere with the ritual. c. Ask why the client checks the pulse. d. Hold client's hand to stop the behavior.

a. Overlook the client's behavior.

To obtain an estimate of a client's systolic B/P. What action should the nurse take first? a. Palpate the client's brachial pulse b. Pump up the blood pressure cuff c. Position the stethoscope diaphragm d. Release the blood pressure cuff valve

a. Palpate the client's brachial pulse

A 26-year-old female client is admitted to the hospital for treatment of a simple goiter, and levothyroxine sodium (Synthroid) is prescribed. Which symptoms indicate to the nurse that the prescribed dosage is too high for this client? The client experiences a. Palpitations and shortness of breath b. Bradycardia and constipation c. Lethargy and lack of appetite d. Muscle cramping and dry, flushed skin

a. Palpitations and shortness of breath

When conducting diet teaching for a client who is on a postoperative soft diet, which foods should eat? (Select all that apply) a. Pasta, noodles, rice. b. Egg, tofu, ground meat. c. Mashed, potatoes, pudding, milk. d. Brussel sprouts, blackberries, seeds. e. Corn bran, whole wheat bread, whole grains.

a. Pasta, noodles, rice. b. Egg, tofu, ground meat. c. Mashed, potatoes, pudding, milk.

The rapid response team's detects return of spontaneous circulation (ROSC) after 2 min of continuous chest compressions. The client has a weak, fast pulse and no respiratory effort, so the healthcare provider performs a successful oral, intubation. What action should the nurse implement? a. Perform bilateral chest auscultation. b. Resume compression for 2 minutes c. Administer a dose of epinephrine d. Program the monitor for cardioversion.

a. Perform bilateral chest auscultation.

The nurse is caring for a client immediately after inserting a PICC line. Suddenly, the client becomes anxious and tachycardiac, and loud churning is heard over the pericardium upon auscultation. What action should the nurse take first? a. Place client in Trendelenburg position on the left side. b. Administer precordial thump c. Monitor the client with a 12-lead electrocardiogram d. Request a STAT portable chest x-ray.

a. Place client in Trendelenburg position on the left side.

In caring for the body of a client who just died, which tasks can be delegate to the unlicensed assistive personnel (UAP)? (Select all that apply.) a. Place personal religious artifacts on the body. b. Confirm the client's wishes for tissue donation c. Observe consent for autopsy signature by family. d. Attach identifying name tags to the body. e. Follow cultural beliefs in preparing the body.

a. Place personal religious artifacts on the body. d. Attach identifying name tags to the body. e. Follow cultural beliefs in preparing the body.

The nurse is caring for a one week old infant who has a ventriculoperitoneal (VP) shunt that was placed 2 days after birth. Which findings are an indication of a postoperative complication? a. Poor feeding and vomiting b. Leakage of CSF from the incisional site c. Hyperactive bowel sound d. Abdominal distention e. WBC count of 10000/mm3

a. Poor feeding and vomiting b. Leakage of CSF from the incisional site d. Abdominal distention

Following an esophagogastroduodenoscopy (EGD) a male client is drowsy and difficult to arouse, and his respiration are slow and shallow. Which action should the nurse implement? Select all that apply. a. Prepare medication reversal agent b. Check oxygen saturation level c. Apply oxygen via nasal cannula d. Initiate bag- valve mask ventilation. e. Begin cardiopulmonary resuscitation

a. Prepare medication reversal agent b. Check oxygen saturation level c. Apply oxygen via nasal cannula

A client with coronary artery disease who is experiencing syncopal episodes is admitted for an electrophysiology study (EPS) and possible cardiac ablation therapy. Which intervention should the nurse delegate to the unlicensed assistive personnel (UAP)? a. Prepare the skin for procedure. b. Identify client's pulse points c. Witness consent for procedure d. Check telemetry monitoring

a. Prepare the skin for procedure.

The nurse is teaching a group of clients with rheumatoid arthritis about the need to modify daily activities. Which goal should the nurse emphasize? a. Protect joint function b. Improve circulation c. Control tremors d. Increase weight bearing.

a. Protect joint function

A 56-years-old man shares with the nurse that he is having difficulty making decision about terminating life support for his wife. What is the best initial action by the nurse? a. Provide an opportunity for him to clarify his values related to the decision b. Encourage him to share memories about his life with his wife and family c. Advise him to seek several opinions before making decision d. Offer to contact the hospital chaplain or social worker to offer support.

a. Provide an opportunity for him to clarify his values related to the decision

A client with a cervical spinal cord injury (SCI) has Crutchfield tongs and skeletal traction applied as a method of closed reduction. Which intervention is most important for the nurse to include in the client's a plan of care? a. Provide daily care of tong insertion sites using saline and antibiotic ointment b. Modify the client's diet to prevent constipation c. Encourage active range of motion q2 to 4 hours. d. Instruct the client to report any symptoms of upper extremity paresthesia.

a. Provide daily care of tong insertion sites using saline and antibiotic ointment

While assessing a client's chest tube (CT), the nurse discovers bubbling in the water seal chamber of the chest tube collection device. The client's vital signs are: blood pressure of 80/40 mmHg, heart rate 120 beats/minutes, respiratory rate 32 breaths/minutes, oxygen saturation 88%. Which interventions should the nurse implement? a. Provide supplemental oxygen b. Auscultate bilateral lung fields c. Administer a nebulizer treatment d. Reinforce occlusive CT dressing e. Give PRN dose of pain medication

a. Provide supplemental oxygen b. Auscultate bilateral lung fields d. Reinforce occlusive CT dressing

A man expresses concern to the nurse about the care his mother is receiving while hospitalized. He believes that her care is not based on any ethical standards and ask what type of care he should expect from a public hospital. What action should the nurse take? a. Provide the man and his mother with a copy of the Patient's Bill of Rights b. Explain that the hospital adheres to all national accreditation standards c. Advise the man to discuss his concerns with his mother's healthcare provider d. Determine if he would like to review the hospital's manual of approved polices.

a. Provide the man and his mother with a copy of the Patient's Bill of Rights

A low-risk primigravida at 28-weeks gestation arrives for her regular antepartal clinic visit. Which assessment finding should the nurse consider within normal limits for this client? a. Pulse increase of 10 beats/minute b. Proteinuria c. Glucosuria d. Fundal height 0f 22 centimeters

a. Pulse increase of 10 beats/minute

The nurse determines that a client's pupils constricts as they change focus from a far object. What documentation should the nurse enter about this finding? a. Pupils reactive to accommodation b. Nystagmus present with pupillary focus. c. Peripheral vision intact d. Consensual pupillary constriction present

a. Pupils reactive to accommodation

While undergoing hemodialysis, a male client suddenly complains of dizziness. He is alert and oriented, but his skin is cool and clammy. His vital signs are: heart rate 128 beats/minute, respirations 18 breaths/minute, and blood pressure 90/60. Which intervention should the nurse implement first? a. Raise the client's legs and feet b. Administer 250 ml saline bolus c. Decrease blood flow from dialyzer d. Stop the hemodialysis procedure.

a. Raise the client's legs and feet

A client who had a right hip replacement 3 day ago is pale has diminished breath sound over the left lower lung fields, a temperature of 100.2 F, and an oxygen saturation rate of 90%. The client is scheduled to be transferred to a skilled nursing facility (SNF) tomorrow for rehabilitative critical pathway. Based on the client's symptoms, what recommendation should the nurse give the healthcare provider? a. Reassess readiness for SNF transfer. b. Obtain specimens for culture analysis c. Confer with family about home care plans d. Arrange physical therapy for strengthening.

a. Reassess readiness for SNF transfer.

The nurse is preparing a community education program on osteoporosis. Which instruction is helpful in preventing bone loss and promoting bone formation? a. Recommend weigh bearing physical activity b. Reduce intake of foods high in vitamin D c. Decrease intake of foods high in fat d. Minimize heavy lifting and bending.

a. Recommend weigh bearing physical activity

Which intervention should the nurse include in a long-term plan of care for a client with Chronic Obstructive Pulmonary Disease (COPD)? a. Reduce risks factors for infection b. Administer high flow oxygen during sleep c. Limit fluid intake to reduce secretions d. Use diaphragmatic breathing to achieve better exhalation

a. Reduce risks factors for infection

A client with diabetic peripheral neuropathy has been taking pregabalin (Lyrica) for 4 days. Which finding indicates to the nurse that the medication is effective? a. Reduced level of pain b. Full volume of pedal pulses c. Granulating tissue in foot ulcer d. Improved visual acuity.

a. Reduced level of pain

The nurse observes an adolescent client prepare to administer a prescribed corticosteroid medication using a metered dose inhaler as seen in the picture. What action should the nurse take? a. Remind the client to hold his breath after inhaling the medication b. Confirm that the client has correctly shaken the inhaler c. Affirm that the client has correctly positioned the inhaler d. Ask the client if he has a spacer to use for this medication

a. Remind the client to hold his breath after inhaling the medication

At 1615, prior to ambulating a postoperative client for the first time, the nurse reviews the client's medical record. Based on date contained in the record, what action should the nurse take before assisting the client with ambulation: a. Remove sequential compression devices. b. Apply PRN oxygen per nasal cannula. c. Administer a PRN dose of an antipyretic. d. Reinforce the surgical wound dressing.

a. Remove sequential compression devices.

The leg of a client who is receiving hospice care have become mottled in appearance. When the nurse observes the unlicensed assistive personal (UAP) place a heating pad on the mottled areas, what action should the nurse take? a. Remove the heating pads and place a soft blanket over the client's leg and feet. b. Advise the UAP to observe the client's skin while the heating pads are in place. c. Elevate the client's feet on a pillow and monitor the client's pedal pulses frequently. d. Instruct the UAP to reposition the heating pads to the sides of the legs and feet.

a. Remove the heating pads and place a soft blanket over the client's leg and feet.

When preparing a client for discharge from the hospital following a cystectomy and a urinary diversion to treat bladder cancer, which instruction is most important for the nurse to include in the client's discharge teaching plan? a. Report any signs of cloudy urine output. b. Seek counseling for body image concerns c. Follow instruction for self-care toileting d. Frequently empty bladder to avoid distension.

a. Report any signs of cloudy urine output.

A child is admitted to the pediatric unit diagnosed with sickle cell crisis. When the nurse walks into the room, the unlicensed assistive personnel (UAP) is encouraging the child to stay in bed in the supine position. Which action should the nurse implement? a. Reposition the client with the head of the bed elevated. b. Commend the UAP for implementing the proper position c. Tell the UAP that this position is harmful to the client d. Encourage the child to ambulate in the room

a. Reposition the client with the head of the bed elevated.

The nurse is preparing a 4-day-old I infant with a serum bilirubin level of 19 mg/dl (325 micromol/L) for discharge from the hospital. When teaching the parents about home phototherapy, which instruction should the nurse include in the discharge teaching plan? a. Reposition the infant every 2 hours. b. Perform diaper changes under the light. c. Feed the infant every 4 hours. d. Cover with a receiving blanket.

a. Reposition the infant every 2 hours.

A client who had a below the knee amputation is experiencing severe phantom limb pain (PLP) and ask the nurse if mirror therapy will make the pain stop. Which response by the nurse is likely to be most helpful? a. Research indicates that mirror therapy is effective in reducing phantom limb pain b. You can try mirror therapy, but do not expect to complete elimination of the pain c. Transcutaneous electrical nerve stimulators (TENS) have been found to be more effective d. Where did you learn about the use of mirror therapy in treating in treating phantom limb pain?

a. Research indicates that mirror therapy is effective in reducing phantom limb pain

During shift report, the central electrocardiogram (EKG) monitoring system alarms. Which client alarm should the nurse investigate first? a. Respiratory apnea of 30 seconds b. Oxygen saturation rate of 88% c. Eight premature ventricular beats every minute d. Disconnected monitor signal for the last 6 minutes.

a. Respiratory apnea of 30 seconds

The nurse is assessing a 3-month-old infant who had a pylorotomy yesterday. This child should be medicated for pain based on which findings? Select all that apply: a. Restlessness b. Clenched Fist c. Increased pulse rate d. Increased respiratory rate. e. Increased temperature f. Peripheral pallor of the skin

a. Restlessness b. Clenched Fist c. Increased pulse rate d. Increased respiratory rate.

The healthcare provider prescribes a low-fiber diet for a client with ulcerative colitis. Which food selection would indicate to the nurse the client understands they prescribed diet? a. Roasted turkey canned vegetables b. Baked potatoes with skin raw carrots c. Pancakes whole-grain cereal's d. Roast pork fresh strawberries

a. Roasted turkey canned vegetables

53- A male client receives a thrombolytic medication following a myocardial infarction. When the client has a bowel movement, what action should the nurse implement? a. Send stool sample to the lab for a guaiac test b. Observe stool for a day-colored appearance. c. Obtain specimen for culture and sensitivity analysis d. Asses for fatty yellow streaks in the client's stool.

a. Send stool sample to the lab for a guaiac test

When administering ceftriaxone sodium (Rocephin) intravenously to a client before... most immediate intervention by the nurse? a. Stridor b. Nausea c. Headache d. Pruritus

a. Stridor

Which assessment finding for a client who is experiencing pontine myelinolysis should the nurse report to the healthcare provider? a. Sudden dysphagia b. Blurred visual field c. Gradual weakness d. Profuse diarrhea

a. Sudden dysphagia

After the nurse witnesses a preoperative client sign the surgical consent form, the nurse signs the form as a witness. What are the legal implications of the nurse's signature on the client's surgical consent form? (Select all that apply) a. The client voluntarily grants permission for the procedure to be done b. The surgeon has explained to the client why the surgery is necessary. c. The client is competent to sign the consent without impairment of judgment d. The client understands the risks and benefits associated with the procedure e. After considering alternatives to surgery, the client elects to have the procedure.

a. The client voluntarily grants permission for the procedure to be done c. The client is competent to sign the consent without impairment of judgment d. The client understands the risks and benefits associated with the procedure

A mother brings her 4-month-old son to the clinic with a quarter taped over his umbilicus, and tells the nurse the quarter is supposed to fix her child's hernia. Which explanations should the nurse provide? a. This hernia is a normal variation that resolves without treatment. b. Restrictive clothing will be adequate to help the hernia go away. c. An abdominal binder can be worn daily to reduce the protrusion. d. The quarter should be secured with an elastic bandage wrap.

a. This hernia is a normal variation that resolves without treatment.

A 2-year-old is bleeding from a laceration on the right lower extremity that occurred as the result of a motor vehicle collision. The nurse is selecting supplies to start an IV access. Which assessment finding is most significant in the nurse's selection of catheter size? a. Thready brachial pulse. b. Respirations of 24/minute c. Right foot cool to touch. d. Swelling at the site of injury

a. Thready brachial pulse.

When caring for a client who has acute respiratory distress syndrome (ARDS), the nurse elevates the head of the bed 30 degrees. What is the reason for this intervention? a. To reduce abdominal pressure on the diaphragm b. to promote retraction of the intercostal accessory muscle of respiration c. to promote bronchodilation and effective airway clearance d. to decrease pressure on the medullary center which stimulates breathing

a. To reduce abdominal pressure on the diaphragm

A client with eczema is experiencing severe pruritus. Which PRN prescriptions should the nurse administer? (Select all that apply) a. Topical corticosteroid. b. Topical scabicide. c. Topical alcohol rub. d. Transdermal analgesic. e. Oral antihistamine

a. Topical corticosteroid. e. Oral antihistamine

Following and gunshot wound, an adult client a hemoglobin level of 4 grams/dl (40 mmol/L SI). The nurse prepares to administer a unit of blood for an emergency transfusion. The client has AB negative blood type and the blood bank sends a unit of type A Rh negative, reporting that there is not type AB negative blood currently available. Which intervention should the nurse implement? a. Transfuse Type A negative blood until type AB negative is available. b. Recheck the client's hemoglobin, blood type and Rh factor. c. Administer normal saline solution until type AB negative is available d. Obtain additional consent for administration of type A negative blood

a. Transfuse Type A negative blood until type AB negative is available.

Following a gun shot wound to the abdomen, a young adult male had an emergency bowel...Multiple blood products while in the operating room. His current blood pressure is 78/52...He is being mechanically ventilated, and his oxygen saturation is 87%. His laboratory values...Grams / dl (70 mmol / L SI), platelets 20,000 / mm 3 (20 x 10 9 / L (SI units), and white blood cells. Based on these assessments findings, which intervention, should the nurse implements first? a. Transfuse packed red blood cells b. Obtain blood and sputum cultures. c. Infuse 1000 ml normal saline d. Titrate oxygen to keep o2 saturation 90%

a. Transfuse packed red blood cells

The healthcare provider prescribes the antibiotic Cefdinir (cephalosporin) 300mg PO every 12 h for a client with postoperative wound infections. Which feeds should the nurse encourage this client to eat? a. Yogurt and/or buttermilk. b. Avocados and cheese c. Green leafy vegetables d. Fresh fruits

a. Yogurt and/or buttermilk.

A female client with severe renal impairment is receiving enoxaparin (lovenox) 30 mg SUBQ BID. Which laboratory value due to enoxaparin should the nurse report to the healthcare provider? a. creatinine clearance 25 mL/ minute b. calcium 9 mg/dl c. hemoglobin 12 grams/dl d. partial thromboplastin time (PTT) 30 seconds

a. creatinine clearance 25 mL/ minute

The nurse is triaging clients in an urgent care clinic. The client with which symptoms should be referred to the health care provider immediately? a. headache, photophobia, and nuchal rigidity b. high fever, skin rash, and a productive cough c. nausea, vomiting, and poor skin turgor d. malaise, fever, and stiff, swollen joints

a. headache, photophobia, and nuchal rigidity

The nurse is conducting the initial assessment of an ill client who is from another culture.... What response should the nurse provide? a- Can you read the written instructions is English? b- "What practices do you believe will help you heal?" c- What prescriptions must be strictly followed to get well. d- You must believe that the medications will help you.

b- "What practices do you believe will help you heal?"

A female client comes to the clinic complaining of fatigue and inability to sleep because she is the full-time caretaker for 22-year-old son who was paralyzed by a motor vehicle collision. She adds that her husband left her because he says he can't take her behavior any more since all she does is care for their son. What intervention should the nurse implement? a- Schedule a home visit in the afternoon to assess the son and client role as caregiver. b- Acknowledge the client's stress and suggest that she consider respite care. c- Provide feedback to the client about her atonement for guilt about her son's impairment. d- Teach the client to problem-solve for herself and establish her own priorities.

b- Acknowledge the client's stress and suggest that she consider respite care.

The nurse provides feeding tube instructions to the wife of a client with end stage cancer. The client's wife performs a return demonstration correctly, but begins crying and tells the nurse, "I just don't think I can do this every day." The nurse should direct further teaching strategies toward which learning domain? a- Cognitive b- Affective c- Comprehension d- Psychomotor

b- Affective

An adult female client is admitted to the psychiatric unit with a diagnosis of major depressive...medication therapy, the nurse notices the client has more energy, is giving her belongings...mood. Which intervention is best for the nurse to implement a- Support the client by telling her what wonderful progress she is making. b- Ask the client if she has had any recent thoughts of harming herself. c- Reassure the client that the antidepressant drugs are apparently effective d- Tell the client to keep her belongings because she will need hem at discharge.

b- Ask the client if she has had any recent thoughts of harming herself.

The nurse is teaching a client with atrial fibrillation about a newly prescribed medication, dronedarone. Which information should the nurse include in client interactions? (Select all that apply) a- Discontinue medication when palpitation subside. b- Avoid eating grapefruit or drinking grapefruit juice. c- Report changes in the use of daily supplements d- Notify your health care provider if your skin looks yellow e- If a dose is missed, the next dose should be double.

b- Avoid eating grapefruit or drinking grapefruit juice. c- Report changes in the use of daily supplements d- Notify your health care provider if your skin looks yellow

A 2-year-old girl is brought to the clinic for a routine assessment and all findings are within the normal limits. However, the mom expresses concern over her daughter's protruding abdomen and tells the nurse that she is worry that her child is becoming overweight. How should the nurse respond to the mother's comment? a- Tell the mother to keep a twenty- four-hour food diary for the child. b- Explain that a protruding abdomen is typical for toddlers. c- Discuss way to increase the child's daily activity level d- Ask the mother is she has weight problems when she was a child.

b- Explain that a protruding abdomen is typical for toddlers.

A male client who was hit by a car while dodging through traffic is admitted to the emergency department with intracranial pressure (ICP). A computerized tomography (CT) scan reveals an intracranial bleed. After evacuation of hematoma, postoperative prescription include: intubation with controlled mechanical ventilation to PaCO2...what is the pathophysiological basis for this ventilator settings? a- Hypoxemia reduces ICP. b- Hypocapnea reduces ICP. c- Hyperventilation reduces need for temperature control. d- Controlled ventilation reduces need for oxygen to brain.

b- Hypocapnea reduces ICP.

The nurse is reviewing a client's electrocardiogram and determines the PR interval (PRI) is prolonged. What does this finding indicate? a- Initiation of the impulses from a location outside the SA node b- Inability of the SA node to initiate an impulse at the normal rate c- Increased conduction time from the SA node to the AV junction d-Interference with the conduction through one or both ventricles

b- Inability of the SA node to initiate an impulse at the normal rate

During the intraoperative phase of care, the circulating nurse observes that the client is not adequately client's privacy. What is the best initial nursing action for the nurse to implement? a- Document the observation in the client's medical record b- Instruct the scrub nurse to re-drape the client c- Ensure that the client in unaware of the surrounding. d- Consult with operating room manager.

b- Instruct the scrub nurse to re-drape the client

A client who developed syndrome of inappropriate antidiuretic hormone (SIADH) associated with small carcinoma of the lung is preparing for discharge. When teaching the client about self-management with demeclocycline (Declomycin), the nurse should instruct the client to report which condition to the health care provider? a- Insomnia b- Muscle cramping c- Increase appetite d- Anxiety.

b- Muscle cramping

A male client recently released from a correctional facility arrives at the clinic with a cough, fever, and chills. His history reveals active tuberculosis (TB) 10 years ago. What action should the nurse implement? (Select all that apply) a- Administer a PPD test b- Schedule the client for the chest radiograph c- Obtain sputum for acid fast bacillus (AFB) testing d- Place a mask on the client until he is moved to isolation. e- Send the client home with instructions for a prescribe antibiotic.

b- Schedule the client for the chest radiograph c- Obtain sputum for acid fast bacillus (AFB) testing d- Place a mask on the client until he is moved to isolation.

In caring for a client receiving the amino glycoside antibiotic gentamicin, it is most important for the nurse to monitor which diagnostic test? a- Urinalysis b- Serum creatinine c- Serum osmolarity d- Liver enzymes.

b- Serum creatinine

A client who is recently diagnosed with type 2 diabetes mellitus (DM) ask the nurse how this type of diabetes leads to high blood sugar. What Pathophysiology mechanism should the nurse explain about the occurrence of hyperglycemia in those who have type 2 DM? a- Immune antibodies attack pancreatic beta cells resulting in no insulin b- The body cells develop resistance to the action of insulin. c- Body organs produce less insulin and more glucagon d- The liver produces excess glucose in response to excess glycotrophic hormones

b- The body cells develop resistance to the action of insulin.

After removing a left femoral arterial sheath, which assessment finding warrant immediately interventions by the nurse? (Select all that applied.) a- Tenderness over insertion b- Unrelieved back and flank pain. c- Cool and pale left leg and foot. d- Left groin egg-size hematoma. e- Quarter size red drainage at site.

b- Unrelieved back and flank pain. c- Cool and pale left leg and foot. d- Left groin egg-size hematoma.

The nurse weighs a 6-month-old infant during a well-baby check-up and determines that the baby's weight has tripled compared to the birth weight of 7 pounds 8 ounces. The mother asks if the baby is gaining enough weight. What response should the nurse offer? a- Your baby is gaining weight right on schedule b- What food does your baby usually eat in a normal day? c- The baby is below the normal percentile for weight gain d- What was the baby's weight at the last well-baby clinic visit

b- What food does your baby usually eat in a normal day?

When conducting diet teaching for a client who was diagnosed with hypoparathyroidism, which foods should the nurse encourage the client to eat? a- Nuts b- Yogurt. c- Fresh turkey d- Fresh chicken e- Processed cheese.

b- Yogurt. e- Processed cheese.

After several hours of non-productive coughing, a client presents to the emergency room complaining of chest tightness and shortness of breath. History includes end stage chronic obstructive pulmonary disease (COPD) and diabetes mellitus. While completing the pulmonary assessment, the nurse hears wheezing and poor air movement bilaterally. Which actions should the nurse implement? (Select all that apply.) a. Apply oxygen via nasal cannula b. Administer PRN nebulizer treatment. c. Obtain 12 lead electrocardiogram. d. Monitor continuous oxygen saturation. e. Give PRN dose of regular insulin

b. Administer PRN nebulizer treatment. c. Obtain 12 lead electrocardiogram. d. Monitor continuous oxygen saturation.

A client was admitted to the cardiac observation unit 2 hours ago complaining of chest pain. On admission, the client's EKG showed bradycardia, ST depression, but no ventricular ectopy. The client suddenly reports a sharp increase in pain, telling the nurse, "I feel like an elephant just stepped on my chest" The EKG now shows Q waves and ST segment elevations in the anterior leads. What intervention should the nurse perform? a. Increase the peripheral IV flow rate to 175 ml/hr to prevent hypotension and shock b. Administer prescribed morphine sulfate IV and provide oxygen at 2 L/min per nasal cannula. c. Obtain a stat 12 lead EKG and perform a venipuncture to check cardiac enzymes levels. d. Notify the healthcare provider of the client's increase chest pain a call for the defibrillator crash cart.

b. Administer prescribed morphine sulfate IV and provide oxygen at 2 L/min per nasal cannula.

A client is scheduled to receive an IW dose of ondansetron (Zofran) eight hours after receiving chemotherapy. The client has saline lock and is sleeping quietly without any restlessness. The nurse caring for the client is not certified in chemotherapy administration. What action should the nurse take? a. Ask a chemotherapy-certified nurse to administer the Zofran b. Administer the Zofran after flushing the saline lock with saline c. Hold the scheduled dose of Zofran until the client awakens d. Awaken the client to assess the need for administration of the Zofran.

b. Administer the Zofran after flushing the saline lock with saline

A client receives a new prescription for simvastatin (Zocor) 5 mg PO daily at bedtime. What action should the nurse take? a. Provide a bedtime snack to be eaten before taking the medication. b. Administer the medication as prescribed with a glass of water c. Contact the prescriber about changing the time of administration. d. Check the client's blood pressure prior to administering the med.

b. Administer the medication as prescribed with a glass of water

A female client with chronic urinary retention explains double voiding technique to the nurse by stating she voids partially, hold the remaining urine in her bladder for three minutes, then voids again to empty her bladder fully. How should the nurse respond? a. Affirm that the client is effectively performing the double voiding. b. Advise the client to empty her bladder fully when she first voids c. Suggest that the client drink water between the two voiding. d. Explain that Kegel exercises help promote full bladder empty.

b. Advise the client to empty her bladder fully when she first voids

A nurse is preparing to feed a 2-month-old male infant with heart failure who was born with congenital heart defect. Which intervention should the nurse implement? a. Feed the infant when he cries b. Allow the infant to rest before feeding c. Weigh before and after feeding. d. Insert a nasogastric feeding tube.

b. Allow the infant to rest before feeding

An adult who is 5 feet 5 inches (165.1 cm) tall and weighs 90 lb. (40.8 Kg) is admitted with a diagnosis of chronic anorexia. The client receives a regular diet for 2 days, and the client's medical records indicates that 100% of the diet provided has been consumed. However the client's weight on the third day morning after admission is 89 lb. (40.4 Kg). What action should the nurse implement? a. Examine the client's room for hidden food. b. Assign staff to monitor what the client eats. c. Ask the client if the food provided is being eaten or discarded. d. Provide the client with a high calorie diet.

b. Assign staff to monitor what the client eats.

Several months after a foot injury, and adult woman is diagnosed with neuropathic pain. The client describes the pain as severe and burning and is unable to put weight on her foot. She asks the nurse when the pain will "finally go away." How should the nurse respond? a. Explain the healing from injury can take many months b. Assist the client in developing a goal of managing the pain. c. Encourage the client to verbalize her fears about the pain d. Complete an assessment of the client's functional ability.

b. Assist the client in developing a goal of managing the pain.

While receiving a male postoperative client's staples de nurse observe that the client's eyes are closed and his face and hands are clenched. The client states, "I just hate having staples removed". After acknowledgement the client's anxiety, what action should the nurse implement? a. Encourage the client to continue verbalize his anxiety b. Attempt to distract the client with general conversation c. Explain the procedure in detail while removing the staples d. Reassure the client that this is a simple nursing procedure.

b. Attempt to distract the client with general conversation

The mother of the 12- month-old with cystic fibrosis reports that her child is experiencing increasing congestion despite the use of chest physical therapy (CPT) twice a day, and has also experiences a loss of appetite. What instruction should the nurse provide? a. Perform CPT after meals to increase appetite and improve food intake. b. CPT should be performed more frequently, but at least an hour before meals. c. Stop using CPT during the daytime until the child has regained an appetite. d. Perform CPT only in the morning, but increase frequency when appetite improves.

b. CPT should be performed more frequently, but at least an hour before meals.

A postpartal client complains that she has the urge to urinate every hour but is only able to void a small amount. What interventions provides the nurse with the most useful information? a. Initiate a perineal pad count b. Catheterize for residual urine after next voiding c. Assess for a perineal hematoma d. Determine the client's usual voiding pattern

b. Catheterize for residual urine after next voiding

The nurse is completing a head to be assessment for a client admitted for observation after falling out of a tree. Which finding warrants immediate intervention by the nurse? a. Sluggish pupillary response to light b. Clear fluid leaking from the nose. c. Complaint of severe headache d. Periorbital ecchymosis of right eye.

b. Clear fluid leaking from the nose.

The nurse is changing a client's IV tubing and closes the roller clamp on the new tubing setup when the bag of solution is....which action should the nurse take to ensure adequate filling of the drip chamber? a. Lower the IV bag to a flat surface b. Compress the drip chamber c. Open the roller clamp d. Squeeze the bag of IV solution

b. Compress the drip chamber

A client with osteoporosis related to long-term corticosteroid therapy receives a prescription for calcium carbonate. Which client's serum laboratory values requires intervention by the nurse? a. Total calcium 9 mg/dl (2.25 mmol/L SI) b. Creatinine 4 mg/dl (354 micromol/L SI) c. Phosphate 4 mg/dl (1.293 mmol/L SI) d. Fasting glucose 95 mg/dl (5.3 mmol/L SI)

b. Creatinine 4 mg/dl (354 micromol/L SI)

An increased number of elderly persons are electing to undergo a new surgical procedure which cures glaucoma. What effect is the nurse likely to note as a result of this increases in glaucoma surgeries? a. Decrease morbidity in the elderly population b. Decrease prevalence of glaucoma in the population. c. Increase mortality in the elderly population d. Increased incidence of glaucoma in the population.

b. Decrease prevalence of glaucoma in the population.

The nurse is preparing to administer a histamine 2-receptor antagonist to a client with peptic ulcer disease. What is the primary purpose of this drug classification? a. Neutralize hydrochloric (HCI) acid in the stomach b. Decreases the amount of HCL secretion by the parietal cells in the stomach c. Inhibit action of acetylcholine by blocking parasympathetic nerve endings. d. Destroys microorganisms causing stomach inflammation.

b. Decreases the amount of HCL secretion by the parietal cells in the stomach

After a colon resection for colon cancer, a male client is moaning while being transferred to the Postanesthesia Care Unit (PACU). Which intervention should the nurse implement first? a. Assess the client's dressing for bleeding b. Determine client's pulse, blood pressure, and respirations c. Administer a PRN dose of IV Morphine d. Check the client's orientation to time and place.

b. Determine client's pulse, blood pressure, and respirations

An adult male who lives alone is brought to the Emergency Department by his daughter who is unresponsive. Initial assessment indicated that the client has minimal respiratory effort, and his pupils are fixed and dilated. At the daughter's request, the client is intubated and... Which nursing intervention has the highest priority? a. Offer to notify the client's minister of his condition. b. Determine if the client has an executed living will c. Provide the family with information about palliative care d. Explore the possibility of organ donation with the family.

b. Determine if the client has an executed living will

A mother runs into the emergency department with s toddler in her arms and tells the nurse that her child got into some cleaning products. The child smells of chemicals on hands, face, and on the front of the child's clothes. After ensuring the airway is patent, what action should the nurse implement first? a. Call poison control emergency number. b. Determine type of chemical exposure. c. Obtain equipment for gastric lavage. d. Assess child for altered sensorium.

b. Determine type of chemical exposure.

During a routine clinic visit, an older female adult tells the nurse that she is concerned that the flu season is coming soon, but is reluctant to obtain the vaccination. What action should the nurse take first? a. Determine when the client last had an influenza vaccination. b. Discuss the concerns expressed by the client about the vaccination. c. Ask about any recent exposure to persons with the flu or other viruses. d. Review the informed consent form for the vaccination with the client.

b. Discuss the concerns expressed by the client about the vaccination.

A client in the emergency center demonstrates rapid speech, flight of ideas, and reports sleeping only three hours during the past 48h. Based on these finding, it is most important for the nurse to review the laboratory value for which medication? a. Olanzapine b. Divalproex. c. Lorazepam d. Fluoxetine

b. Divalproex.

After a sudden loss of consciousness, a female client is taken to the ED and initial assessment indicate that her blood glucose level is critically low. Once her glucose level is stabilized, the client reports that was recently diagnosed with anorexia nervosa and is being treated at an outpatient clinic. Which intervention is more important to include in this client's discharge plan? a. Describe the signs and symptoms of hypoglycemia. b. Encourage a low-carbohydrate and high-protein diet c. Reinforce the need to continue outpatient treatment d. Suggest wearing a medical alert bracelet at all time.

b. Encourage a low-carbohydrate and high-protein diet

The nurse plans to use an electronic digital scale to weight a client who is able to stand. Which intervention should the nurse implement to ensure that measurement of the client's weight is accurate? a. Ask the client to remove shoes before stepping on the scale b. Ensure that the scale is calibrated before a weight is obtained. c. Slide the balancing weights until the scale is at zero. d. Compare client's weight at various time of the day.

b. Ensure that the scale is calibrated before a weight is obtained.

A male client with impaired renal function who takes ibuprofen daily for chronic arthritis...gastrointestinal (GI) bleeding. After administering IV fluids and a blood transfusion, his blood pressure is 100/70, and his renal output is 20 ml / hour. Which intervention should the nurse include in hours? a. Maintain the client NPO during the diuresis phase b. Evaluate daily serial renal laboratory studies for progressive elevations. c. Observe the urine character for sedimentation and cloudy appearance. d. Monitor for onset of polyuria greater than 150ml/hr.

b. Evaluate daily serial renal laboratory studies for progressive elevations.

The nurse observes an unlicensed assistive personnel (UAP) using an alcohol-based clean...tray to the room. The UAP rub both hands thoroughly for 2 minutes while standing at the...should the nurse take? a. Encourage the UAP to remain in the client's room, until completed b. Explain that the hand rub can be completed in less than 2 minutes. c. Inform the UAP that handwashing helps to promote better asepsis. d. Determine why the UAP was not wearing gloves in the client's room

b. Explain that the hand rub can be completed in less than 2 minutes.

The pathophysiological mechanism are responsible for ascites related to liver failure? (Select all that apply) a. Bleeding that results from a decreased production of the body's clotting factors b. Fluid shifts from intravascular to interstitial area due to decreased serum protein c. Increased hydrostatic pressure in portal circulation increases fluid shifts into abdomen d. Increased circulating aldosterone levels that increase sodium and water retention e. Decreased absorption of fatty acids in the duodenum leading to abdominal distention.

b. Fluid shifts from intravascular to interstitial area due to decreased serum protein c. Increased hydrostatic pressure in portal circulation increases fluid shifts into abdomen d. Increased circulating aldosterone levels that increase sodium and water retention

The nurse is assisting the mother of a child with phenylketonuria (PKU) to select foods that are in keeping with the child's dietary restrictions. Which foods are contraindicated for this child? a. Wheat products b. Foods sweetened with aspartame. c. High fat foods d. High calories foods.

b. Foods sweetened with aspartame.

In caring for a client with Cushing syndrome, which serum laboratory value is most important for the nurse to monitor? a. Lactate b. Glucose c. Hemoglobin d. Creatinine

b. Glucose

A female client presents in the Emergency Department and tells the nurse that she was raped last night. Which question is most important for the nurse to ask? a. Does she know the person who raped her? b. Has she taken a bath since the raped occurred? c. Is the place where she lived a safe place? d. Did she report the rape to the police Department?

b. Has she taken a bath since the raped occurred?

A client presents in the emergency room with right-sided facial asymmetry. The nurse asks the client to perform a series of movements that require use of the facial muscles. What symptoms suggest that the client has most likely experience a Bell's palsy rather than a stroke? a. Slow onset of facial drooping associated with headache b. Inability to close the affected eye, raise brow, or smile c. A flat nasolabial fold on the right resulting in facial asymmetry. d. Drooling is present on right side of the mouth, but not on the left.

b. Inability to close the affected eye, raise brow, or smile

Assessment by the home health nurse of an older client who lives alone indicates that client has chronic constipations. Daily medications include furosemide for hypertension and heart failure and laxatives. To manage the client's constipation, which suggestions should the nurse provide? (Select all that apply) a. Decrease laxative use to every other day and use oil retention enemas as needed. b. Include oatmeal with stewed pruned for breakfast as often as possible. c. Increase fluid intake by keeping water glass next to recliner. d. Recommend seeking help with regular shopping and meal preparation. e. Report constipation to healthcare provider related to cardiac medication side effects.

b. Include oatmeal with stewed pruned for breakfast as often as possible. c. Increase fluid intake by keeping water glass next to recliner. d. Recommend seeking help with regular shopping and meal preparation.

An older adult female admitted to the intensive care unit (ICU) with a possible stroke is intubated with ventilator setting of tidal volume 600, PlO2 40%, and respiratory rate of 12 breaths/minute. The arterial blood gas (ABG) results after intubation are PH 7.31. PaCO2 60, PaO2 104, SPO2 98%, HCO3 23. To normalize the client's ABG finding, which action is required? a. Report the results to the healthcare provider. b. Increase ventilator rate. c. Administer a dose of sodium carbonate. d. Decrease the flow rate of oxygen.

b. Increase ventilator rate.

The nurse observes an unlicensed assistive personnel (UAP) positioning a newly admitted client who has a seizure disorder. The client is supine and the UAP is placing soft pillows along the side rails. What action should the nurse implement? a. Ensure that the UAP has placed the pillows effectively to protect the client. b. Instruct the UAP to obtain soft blankets to secure to the side rails instead of pillows. c. Assume responsibility for placing the pillows while the UAP completes another task. d. Ask the UAP to use some of the pillows to prop the client in a side lying position.

b. Instruct the UAP to obtain soft blankets to secure to the side rails instead of pillows.

The nurse is interacting with a female client who is diagnosed with postpartum depression. Which finding should the nurse document as an objective signs of depression? (Select all that apply) a. Report feeling sad b. Interacts with a flat affect. c. Avoids eye contact. d. Has a disheveled appearance. e. Express suicidal thoughts.

b. Interacts with a flat affect. c. Avoids eye contact. d. Has a disheveled appearance.

The mother of a one-month-old boy born at home brings the infant to his first well...was born two weeks after his due date, and that he is a "good, quiet baby" who almost... hypothyroidism, what question is most important for the nurse to ask the mother? a. Has your son had any immunizations yet? b. Is your son sleepy and difficult to feed? c. Are you breastfeeding or bottle feeding your son? d. Were any relatives born with birth defects?

b. Is your son sleepy and difficult to feed?

The nurse is teaching a client how to perform colostomy irrigations. When observing the client's return demonstration, which action indicated that the client understood the teaching? a. Turns to left the side to instill the irrigating solution into the stoma b. Keeps the irrigating container less than 18 inches above the stoma c. Instills 1,200 ml of irrigating solution to stimulate bowel evacuation d. Inserts irrigating catheter deeper into stoma when cramping occurs

b. Keeps the irrigating container less than 18 inches above the stoma

A young adult female presents at the emergency center with acute lower abdominal pain. Which assessment finding is most important for the nurse to report to the healthcare provider? a. Pain scale rating at 9 on a 0-10 scale b. Last menstrual period was 7 weeks ago c. Reports white curdy vaginal discharge d. History of irritable bowel syndrome IBS

b. Last menstrual period was 7 weeks ago

A client with Alzheimer's disease falls in the bathroom. The nurse notifies the charge nurse and completes a fall follow-up assessment. What assessment finding warrants immediate intervention by the nurse? a. Urinary incontinence b. Left forearm hematoma c. Disorientation to surroundings d. Dislodge intravenous site

b. Left forearm hematoma

During a 26-week gestation prenatal exam, a client reports occasional dizziness...What intervention is best for the nurse to recommend to this client? a. Elevate the head with two pillows while sleeping b. Lie on the left or right side when sleeping or resting. c. Increase intake of foods that are high in iron d. Decrease the amount of carbohydrates in the diet.

b. Lie on the left or right side when sleeping or resting.

The daughter of an older female client tells the clinic nurse that she is no longer able to care for her mother since her mother has lost the ability to perform activities of daily living (ADLs) due to aging. Which options should the nurse discuss with the daughter? a. Home hospice agency b. Long-term care facility c. Rehabilitation facility d. Independent senior apartment e. Home health agency

b. Long-term care facility e. Home health agency

When providing diet teaching for a client with cholecystitis, which types of food choices the nurse recommend to the client? a. High protein b. Low fat c. Low sodium d. High carbohydrate.

b. Low fat

A client who had an emergency appendectomy is being mechanically ventilated, and soft wrist restrain are in place to prevent self extubation. Which outcome is most important for the nurse to include in the client's plan of care? a. Understand pain management scale b. Maintain effective breathing patterns c. Absence of ventilator associated pneumonia d. No injuries refer to soft restrains occur

b. Maintain effective breathing patterns

A nurse is planning discharge care for a male client with metastatic cancer. The client tells the nurse that he plans to return to work despite pain, fatigue, and impending death. Which goals is most important to include in this client's plan of care? a. Implements decisions about future hospices services within the next 3 months. b. Maintaining pain level below 4 when implementing outpatient pain clinic strategies. c. Request home health care if independence become compromised for 5 days. d. Arranges for short term counseling stressors impact work schedule for 2 weeks.

b. Maintaining pain level below 4 when implementing outpatient pain clinic strategies.

A client with type 2 diabetes mellitus is admitted for antibiotic treatment for a leg ulcer. To monitor the client for the onset of hyperosmolar hyperglycemic nonketotic syndrome (HHNS), what actions should the nurse take? (Select all that apply) a. Check urine for ketones b. Measure blood glucose c. Monitor vital signs d. Assessed level of consciousness e. Obtain culture of wound

b. Measure blood glucose c. Monitor vital signs d. Assessed level of consciousness

Ten years after a female client was diagnosed with multiple sclerosis (MS), she is admitted to a community palliative care unit. Which intervention is most important for the nurse to include in the client's plan of care? a. Allow the family to visit whenever they wish b. Medicate as needed for pain and anxiety. c. Allow client to participate in care provided d. Maintain quiet, low lighting environment

b. Medicate as needed for pain and anxiety.

The nurse is planning care for a client admitted with a diagnosis of pheochromocytoma. Which intervention has the highest priority for inclusion in this client's plan of care? a. Record urine output every hour b. Monitor blood pressure frequently c. Evaluate neurological status d. Maintain seizure precautions

b. Monitor blood pressure frequently

Which intervention should the nurse include in the plan of care for a client with leukocytosis? a. Avoid intramuscular injections b. Monitor temperature regularly c. Assess skin for petechiae or bruising d. Implement protective isolation measures

b. Monitor temperature regularly

A client with superficial burns to the face, neck, and hands resulting from a house fire... which assessment finding indicates to the nurse that the client should be monitored for carbon monoxide...? a. Expiratory stridor and nasal flaring b. Mucous membranes cherry red color c. Carbonaceous particles in sputum d. Pulse oximetry reading of 80 percent

b. Mucous membranes cherry red color

A client with a history of cirrhosis and alcoholism is admitted with severe dyspnea and ascites. Which assessment finding warrants immediate intervention by the nurse? a. Jaundice skin tone b. Muffled heart sounds c. Pitting peripheral edema d. Bilateral scleral edema

b. Muffled heart sounds

The nurse is preparing a teaching plan for an older female client diagnosed with osteoporosis. What expected outcome has the highest priority for this client? a. Identifies 2 treatments for constipation due to immobility. b. Names 3 home safety hazards to be resolve immediately. c. State 4 risk factors for the development of osteoporosis. d. Lists 5 calcium-rich foods to be added to her daily diet.

b. Names 3 home safety hazards to be resolve immediately.

One day following an open reduction and internal fixation of a compound fracture of the leg, a male client complains of "a tingly sensation" in his left foot. The nurse determines the client's left pedal pulses are diminished. Based on these finding, what is the client's greatest risk? a. Reduce pulmonary ventilation and oxygenation related to fat embolism. b. Neurovascular and circulation compromise related to compartment syndrome. c. Wound infection and delayed healing to fractured bone protrusion. d. Venous stasis and thrombophlebitis related to postoperative immobility.

b. Neurovascular and circulation compromise related to compartment syndrome.

The nurse is caring for several clients on a telemetry unit. Which client should the nurse assess first? The client who is demonstrating? a. A paced rhythm with 100% capture after pacemaker replacement b. Normal sinus rhythm and complaining of chest pain c. Atrial fibrillation with congestive heart failure and complaining of fatigue d. Sinus tachycardia 3 days after a myocardial infarction

b. Normal sinus rhythm and complaining of chest pain

An older adult client with heart failure (HF) develops cardiac tamponade. The client has muffled, distant, heart sounds, and is anxious and restless. After initiating oxygen therapy and IV hydration, which intervention is most important for the nurse to implement? a. Observe neck for jugular vein distention b. Notify healthcare provider to prepare for pericardiocentesis c. Asses for paradoxical blood pressure d. Monitor oxygen saturation (Sp02) via continuous pulse oximetry

b. Notify healthcare provider to prepare for pericardiocentesis

After the risk and benefits of having a cardiac catheterization are reviewed by the healthcare provider, an older adult with unstable angina is scheduled for the procedure. When the nurse presents the consent form for signature, the client asks how the wires will keep a heart heating during the procedure. What action should the nurse take? a. Explain the procedure again in detail and clarify any misconceptions. b. Notify the healthcare provider of the client's lack of understanding. c. Call the client's next of kin and have them provide verbal consent. d. Postpone the procedure until the client understands the risk and benefits.

b. Notify the healthcare provider of the client's lack of understanding.

A client is admitted for type 2 diabetes mellitus (DM) and chronic Kidney disease (CKD)... which breakfast selection by the client indicates effective learning? a. Scrambled eggs, bacon, one slice of whole wheat toast with butter and jam. b. Oatmeal with butter, artificial sweetener, and strawberries, and 6 ounces' coffee. c. Banana pancake with maple syrup, sausage links, half grapefruit, and low -fat milk d. Orange juice, yogurt with berries, cold cereal with milk, bran muffin with margarine.

b. Oatmeal with butter, artificial sweetener, and strawberries, and 6 ounces' coffee.

53- A male client with a long history of alcoholism is admitted because of mild confusion and fine motor tremors. He reports that he quit drinking alcohol and stopped smoking cigarettes one month ago after his brother died of lung cancer. Which intervention is most important for the nurses to include in the client's plan of care? a. Determine client's level current blood alcohol level. b. Observe for changes in level of consciousness. c. Involve the client's family in healthcare decisions. d. Provide grief counseling for client and his family.

b. Observe for changes in level of consciousness.

The nurse is teaching a male adolescent recently diagnosed with type 1 diabetes mellitus (DM) about self-injecting insulin. Which approach is best for the nurse to use to evaluate do you effectiveness of the teaching? a. Ask the adolescent to describe his level of comfort with injecting himself with insulin. b. Observe him as he demonstrates self-injection technique in another diabetic adolescent c. Have the adolescent list the procedural steps for safe insulin administration. d. Review his glycosylated hemoglobin level 3 months after the teaching session.

b. Observe him as he demonstrates self-injection technique in another diabetic adolescent

An elderly client seems confused and reports the onset of nausea, dysuria, and urgency with incontinence. Which action should the nurse implement a. Auscultate for renal bruits b. Obtain a clean catch mid-stream specimen c. Use a dipstick to measure for urinary ketone d. Begin to strain the client's urine.

b. Obtain a clean catch mid-stream specimen

The father of 4-year-old has been battling metastatic lung cancer for the past 2 years. After discussing the remaining options with his healthcare provider, the client requests that all treatment stop and that no heroic measures be taken to save his life. When the client is transferred to the palliative care unit, which action is most important for the nurse working on the palliative care unit to take in facilitating continuity of care? a. Reassure the client that his child will be allowed to visit b. Obtain a detailed report from the nurse transferring the client. c. Mark the chart with client's request for no heroic measure d. Provide the client whitening information about end-of-life care

b. Obtain a detailed report from the nurse transferring the client.

A client with pneumonia has an IV of lactated ringer's solution infusing at 30ml/hr current labor....sodium level of 155 mEq/L, a serum potassium level of 4mEq/L.... what nursing intervention is most important? a.Provide a high-potassium snack, such as bananas. b. Obtain a prescription to increase the IV rate c. Administer the next scheduled dose of antibiotic d. Review the report of the most recent chest x-ray.

b. Obtain a prescription to increase the IV rate

A client with a postoperative wound that eviscerated yesterday has an elevated temperature...most important for the nurse to implement? a. Initiate contact isolation b. Obtain a wound swab for culture and sensitivity c. Assess temperature q4 hours d. Use alcohol-based solutions for hand hygiene.

b. Obtain a wound swab for culture and sensitivity

Following a motor vehicle collision, an adult female with a ruptured spleen and a blood pressure of 70/44, had an emergency splenectomy. Twelve hours after the surgery, her urine output is 25 ml/hour for the last two hours. What pathophysiological reason supports the nurse's decision to report this finding to the healthcare provider a. This output is not sufficient to cleat nitrogenous waste b. Oliguria signals tubular necrosis related to hypoperfusion c. Low urine output puts the client at risk for fluid overload d. An increased urine output is expected after splenectomy

b. Oliguria signals tubular necrosis related to hypoperfusion

The nurse is assessing a postpartum client who is 36 hours post-delivery. Which finding should the nurse report to the healthcare provider? a. White blood count of 19,000 mm3 b. Oral temperature of 100.6 F c. Fundus deviated to the right side d. Breasts are firm when palpated

b. Oral temperature of 100.6 F

Which intervention is most important for the nurse to include in the plan of care for an older woman with osteoporosis? a. Evaluate the client's orientation to time and place b. Place the client on fall precautions c. Encourage the client to drink milk with meals d. Assess the client's breath sounds daily.

b. Place the client on fall precautions

A client who underwent an uncomplicated gastric bypass surgery is having difficult with diet management. What dietary instruction is most important for the nurse to explain to the client? a. Chew food slowly and thoroughly before attempting to swallow b. Plan volume-controlled evenly-space meal thorough the day c. Sip fluid slowly with each meal and between meals d. Eliminate or reduce intake fatty and gas forming food

b. Plan volume-controlled evenly-space meal thorough the day

The nurse finds a client at 33 weeks gestation in cardiac arrest. What adaptation to cardiopulmonary resuscitation (CPR) should the nurse implement? a. Apply oxygen by mask after opening the airway b. Position a firm wedge to support pelvis and thorax at 30-degree tilt. c. Give continuous compression with a ventilation ratio at 20:3 d. Apply less compression force to reduce aspiration

b. Position a firm wedge to support pelvis and thorax at 30-degree tilt.

While assessing a radial artery catheter, the client complains of numbness and pain distal to the insertion site. What interventions should the nurse implement? a. Determine of aspirin was given prior to radial artery catheter insertion. b. Promptly remove the arterial catheter from the radial artery. c. Irrigate the arterial line using a syringe with sterile saline d. Administer a PRN analgesic and assess numbness in 30 mints

b. Promptly remove the arterial catheter from the radial artery.

The nurse is assessing a client with a small bowel obstruction who was hospitalized 24 hours ago. Which assessment finding should the nurse report immediately to the healthcare provider? a. Hypoactive bowel sounds in the lower quadrant. b. Rebound tenderness in the upper quadrants. c. Tympani with percussion of the abdomen. d. Light colors gastric aspirate via the nasogastric tube.

b. Rebound tenderness in the upper quadrants.

A client with a history of dementia has become increasingly confused at night and is picking at an abdominal surgical dressing and the tape securing the intravenous (IV) line. The abdominal dressing is no longer occlusive, and the IV insertion site is pink. What intervention should the nurse implement? a. Replace the IV site with a smaller gauge. b. Redress the abdominal incision c. Leave the lights on in the room at night. d. Apply soft bilateral wrist restraints.

b. Redress the abdominal incision

The nurse is reinforcing home care instructions with a client who is being discharged following...prostate (TURP). Which intervention is most important for the nurse to include in the client... a. Avoid strenuous activity for 6 weeks b. Report fresh blood in the urine. c. Take acetaminophen for fever 101 d. Consume 6 to 8 glasses of water daily.

b. Report fresh blood in the urine.

A client with hypertension receives a prescription for enalapril, an angiotensin... instruction should the nurse include in the medication teaching plan? a. Increase intake of potassium-rich foods b. Report increased bruising of bleeding c. Stop medication if a cough develops d. Limit intake of leafy green vegetables

b. Report increased bruising of bleeding

An older adult male who had an abdominal cholecystectomy has become increasingly confused and disoriented over the past 24 hours. He is found wandering into another client's room and is return to his room by the unlicensed assistive personnel (UAP). What actions should the nurse take? (Select all that apply). a. Apply soft upper limb restrains and raise all four bed rails b. Report mental status change to the healthcare provider c. Assess the client's breath sounds and oxygen saturation d. Assign the UAP to re-assess the client's risk for falls e. Review the client's most recent serum electrolyte values

b. Report mental status change to the healthcare provider c. Assess the client's breath sounds and oxygen saturation e. Review the client's most recent serum electrolyte values

The nurse enters a client's room and observes the client's wrist restraint secured as seen in the picture. What action should the nurse take? a. Use a full knot to secure the restrain tie. b. Reposition the restraint tie onto the bedframe. c. Raise the button side rail of the client's bed d. Document that the restrain is secured.

b. Reposition the restraint tie onto the bedframe.

To reduce staff nurse role ambiguity, which strategy should the nurse manager implemented? a. Confirm that all the staff nurses are being assigned to equal number of clients. b. Review the staff nurse job description to ensure that it is clear, accurate, and recurrent. c. Assign each staff nurse a turn unit charge nurse on a regular, rotating basis. d. Analyze the amount of overtime needed by the nursing staff to complete assignments.

b. Review the staff nurse job description to ensure that it is clear, accurate, and recurrent.

A client who had a gestational trophoblastic disease (GTD) evacuated 2 days ago is being...18 months-old child and lives in a rural area. Her husband takes the family car to work daily...transportation during the day. What intervention is most important for the nurse to implement? a. Teach a client amount the use of a home pregnancy test. b. Schedule a weekly home visit to draw hCG values. c. Make a 5 week follow- up with healthcare provider d. Begin chemotherapy administration during the first home visit

b. Schedule a weekly home visit to draw hCG values.

When preparing to discharge a male client who has been hospitalized for an adrenal crisis, the client expresses concern about having another crisis. He tells the nurse that he wants to stay in the hospital a few more days. Which intervention should the nurse implement? a. Administer anti-anxiety medication prior to providing discharge instructions b. Schedule an appointment for an out-patient psychosocial assessment. c. Obtain a blood cortisol level after last dose of synthetic ACTH d. Encourage the healthcare provider to delay the client's discharge.

b. Schedule an appointment for an out-patient psychosocial assessment.

An adult client present to the clinic with large draining ulcers on both lower legs that are characteristics of Kaposi's sarcoma lesions. The client is accompanied by two family member. Which action should the nurse take? a. Ask family member to wear gloves when touching the patient b. Send family to the waiting area while the client's history is taking c. Obtain a blood sample to determine is the client is HIV positive d. Complete the head to toes assessment to identify other sign of HIV

b. Send family to the waiting area while the client's history is taking

A male client who is admitted to the mental health unit for treatment of bipolar disorder has a slightly slurred speech pattern and an unsteady gait. Which assessment finding is most important for the nurse to report to the healthcare provider? a. Blood alcohol level of 0.09% b. Serum lithium level of 1.6 mEq/L or mmol/l (SI) c. Six hours of sleep in the past three days. d. Weight loss of 10 pounds (4.5 kg) in past month.

b. Serum lithium level of 1.6 mEq/L or mmol/l (SI)

A client refuses to ambulate, reporting abdominal discomfort and bloating caused by "too much gas buildup" the client's abdomen is distended. Which prescribed PRN medication should the nurse administer? a. Hydrocodone/Acetaminophen (Lortab) b. Simethicone (Mylicon) c. Promethazine (Phenergan) d. Nalbupine (Nubain)

b. Simethicone (Mylicon)

A nurse stops at the site of a motorcycle accident and finds a young adult male lying face down in the road in a puddle of water. It is raining, no one is available to send for help, and the cell phone is in the car about 50 feet away. What action should the nurse take first? a. Examine the victim's body surfaces for arterial bleeding b. Stabilize the victim's neck and roll over to evaluate his status c. Return to the car to call emergency response 911 for help d. Open the airway and initiate resuscitative measures

b. Stabilize the victim's neck and roll over to evaluate his status

While visiting a female client who has heart failure (HF) and osteoarthritis, the home health nurse determines that the client is having more difficulty getting in and out of the bed than she did previously. Which action should the nurse implement first? a. Inquire about an electric bed for the client's home use b. Submit a referral for an evaluation by a physical therapist. c. Explain the usual progression of osteoarthritis and HF d. Request social services to review the client's resources.

b. Submit a referral for an evaluation by a physical therapist.

A male client is having abdominal pain after a left femoral angioplasty and stent, and is asking for additional pain medication for right lower quadrant pain (9/10), two hours ago, he received hydrocodone / acetaminophen 7.5/7.50 mg his vital signs are elevated from reading of a previous hour: temperature 97.8 F, heart rate 102 beats / minute, respiration 20 breaths/minutes. His abdomen is swollen, the groin access site is tender, peripheral pulses are present, but left is greater than right. Preoperatively, clopidrogel was prescribed for a history of previous peripheral stents. Another nurse is holding manual pressure on the femoral arterial access site which may be leaking into the abdomen. What data is needed to make this report complete? a. Client's lungs are clear bilaterally and oxygen saturation is 97% b. Surgeon needs to see client immediately to evaluate the situation c. Left peripheral pulses were present only by Doppler pre-procedure d. Client' history includes multiple back surgeries and chronic pain.

b. Surgeon needs to see client immediately to evaluate the situation

The nurse is caring for a group of clients with the help of a licensed practical nurse (LPN) and an experienced unlicensed assistive personnel (UAP). Which procedures can the nurse delegate to the UAP? (Select all that apply) a. Change a saturated surgical dressing for a client who had an abdominal hysterectomy. b. Take postoperative vital signs for a client who has an epidural following knee arthroplasty c. Start a blood transfusion for client who had a below-the knee amputation. d. Collect a sputum specimen for a client with a fever of unknown origin e. Ambulate a client who had a femoral-popliteal bypass graft yesterday

b. Take postoperative vital signs for a client who has an epidural following knee arthroplasty d. Collect a sputum specimen for a client with a fever of unknown origin e. Ambulate a client who had a femoral-popliteal bypass graft yesterday

A male client returns to the mental health clinic for assistance with his anxiety reaction that is manifested by a rapid heartbeat, sweating, shaking, and nausea while driving over the bay bridge. What action I the treatment plan should the nurse implement? a. Tell the client to drive over the bridge until fear is manageable b. Teach client to listen to music or audio books while driving c. Encourage client to have spouse drive in stressful places. d. Recommend that the client avoid driving over the bridge.

b. Teach client to listen to music or audio books while driving

A client with muscular dystrophy is concerned about becoming totally dependent and is reluctant to call the nurse to assist with activities of daily living (ADLs). To achieve maximum mobility and independence, which intervention is most important for the nurse to include in the client's plan of care? a. Elevate lower extremities while out of bed b. Teach family proper range of motion exercises. c. Maintain proper body alignment when in bed d. Encourage diaphragmatic breathing exercises.

b. Teach family proper range of motion exercises.

One year after being discharged from the burn trauma unit, a client with a history of 40% full-thickness burns is admitted with bone pain and muscle weakness. Which intervention should the nurse include in the clients plan of care a. Encourage Progressive active range of motion b. Teach need for dietary and supplementary vitamin D3 c. Explain the need for skin exposure to sunlight without sunscreen d. Instruct the client to use of muscle strengthening exercises

b. Teach need for dietary and supplementary vitamin D3

A client with polycystic kidney disease (PKD) receiving antibiotics for an infected cyst is experiencing severe pain. What action should the nurse implement? a. Hold the next dose of antibiotic until contacting the healthcare provider b. Teach the client how to use a dry heating pad over the painful area c. Encourage the client to practice pelvic floor exercises every hour d. Assist the client to splint the site by applying an abdominal binder

b. Teach the client how to use a dry heating pad over the painful area

A client who recently underwent a tracheostomy is being prepared for discharge to home. Which instructions is most important for the nurse to include in the discharge plan? a. Explain how to use communication tools. b. Teach tracheal suctioning techniques c. Encourage self-care and independence. d. Demonstrate how to clean tracheostomy site.

b. Teach tracheal suctioning techniques

A client with multiple sclerosis (MS) has decreased motor function after taking a hot bath (Uhthoff's sign). Which pathophysiological mechanism supports this response a. Arterial Constriction b. Temporary vasodilation c. Poor temperature control d. Severe dehydration.

b. Temporary vasodilation

A client with a prescription for "do not resuscitate" (DNR) begins to manifest signs of impending death. After notifying the family of the client's status, what priority action should the nurse implement? a. The impending signs of death should be documented b. The client's need for pain medication should be determined. c. The nurse manager should be updated on the client's status d. The client's status should be conveyed to the chaplain

b. The client's need for pain medication should be determined.

Following breakfast, the nurse is preparing to administer 0900 medications to clients on a medical floor. Which medication should be held until a later time? a. The loop-diuretic furosemide (Lasix) for a client with a serum potassium level of 4.2 mEq/L b. The mucosal barrier, sucralfate (Carafate), for a client diagnosed with peptic ulcer disease. c. The antiplatelet agent aspirin, for a client who is scheduled to be discharged within the hour d. The antifungal nystatin (mycostatin) suspension, for a client who has just brushed his teeth.

b. The mucosal barrier, sucralfate (Carafate), for a client diagnosed with peptic ulcer disease.

What explanation is best for the nurse to provide a client who asks the purpose of using the log-rolling technique for turning? a. working together can decrease the risk for back injury b. The technique is intended to maintain straight spinal alignment. c. Using two or three people increases client safety. d. turning instead of pulling reduces the likelihood of skin damage

b. The technique is intended to maintain straight spinal alignment.

A female client with rheumatoid arthritis (RA) comes to the clinic complaining of joint pain and swelling. The client has been taking prednisone (Deltasone) and ibuprofen (Motrin Extra Strength) every day. To assist the client with self-management of her pain, which information should the nurse obtain? a. Presence of bruising, weakness, or fatigue b. Therapeutic exercise included in daily routine. c. Average amount of protein eaten daily d. Existence of gastrointestinal discomfort

b. Therapeutic exercise included in daily routine.

An elderly client with degenerative joint disease asks if she should use the rubber jar openers that are available. The nurse's response should be based on which information about assistive devices? a. They can contribute to increased dependency b. They decrease the risk for joint trauma c. They promote muscle strength d. They diminish range of motion ability.

b. They decrease the risk for joint trauma

The nurse is explaining the need to reduce salt intake to a client with primary hypertension. What explanation should the nurse provide? a. High salt can damage the lining of the blood vessels b. Too much salt can cause the kidneys to retain fluid c. Excessive salt can cause blood vessels to constrict d. Salt can cause information inside the blood vessels

b. Too much salt can cause the kidneys to retain fluid

The nurse plans to collect a 24- hour urine specimen for a creatinine clearance test. Which instruction should the nurse provide to the adult male client? a. Clearance around the meatus, discard first portion of voiding, and collect the rest in a sterile bottle b. Urinate at specific time, discard the urine, and collect all subsequent urine during the next 24 hours. c. For the next 24 hours, notify the nurse when the bladder is full, and the nurse will collect catheterized specimens. d. Urinate immediately into a urinal, and the lab will collect specimen every 6 hours, for the next 24 hours.

b. Urinate at specific time, discard the urine, and collect all subsequent urine during the next 24 hours.

The nurse is managing the care of a client with Cushing's syndrome. Which interventions should the nurse delegate to the unlicensed assistive personnel (UAP)? (Select all that apply) a. Evaluate the client for sleep disturbances b. Weigh the client and report any weight gain. c. Report any client complaint of pain or discomfort. d. Assess the client for weakness and fatigue e. Note and report the client's food and liquid intake

b. Weigh the client and report any weight gain. c. Report any client complaint of pain or discomfort. e. Note and report the client's food and liquid intake

Which client should the nurse assess frequently because of the risk for overflow incontinence? A client a. Who is bedfast, with increased serum BUN and creatinine levels b. Who is confused and frequently forgets to go to the bathroom c. With hematuria and decreasing hemoglobin and hematocrit levels d. Who has a history of frequent urinary tract infections.

b. Who is confused and frequently forgets to go to the bathroom

A male Korean-American client looks away when asked by the nurse to describe his problem. What is the best initial nursing action? a- Ask social services to dins a Korean interpreter b- Establish direct eye contact with the client. c- Allow several minutes for the client to respond. d- Repeat the question slowly and distinctly.

c- Allow several minutes for the client to respond.

A client present at the clinic with blepharitis. What instructions should the nurse provide for home care? a- Use bilateral eyes patches while sleeping to prevent injury to eyes. b- Wear sunglasses when out of doors to prevent photophobia c- Apply warm moist compresses then gently scrub eyelids with dilute baby shampoo

c- Apply warm moist compresses then gently scrub eyelids with dilute baby shampoo

When five family members arrive at the hospital, they all begin asking the nurse questions regarding the prognosis of their critically ill mother. What intervention should the nurse implement first? a- Include the family in client's care b- Request the chaplain's presence c- Ask the family to identify a specific spokesperson d- Page the healthcare provider to speak with family.

c- Ask the family to identify a specific spokesperson

During the transfer of a client who had major abdominal surgery this morning, the post anesthesia care unit (PACU) nurse reports that the client, who is awake and responsive continues to report pain and nausea after receiving morphine 2 mg IV and ondansetron 4 mg IV 45 mints ago. Which elements of SBAR communication are missing from the report given by the PACU nurse? (Select all that apply) a- Situation b- Background c- Assessment d- Recommendation e- Rationales.

c- Assessment d- Recommendation e- Rationales.

An unlicensed assistive personnel (UAP) assigned to obtain client vital signs reports to the charge nurse that a client has a weak pulse with a rate of 44 beat/ minutes. What action should the charge nurse implement? a- Instruct the UAP to count the client apical pulse rate for sixty seconds b- Determine if the UAP also measured the client's capillary refill time. c- Assign a practical nurse (LPN) to determine if an apical radial deficit is present. d- Notify the health care provider of the abnormal pulse rate and pulse volume.

c- Assign a practical nurse (LPN) to determine if an apical radial deficit is present.

A client with gestational diabetes, at 39 weeks of gestation, is in the second stage of labor. After delivering of the fetal head, the nurse recognizes that shoulder dystocia is occurring. What intervention should the nurse implement first? a- Prepare the client for an emergency cesarean birth b- Encourage the client to move to a hands-and-knees position. c- Assist the client to sharply flex her thighs up again the abdomen. d- Lower the head of the bed an apply suprapubic pressure.

c- Assist the client to sharply flex her thighs up again the abdomen.

The nurse is assessing the emotional status of a client with Parkinson's disease. Which client finding is most helpful in planning goals to meet the client's emotional needs? a- Stares straight ahead without blinking b- Face does not convey any emotion c- Cries frequently during the interview d- Uses a monotone when speaking

c- Cries frequently during the interview

The nurse is triaging victims of a tornado at an emergency shelter. An adult woman who has been wandering and crying comes to the nurse. What action should the nurse take? a- Check the client's temperature, blood sugar, and urine output. b- Transport the client for laboratory client for laboratory test and electrocardiogram (EKG) c- Delegate care of the crying client to an unlicensed assistant d- Send the client to the shelter's nutrient center to obtain water and food.

c- Delegate care of the crying client to an unlicensed assistant

The nurse is presenting information about fetal development to a group of parents with...when discussing cephalocaudal fetal development, which information should the nurse gives the parents? a- set order in fetal development is expected b- Growth normally occurs within one organ at a time c- Development progress from head to rump d- Organ formation is directed by brain development

c- Development progress from head to rump

The nurse reviews the signs of hypoglycemia with the parents of a child with Type I diabetes mellitus. The parents correctly understand signs of hypoglycemia if they include which symptoms? a- Fruity breath odor b- Polyphagia c- Diaphoresis d- Polydipsia

c- Diaphoresis

The nurse is caring a client with NG tube. Which task can the nurse delegate to the UAP? a- Replace the NG tube as prescribed by the healthcare provider b- Secure the NG tube if it slides out of the client's nasal passage c- Disconnect the NG suction so the client can ambulate in the hallway. d- Reconnect the NG suction when the client returns form ambulating.

c- Disconnect the NG suction so the client can ambulate in the hallway.

A woman just received the Rubella vaccine after a delivery of a normal new born, has two children at home, ages 13 months and 3 years. Which instruction is most important to provide to the client? a- Refrain from eating foods containing eggs for 24 hors b- Breast feeding is recommended to prevent ovulation. c- Do not get pregnant for at least 3 months. d- Avoid exposure to the sunlight for 36 hours.

c- Do not get pregnant for at least 3 months.

During the initial newborn assessment, the nurse finds that a newborn's heart rate is irregular. Which intervention should the nurse implement? a- Notify the pediatrician immediately. b- Teach the parents about congenital heart defects. c- Document the finding in the infant's record. d- Apply oxygen per nasal cannula at 3 L/min.

c- Document the finding in the infant's record.

When finding a client sitting on the floor, the nurse calls for help from the unlicensed assistive personnel (UAP). Which task should the nurse ask the UAP to do? a- Check for any abrasions or bruises. b- Help the client to stand. c- Get a blood pressure cuff. d- Report the fall to the nurse-manager.

c- Get a blood pressure cuff.

A 16-year-old male is admitted to the pediatric intensive care unit after being involved in a house fire. He has full thickness burns to his lower torso and extremities. Before a dressing change to his legs, which intervention is most important for the nurse to implement? a- Encourage the parents to stay at the bedside b- Use distraction techniques to reduce pain. c- Maintain strict aseptic technique d- Place a drape over the pubic area.

c- Maintain strict aseptic technique

What is the nurse's priority goal when providing care for a 2-year-old child experiencing seizure... a- Stop the seizure activity b- Decrease the temperature c- Manage the airway d- Protect the body from injury

c- Manage the airway

In determine the client position for insertion of an indwelling urinary catheter, it is most important for the nurse to recognize which client condition? a- High urinary PH b- Abdominal Ascites c- Orthopnea d- Fever.

c- Orthopnea

A client who has been in active labor for 12 hours suddenly tells the nurse that she has a strong urge to have a bowel movement. What action should the nurse take? a- Allow the client to use a bedpan. b- Assist the client to the bathroom c- Perform a sterile vaginal exam d- Explain the fetal head is descending. Perform a sterile vaginal exam

c- Perform a sterile vaginal exam

A nurse receive a shift report about a male client with Obsessive compulsive disorder (OCD). The nurse does morning rounds and reaches the client while he is repeatedly washing the top of the same table. What intervention should the nurse implement? a. Encourage the client to be calm and relax for a little while b. Assist the client to identify stimuli that precipitates the activity. c. Allow time for the behavior and then redirect the clients to other activities. d. Teach the client thought stopping techniques and ways to refocus.

c. Allow time for the behavior and then redirect the clients to other activities.

Based on principles of asepsis, the nurse should consider which circumstance to be sterile? a. One inch- border around the edge of the sterile field set up in the operating room b. A wrapped unopened, sterile 4x4 gauze placed on a damp table top. c. An open sterile Foley catheter kit set up on a table at the nurse waist level d. Sterile syringe is placed on sterile area as the nurse riches over the sterile field.

c. An open sterile Foley catheter kit set up on a table at the nurse waist level

In assessing an older female client with complication associated with chronic obstructive pulmonary disease (COPD), the nurse notices a change in the client's appearance. Her face appears tense and she begs the nurse not to leave her alone. Her pulse rate is 100, and respirations are 26 per min. What is the primary nursing diagnosis? a. Impaired gas exchange related to narrowing of small airways b. Death anxiety related to concern about prognosis c. Anxiety related to fear of suffocation. d. Ineffective coping related to knowledge deficit about COPD

c. Anxiety related to fear of suffocation.

After repositioning an immobile client, the nurse observes an area of hyperemia. To assess for blanching, what action should the nurse take? a. Note the skin color around the area b. Measure the degree of... c. Apply light pressure over the area. d. Palpate the temperature of the area.

c. Apply light pressure over the area.

When should intimate partner violence (IPV) screening occur? a. As soon as the clinician suspects a problem b. Only when a client presents with an unexplained injury c. As a routine part of each healthcare encounter d. Once the clinician confirms a history of abuse

c. As a routine part of each healthcare encounter

While teaching a young male adult to use an inhaler for his newly diagnosed asthma, the client stares into the distance and appears to be concentrating on something other than the lesson the nurse is presenting. What action should the nurse take a. Remind the client that a rescue inhaler might save his life b. Leave the client alone so that he can grieve his illness c. Ask the client what he is thinking about at his time. d. Gently touch the client then continue with teaching.

c. Ask the client what he is thinking about at his time.

On a busy day, one hour after the shift report is completed, the charge nurse learns that a female staff nurse who lives one hour away from the hospital forgot her prescription eye glasses at home. What action should the charge nurse take? a. Encourage the nurse purchase the reading glasses in the hospital gift shop b. Request another nurse to assist the staff nurse with her documentation c. Ask the nurse to return home and get her prescription eyeglasses for work. d. Tell the staff nurse to take a day off and change her weekly work schedule.

c. Ask the nurse to return home and get her prescription eyeglasses for work.

A client who had a percutaneous transluminal coronary angioplasty (PTCA) two weeks ago returns to the clinic for a follow up visit. The client has a postoperative ejection fraction ejection fraction of 30%. Today the client has lungs which are clear, +1 pedal edema, and a 5pound weight gain. Which intervention the nurse implement? a. Arrange transport for admission to the hospital. b. Insert saline lock for IV diuretic therapy. c. Assess compliance with routine prescriptions. d. Instruct the client to monitor daily caloric intake.

c. Assess compliance with routine prescriptions.

A male client has received a prescription for orlistat for weight and nutrition management. In addition to the medication, the client states he plans to take a multivitamin. What teaching should the nurse provide? a. As a nutritional supplement, orlistat already contains all the recommended daily vitamins and minerals. b. Multivitamins are contraindicated. During treatment with weight-control medications such as orlistat c. Be sure to take the multivitamin and the medication at least two hours apart for best absorption and effectiveness. d. Following a well-balanced diet is a much healthier approach to good nutrition than depending on a multivitamin.

c. Be sure to take the multivitamin and the medication at least two hours apart for best absorption and effectiveness.

A nurse who is working in the emergency department triage area is presented with four clients at the same time. The client presented with which symptoms requires the most immediate intervention by the nurse? a. Low-grade fever, headache, and malaise for the past 72 hours b. Unable to bear weight on the left foot, with the swelling and bruising c. Chest discomfort one hour after consuming a large, spicy meal d. One-inch bleeding laceration on the chain of the crying five-year-old

c. Chest discomfort one hour after consuming a large, spicy meal

A 7-year-old boy is brought to the clinic because of facial edema. He reports that he has been voiding small amounts of dark, cloudy, tea-colored urine. The parents state that their son had a sore throat 2 weeks earlier, but it has resolved. After assessing the child's vital signs and weight, what intervention should the nurse implement next? a. Perform an otoscopic examination b. Measure the child's abdominal girth c. Collect a urine specimen for routine urinalysis d. Obtain a blood specimen for serum electrolytes.

c. Collect a urine specimen for routine urinalysis

An adult female client is admitted to the psychiatric unit because of a complex handwashing ritual she performs daily that takes two hours or longer to complete. She worries about staying clean and refuses to sit on any of the chairs in the day area. This client's handwashing is an example of which clinical behavior? a. Addiction b. Phobia c. Compulsion d. Obsession

c. Compulsion

A client who sustained a head injury following an automobile collision is admitted to the hospital. The nurse include the client's risk for developing increased intracranial pressure (ICP) in the plan of care. Which signs indicate to the nurse that ICP has increased? a. Increased Glasgow coma scale score. b. Nuchal rigidity and papilledema. c. Confusion and papilledema d. Periorbital ecchymosis.

c. Confusion and papilledema

While in the medical records department, the nurse observes several old medical records with names visible in waste container. What action should the nurse implement? a. Place the records in a separate trash bag and tie the bag securely closed b. Point out the record to a worker in the medical records department c. Contact the medical records department supervisor. d. Immediately remove and shred the records.

c. Contact the medical records department supervisor.

A preeclamptic client who delivered 24h ago remains in the labor and delivery recovery room. She continues to receive magnesium sulfate at 2 grams per hour. Her total input is limited to 125 ml per hour, and her urinary output for the last hour was 850 ml. What intervention should the nurse implement? a. discontinue the magnesium sulfate immediately b. Decrease the client's iv rate to 50 ml per hour c. Continue with the plan of care for this client d. Change the client's to NPO status

c. Continue with the plan of care for this client

While caring for a client's postoperative dressing, the nurse observes purulent drainage at the wound. Before reporting this finding to the healthcare provider, the nurse should review which of the client's laboratory values? a. Serum albumin b. Creatinine level c. Culture for sensitive organisms. d. Serum blood glucose (BG) level

c. Culture for sensitive organisms.

A client exposed to tuberculosis is scheduled to begin prophylactic treatment with isoniazid. Which information is most important for the nurse to note before administering the initial dose? a. Conversion of the client's PPD test from negative to positive. b. Length of time of the exposure to tuberculosis. c. Current diagnosis of hepatitis B. d. History of intravenous drug abuse.

c. Current diagnosis of hepatitis B.

A client with hyperthyroidism is receiving propranolol (Inderal). Which finding indicates that the medication is having the desired effect a. Decrease in serum T4 levels b. Increase in blood pressure c. Decrease in pulse rate d. Goiter no longer palpable.

c. Decrease in pulse rate

A new mother tells the nurse that she is unsure if she will be able to transition into parenthood. What action should the nurse take? a. Provide reassurance to the client that these feeling are normal after delivery b. Discuss delaying the client's discharge from the hospital for another 24 hrs. c. Determine if she can ask for support from family, friend, or the baby's father. d. Explain the differences between postpartum blues and postpartum depression.

c. Determine if she can ask for support from family, friend, or the baby's father.

A client arrives on the surgical floor after major abdominal surgery. What intervention should the nurse perform first? a. Administer prescribed pain medication b. Assess surgical site c. Determine the client's vital sign. d. Apply warmed blankets

c. Determine the client's vital sign.

A young boy who is in a chronic vegetative state and living at home is readmitted to the hospital with pneumonia and pressure ulcers. The mother insists that she is capable of caring for her son and which action should the nurse implement next? a. Report the incident to the local child protective services. b. Find a home health agency that specializes in brain injuries. c. Determine the mother's basic skill level in providing care. d. Consult the ethics committee to determine how to proceed.

c. Determine the mother's basic skill level in providing care.

A female client with acute respiratory distress syndrome (ARDS) is chemically paralyzed and sedated while she is on as assist-control ventilator using 50% FIO2. Which assessment finding warrants immediate intervention by the nurse? a. Premature atrial contractions (PAC) b. Hemoccult-positive nasogastric fluid c. Diminished left lower lobe sounds. d. Increasing endotracheal secretions.

c. Diminished left lower lobe sounds.

The nurse is preparing to administer 1.6 ml of medication IM to a 4 month old infant. Which action should the nurse include? a. Select a 22 gauge 1 ½ inch (3.8 cm) needle for the intramuscular injection b. Administer into the deltoid muscle while the parent holds the infant securely c. Divide the medication into two injections with volumes under 1ml d. Use a quick dart-like motion to inject into the dorsogluteal site.

c. Divide the medication into two injections with volumes under 1ml

The nurse note a visible prolapse of the umbilical cord after a client experiences spontaneous rupture of the membranes during labor. What intervention should the nurse implement immediately? a. Administer oxygen by face mask at 6L/mint b. Transport the client for a cesarean delivery c. Elevate the presenting part off the cord. d. Place the client to a knee-chest position.

c. Elevate the presenting part off the cord.

The nurse administers an oral antiviral to a client with shingles. Which finding is most important for the nurse report to the health care provider? a. Decreased white blood cell count b. Pruritus and muscle aches c. Elevated liver function tests d. Vomiting and diarrhea

c. Elevated liver function tests

In preparing a diabetes education program, which goal should the nurse identify as the primary emphasis for a class on diabetes self-management? a. Prepare the client to independently treat their disease process b. Reduce healthcare costs related to diabetic complications c. Enable clients to become active participating in controlling the disease process d. Increase client's knowledge of the diabetic disease process and treatment options.

c. Enable clients to become active participating in controlling the disease process

A male client's laboratory results include a platelet count of 105,000/ mm3 Based on this finding the nurse should include which action in the client's plan of care? a. Cluster care to conserve energy b. Initiate contact isolation c. Encourage him to use an electric razor d. Asses him for adventitious lung sounds

c. Encourage him to use an electric razor

A 12 year old client who had an appendectomy two days ago is receiving 0.9% normal saline at 50 ml/hour. The client's urine specific gravity is 1.035. What action should the nurse implement? a. Evaluate postural blood pressure measurements b. Obtain specimen for uranalysis c. Encourage popsicles and fluids of choice d. Assess bowel sounds in all quadrants

c. Encourage popsicles and fluids of choice

A community health nurse is concerned about the spread of communicable diseases among migrant farm workers in a rural community. What action should the nurse take to promote the success of a healthcare program designed to address this problem? a. Conduct face to face prevention education group session is Spanish b. Offer low literacy material that explain respiratory hygiene and handwashing techniques c. Establish trust with community leaders and respect cultural and family values. d. Provide public services announcements advising those who aril o seek prompt medical attention.

c. Establish trust with community leaders and respect cultural and family values.

The nurse manager is conducting an in-services education program on the fire evacuation of the newborn recovery. What intervention should the nurse manager disseminate to the staff? a. Place infants on a blanket for evacuation via stairwell b. Secure three infants on a stretcher for transport c. Evacuate each infant with mother via wheelchair d. Use the bassinet in evacuate two infants at a time. Evacuate each infant with mother via wheelchair

c. Evacuate each infant with mother via wheelchair

Two clients ring their call bells simultaneously requesting pain medication. What action should the nurse implement first? a. Prepared both client's medication and take to them at once b. Determine when each client last received pain medication. c. Evaluate both client's pain using a standardized pain scale d. Provide non-pharmacologic pain management interventions.

c. Evaluate both client's pain using a standardized pain scale

The nurse ends the assessment of a client by performing a mental status exam. Which statement correctly describes the purpose of the mental status exam? a. Determine the client's level of emotional functioning' b. Assess functional ability of the primary support system. c. Evaluate the client's mood, cognition and orientation. d. Review the client's pattern of adaptive coping skill

c. Evaluate the client's mood, cognition and orientation.

Diagnostic studies indicate that the elderly client has decreased bone density. In providing client teaching, which area of instruction is most important for the nurse to include? a. Application of joint splints b. Effective body mechanisms c. Fall prevention measures. Low fat, high protein diet

c. Fall prevention measures.

Following a motor vehicle collision (MCV), a male adult in severe pain is brought to the emergency department via ambulance. His injured left leg is edematous, ecchymotic around the impact of injury on the thigh, and shorter than his right leg. Based on these findings, the client is at greatest risk for which complication? a. Arterial ischemia b. Tissue necrosis c. Fat embolism d. Nerve damage

c. Fat embolism

The nurse is evaluating a client's symptoms, and formulates the nursing diagnosis, "high risk for injury due to possible urinary tract infection." Which symptoms indicate the need for this diagnosis? a. Hematuria and proteinuria b. Azotemia and anorexia c. Fever and dysuria. d. Straining on urination and nocturia

c. Fever and dysuria.

A client with C-6 spinal cord injury rehabilitation. In the middle of the night the client reports a severe, pounding headache, and has observable piloerection or "goosebumps". The nurse should asses for which trigger? a. Loud hallway noise. b. Fever c. Full bladder d. Frequent cough.

c. Full bladder

When development a teaching plan for a client newly diagnosed type 1 diabetes, the nurse should explain that an increase thirst is an early sing of diabetes ketoacidosis (DKA), which action should the nurse instruct the client to implement if this sign of DKA occur? a. Resume normal physical activity b. Drink electrolyte fluid replacement c. Give a dose of regular insulin per sliding scale d. Measure urinary output over 24 hours.

c. Give a dose of regular insulin per sliding scale

If the nurse is initiating IV fluid replacement for a child who has dry, sticky mucous membranes, flushed skin, and fever of 103.6 F. Laboratory finding indicate that the child has a sodium concentration of 156 mEq/L. What physiologic mechanism contributes to this finding? a. The intravenous fluid replacement contains a hypertonic solution of sodium chloride b. Urinary and Gastrointestinal fluid loss reduce blood viscosity and stimulate thirst c. Insensible loss of body fluids contributes to the hemoconcentration of serum solutes d. Hypothalamic resetting of core body temperature causes vasodilation to reduce body heat

c. Insensible loss of body fluids contributes to the hemoconcentration of serum solutes

The nurse caring for a client with acute renal fluid (ARF) has noted that the client has voided 800 ml of urine in 4 hours. Based on this assessment, what should the nurse anticipate that client will need? a. Treatment for acute uremic symptoms within 24 hours b. Change to a regular diet c. Large amounts of fluid and electrolyte replacement. d. Unrestricted sodium intake

c. Large amounts of fluid and electrolyte replacement.

After placing a stethoscope as seen in the picture, the nurse auscultates S1 and S2 heart sounds. To determine if an S3 heart sound is present, what action should the nurse take first a. Side the stethoscope across the sternum. b. Move the stethoscope to the mitral site c. Listen with the bell at the same location d. Observe the cardiac telemetry monitor

c. Listen with the bell at the same location

A client at 30 week gestation is admitted due to preterm labor. A prescription of terbutaline sulfate 8.35 mg is gives subcutaneously. Based on which finding should the nurse withhold the next dose of this drug? a. Maternal blood pressure of 90/60 b. Fetal heart rate of 170 beats per minute for 15 mints c. Maternal pulse rate of 162 beats per min d. Serum potassium of 2.3 mg/dl

c. Maternal pulse rate of 162 beats per min

While changing a client's chest tube dressing, the nurse notes a crackling sensation when gentle pressure is applied to the skin at the insertion site. What is the best action for the nurse to take? a. Apply a pressure dressing around the chest tube insertion site. b. Assess the client for allergies to topical cleaning agents. c. Measure the area of swelling and crackling. d. Administer an oral antihistamine per PRN protocol.

c. Measure the area of swelling and crackling.

A 66-year-old woman is retiring and will no longer have a health insurance through her place of employment. Which agency should the client be referred to by the employee health nurse for health insurance needs? a. Woman, Infant, and Children program b. Medicaid c. Medicare d. Consolidated Omnibus Budget Reconciliation Act provision.

c. Medicare

Which intervention should the nurse include in the plan of care for a child with tetanus? a. Open window shades to provide natural light b. Reposition side to side every hour. c. Minimize the number of stimuli in the room. d. Encourage coughing and deep breathing

c. Minimize the number of stimuli in the room.

A mother brings her 3-year-old son to the emergency room and tells the nurse the he has had an upper respiratory infection for the past two days. Assessment of the child reveals a rectal temperature of 102 F. he is drooling and becoming increasingly more restless. What action should the nurse take first? a. Put a cold cloth on his head and administer acetaminophen. b. Listen to lung sounds and place him in a mist tent. c. Notify the healthcare provider and obtain a tracheostomy tray d. Assist the child to lie down and examine his throat.

c. Notify the healthcare provider and obtain a tracheostomy tray

A client with leukemia undergoes a bone marrow biopsy. The client's laboratory values indicate the client has thrombocytopenia. Based on this data, which nursing assessment is most important following the procedure? a. Assess body temperature b. Monitor skin elasticity c. Observe aspiration site. d. Measure urinary output

c. Observe aspiration site.

In evaluating the effectiveness of a postoperative client's intermittent pneumatic compression devices, which assessment is most important for the nurse to complete? a. Evaluate the client's ability to use an incentive spirometer b. Monitor the amount of drainage from the client's incision c. Observe both lower extremities for redness and swelling d. Palpate all peripheral pulse points for volume and strength

c. Observe both lower extremities for redness and swelling

A client with a history of heart failure presents to the clinic with a nausea, vomiting, yellow vision and palpitations. Which finding is most important for the nurse to assess to the client? a. Determine the client's level of orientation and cognition b. Assess distal pulses and signs of peripheral edema c. Obtain a list of medications taken for cardiac history. d. Ask the client about exposure to environmental heat.

c. Obtain a list of medications taken for cardiac history.

After applying an alcohol-based hand rub to the palms of the hand and rubbing the hand together, what action should the nurse do next? a. Vigorous rub both hands together under running water b. Path both hands dry keeping the fingers lower that the arm c. Place one hand on top of the other and interlace the fingers d. Hold both hand with the fingers pointing upward until dry.

c. Place one hand on top of the other and interlace the fingers

During a Woman's Health fair, which assignment is the best for the Practical Nurse (PN) who is working with a register nurse (RN) a. Encourage the woman at risk for cancer to obtain colonoscopy. b. Present a class of breast-self examination c. Prepare a woman for a bone density screening d. Explain the follow-up need it for a client with prehypertension.

c. Prepare a woman for a bone density screening

A client's telemetry monitor indicates ventricular fibrillation (VF). What should the nurse do first? a. Administer epinephrine IV b. Give an IV bolus of amiodarone c. Provide immediate defibrillation d. Prepare for synchronized cardioversion

c. Provide immediate defibrillation

A client with emphysema is being discharged from the hospital. The nurse enters the client's room to complete discharge teaching. The client reports feeling a little short of breath and is anxious about going home. What is the best course of action? a. Postpone discharge instructions at this time and offer to contact the client by phone in a few days b. Invite the client to return to the unit for discharge teaching in a few days, when there is less anxiety c. Provide only necessary information in short, simple explanations with written instructions to take home d. Give detailed instructions speaking slowly and clearly while looking directly at the client when speaking

c. Provide only necessary information in short, simple explanations with written instructions to take home

When assessing and adult male who presents as the community health clinic with a history of hypertension, the nurse note that he has 2+ pitting edema in both ankles. He also has a history of gastroesophageal reflex disease (GERD) and depression. Which intervention is the most important for the nurse to implement? a. Arrange to transport the client to the hospital b. Instruct the client to keep a food journal, including portions size. c. Review the client's use of over the counter (OTC) medications. d. Reinforce the importance of keeping the feet elevated.

c. Review the client's use of over the counter (OTC) medications.

Following discharge teaching, a male client with duodenal ulcer tells the nurse the he will drink plenty of dairy products, such as milk, to help coat and protect his ulcer. What is the best follow-up action by the nurse? a. Remind the client that it is also important to switch to decaffeinated coffee and tea. b. Suggest that the client also plan to eat frequent small meals to reduce discomfort c. Review with the client the need to avoid foods that are rich in milk and cream. d. Reinforce this teaching by asking the client to list a dairy food that he might select.

c. Review with the client the need to avoid foods that are rich in milk and cream.

A young adult client is admitted to the emergency room following a motor vehicle collision. The client's head hit the dashboard. Admission assessment include: Blood pressure 85/45 mm Hg, temperature 98.6 F, pulse 124 beat/minute and respirations 22 breath/minute. Based on these data, the nurse formulates the first portion of nursing diagnosis as " Risk of injury" What term best expresses the "related to" portion of nursing diagnosis? a. Infection b. Increase intracranial pressure c. Shock d. Head Injury.

c. Shock

A client who was admitted yesterday with severe dehydration is complaining of pain a 24 gauge IV with normal saline is infusing at a rate of 150 ml/hour. Which intervention should the nurse implement first? a. Establish the second IV site b. Asses the IV for blood return c. Stop the normal saline infusion. d. Discontinue the 24-gauge IV

c. Stop the normal saline infusion.

A male client with hypertension, who received new antihypertensive prescriptions at his last visit returns to the clinic two weeks later to evaluate his blood pressure (BP). His BP is 158/106 and he admits that he has not been taking the prescribed medication because the drugs make him "feel bad". In explaining the need for hypertension control, the nurse should stress that an elevated BP places the client at risk for which pathophysiological condition? a. Blindness secondary to cataracts b. Acute kidney injury due to glomerular damage c. Stroke secondary to hemorrhage d. Heart block due to myocardial damage

c. Stroke secondary to hemorrhage

The RN is assigned to care for four surgical clients. After receiving report, which client should the nurse see first? The client who is: a. Two days postoperative bladder surgery with continuous bladder irrigation infusing. b. One day postoperative laparoscopic cholecystectomy requesting pain medication. c. Three days postoperative colon resection receiving transfusion of packed RBCs. d. Preoperative, in buck's traction, and scheduled for hip arthroplasty within the next 12 hours.

c. Three days postoperative colon resection receiving transfusion of packed RBCs.

A client admitted with an acute coronary syndrome (ACS) receives eptifibatide, a glycoprotein (GP) IIB IIIA inhibitor, which important finding places the client at greatest risk? a. Blood pressure of 100/60 b. Incontinent with blood in urine c. Unresponsive to painful stimuli d. Presence of hematemesis.

c. Unresponsive to painful stimuli

A client who is experiencing musculoskeletal pain receives a prescription for ketorolac 15mg IM q6 hours. The medication is depended in a 39mg/ml pre-filled syringe. Which action should the nurse implement when giving the medication? a. Administer the entire pre-filled syringe deep in the dorsogluteal site. b. Use a separate syringe to remove 15mg from the pre-filled syringe and give in the back of the arm. c. Waste 0.5 ml from the pre-filled syringe and inject the medication in the ventrogluteal site. d. Call the healthcare provider to request a prescription change to match the dispensed 30mg dose.

c. Waste 0.5 ml from the pre-filled syringe and inject the medication in the ventrogluteal site.

An African-American man come into the hypertension screening booth at a community fair. The nurse finds that is blood pressure is 170/94 mmHg. The client tells the nurse that he has never been treated for high blood pressure. What response should the nurse make? a. Your blood pressure indicate that you have hypertension. You need to see a physician at once. b. Your blood pressure is quite high. Go to the closest emergency room for immediate treatment. c. Your blood pressure is a little high. You need to have it rechecked within one week. d. Your blood pressure is little high, but it is within the normal range for your age group.

c. Your blood pressure is a little high. You need to have it rechecked within one week.

The nurse is assessing a client's nailbeds. Witch appearance indicates further follow-up is needed for problems associated with chronic hypoxia?

clubbing

The nurse prepares to insert an oral airway by first measuring for the correct sized airway. Which picture shows the correct approach to airway size measurement?

corner of the mouth to the tip of the ear

The nurse is teaching a male client with multiple sclerosis how to empty his bladder using the Crede Method. When performing a return demonstration, the client applies pressure to the umbilical areas of his abdomen. What instruction should the nurse provide? a- Stroke the inner thigh below the perineum to initiate urinary flow b- Contract, hold, and then relax the pubococcygeal muscle c- Pour warm water over the external sphincter at the distal glans d- Apply downward manual pressure at the suprapubic regions.

d- Apply downward manual pressure at the suprapubic regions.

An older female client tells the nurse that her muscles have gradually been getting weak...what is the best initial response by the nurse? a- Explain that this is an expected occurrence with aging. b- Observe the lower extremity for signs of muscle atrophy c- Review the medical record for recent diagnosis test results. d- Ask the client to describe the changes that have occurred

d- Ask the client to describe the changes that have occurred

Artificial rupture of the membrane of a laboring reveals meconium-stained fluid, what is... the priority? 545. a- Clean the perineal are to prevent infection b- Assess the mother's blood pressure to check for signs of preeclampsia c- Assess the mother temperature to check for development of sepsis. d- Have a meconium aspirator available at delivery.

d- Have a meconium aspirator available at delivery.

An older woman who has difficulty hearing is being discharged from day surgery following a cataract extraction & lens implantation. Which intervention is most important for the nurse to implement to ensure the client's compliance with self-care? a- Speak clearly and face the clients for lip reading b- Provide written instructions for eyes drop administration c- Ensure that someone will stay with the client for 24 hours. d- Have the client vocalize the instructions provided.

d- Have the client vocalize the instructions provided.

The nurse plans to administer a schedule dose of metoprolol (Toprol SR) at 0900 to a client with hypertension. At 0800, the nurse notes that client's telemetry pattern shows a second degree heart block with a ventricular rate of 50. What action should the nurse take? a- Administer the Tropol immediately and monitor the client until the heart rate increases. b- Provide the dose of Tropol as scheduled and assign a UAP to monitor the client's BP q30 minutes. c- Give the Tropol as scheduled if the client's systolic blood pressure reading is greater than 180. d- Hold the scheduled dose of Tropol and notify the healthcare provider of the telemetry pattern.

d- Hold the scheduled dose of Tropol and notify the healthcare provider of the telemetry pattern.

An adult male who was admitted two days ago following a cerebrovascular accident (CVA) is confused and experiencing left-side weakness. He has tried to get out of bed several times, but is unable to ambulate without assistance. Which intervention is most important for the nurse to implement? a- Ask a family member to sit with the client b- Apply bilateral soft wrist restraints c- Assign staff to check client q15 minutes d- Install a bed exit safety monitoring device

d- Install a bed exit safety monitoring device

A male infant born at 28-weeks gestation at an outlying hospital is being prepared for transport to a respiration are 92 breaths/minute and his heart rate is 156 beats/minute. Which drug is the transport administration to this infant? a- Give ampicillin 25 mg/kg slow IV push b- Deliver 1:10,000 epinephrine 0.1 ml/kg per endotracheal tube c- Administer digoxin 20 mcg/kg IV d- Instill beractant 100 mg/kg in endotracheal tube.

d- Instill beractant 100 mg/kg in endotracheal tube.

After receiving lactulose, a client with hepatic encephalopathy has several loose stools. What action should the nurse implement? a- Send stool specimen to the lab b- Measure abdominal girth c- Encourage increased fiber in diet. d- Monitor mental status.

d- Monitor mental status.

The nurse should observe most closely for drug toxicity when a client receives a medication that has which characteristic? a- Low bioavailability b- Rapid onset of action c- Short half life d- Narrow therapeutic index.

d- Narrow therapeutic index.

During a cardiopulmonary resuscitation of an intubated client, the nurse detects a palpable pulse throughout the two minutes cycle chest compression and absent breath sounds over the left lung. What action should the nurse implement? a- Instruct the compressor to stop chest compression. b- Advise ventilator to increase bag-mask ventilation rate. c- Plan to suction endotracheal tube at two-minute check. d- Prepare for the endotracheal tube to be repositioned

d- Prepare for the endotracheal tube to be repositioned

Following routine diagnostic test, a client who is symptom-free is diagnosed with Paget's disease. Client teaching should be directed toward what important goal for this client? a- Maintain adequate cardiac output b- Promote adequate tissue perfusion c- Promote rest and sleep d- Reduce the risk for injury

d- Reduce the risk for injury

A client who is schedule for an elective inguinal hernia repair today in day surgery is seem eating in the waiting area. What action should be taken by the nurse who is preparing to administer the preoperative medications? a- Review the surgical consent with the client b- Explain that vomiting can occur during surgery c- Remove the food from the client d- Withhold the preoperative medication

d- Withhold the preoperative medication

A female client reports that her hair is becoming coarse and breaking off, that the outer part of her eyebrows have disappeared, and that her eyes are all puffy. Which follow-up question is best for the nurse to ask? a. "Is there a history of female baldness in your family?" b. "Are you under any unusual stress at home or work?" c. "Do you work with hazardous chemicals?" d. "Have you noticed any changes in your fingernails?"

d. "Have you noticed any changes in your fingernails?"

The charge nurse is planning for the shift and has a registered nurse (RN) and a practical nurse (PN) on the team. Which client should the charge nurse assign to the RN? a. A 64-year-old client who had a total hip replacement the previous day. b. A 75-year-old client with renal calculi who requires urine straining. c. An adolescent with multiple contusions due to a fall that occurred 2 days ago. d. A 30-year-old depressed client who admits to suicide ideation.

d. A 30-year-old depressed client who admits to suicide ideation.

A nurse working on an endocrine unit should see which client first? a. An adolescent male with diabetes who is arguing about his insulin dose. b. An older client with Addison's disease whose current blood sugar level is 62mg/dl (3.44 mmol/l). c. An adult with a blood sugar of 384mg/dl (21.31mmol/l) and urine output of 350 ml in the last hour. d. A client taking corticosteroids who has become disoriented in the last two hours.

d. A client taking corticosteroids who has become disoriented in the last two hours.

The nurse working in the psychiatric clinic has phone messages from several clients. Which call should the nurse return first? a. A young man with schizophrenia who wants to stop taking his medication b. The mother of a child who was involved in a physical fight at school today. c. A client diagnosed with depression who is experiencing sexual dysfunction. d. A family member of a client with dementia who has been missing for five hours

d. A family member of a client with dementia who has been missing for five hours

The nurse who is working on a surgical unit receives change of shift report on a group of clients for the upcoming shift. A client with which condition requires the most immediate attention by the nurse? a. Gunshot wound three hours ago with dark drainage of 2 cm noted on the dressing. b. Mastectomy 2 days ago with 50 ml bloody drainage noted in the Jackson-pratt drain. c. Collapsed lung after a fall 8h ago with 100 ml blood in the chest tube collection container d. Abdominal-perineal resection 2 days ago with no drainage on dressing who has fever and chills.

d. Abdominal-perineal resection 2 days ago with no drainage on dressing who has fever and chills.

A postoperative female client has a prescription for morphine sulfate 10 mg IV q3 hours for pain. One dose of morphine was administered when the client was admitted to the post anesthesia care unit (PACU) and 3 hours later, the client is again complaining of pain. Her current respiratory rate is 8 breaths/minute. What action should the nurse take? a. Provide oxygen 100% via facemask b. Check peripheral tendon reflexes c. Give another IV dose of morphine d. Administer Naloxone IV

d. Administer Naloxone IV

While making rounds, the charge nurse notices that a young adult client with asthma who was admitted yesterday is sitting on the side of the bed and leaning over the bed-side-table. The client is currently receiving at 2 litters/minute via nasal cannula. The client is wheezing and is using pursed-lip breathing. Which intervention should the nurse implement? a. Assist the client to lie back in bed b. Call for an Ambu resuscitating bag c. Increase oxygen to 6 litters/minute d. Administer a nebulizer Treatment

d. Administer a nebulizer Treatment

A client with a history of chronic pain requests a nonopioid analgesic. The client is alert but has difficulty describing the exact nature and location of the pain to the nurse. What action should the nurse implement next? a. Assess the client's vital signs b. Observe the client's pupils for dilation c. Document the client's drug tolerance d. Administer the analgesic as requested

d. Administer the analgesic as requested

Following surgery, a male client with antisocial personality disorder frequently requests that a specific nurse be assigned to his care and is belligerent when another nurse is assigned. What action should the charge nurse implement? a. Ask the client to explain why he constantly request the nurse b. Encourage the client to verbalize his feelings about the nurse c. Reassure the client that his request will be met whenever possible. d. Advise the client that assignments are not based on client requests

d. Advise the client that assignments are not based on client requests

The nurse makes a supervisory home visit to observe an unlicensed assistive personnel (UAP) who is providing personal care for a client with Alzheimer's disease. The nurse observes that whenever the client gets upset, the UAP changes the subject. What action should the nurse take in response to this observation? a. Tell the UAP to offer more choices during the personal care to prevent anxiety b. Meet with the UAP later to role model more assertive communication techniques c. Assume care of the client to ensure that effective communication is maintained. d. Affirm that the UAP is using and effective strategy to reduce the client's anxiety.

d. Affirm that the UAP is using and effective strategy to reduce the client's anxiety.

The charge nurse of a critical care unit is informed at the beginning of the shift that less than the optimal number of registered nurses will be working that shift. In planning assignments, which client should receive the most care hours by a registered nurse (RN) a. A 34-year -old admitted today after an emergency appendendectomy who has a peripheral intravenous catheter and a Foley catheter. b. A 48-year-old marathon runner with a central venous catheter who is experiencing nausea and vomiting due to electrolyte disturbance following a race. c. A 63-year-old chain smoker admitted with chronic bronchitis who is receiving oxygen via nasal cannula and has a saline-locked peripheral intravenous catheter. d. An 82-year-old client with Alzheimer's disease newly-fractures femur who has a Foley catheter and soft wrist restrains applied

d. An 82-year-old client with Alzheimer's disease newly-fractures femur who has a Foley catheter and soft wrist restrains applied

An older male client is admitted with the medical diagnosis of possible cerebral vascular accident (CVA). He has facial paralysis and cannot move his left side. When entering the room, the nurse finds the client's wife tearful and trying unsuccessfully to give him a drink of water. What action should the nurse take? a. Give the wife a straw to help facilitate the client's drinking. b. Assist the wife and carefully give the client small sips of water c. Obtain a thickening powder before providing any more fluids. d. Ask the wife to stop and assess the client's swallowing reflex.

d. Ask the wife to stop and assess the client's swallowing reflex.

Which nursing intervention has the highest priority for a multigravida who delivered? a. Maintain cold packs to the perineum for 24 hrs. b. Assess the client pain level frequently c. Observe for appropriate interaction with the infants. d. Assess fundal tone and lochia flow

d. Assess fundal tone and lochia flow

The nurse who works in labor and delivery is reassigned to the cardiac care unit for the day because of a low census in labor and delivery. Which assignments is best for the nurse to give this nurse? a. Transfer a client to another unit b. Monitor the central telemetry c. Perform the admission d. Assist cardiac nurses with their assignments

d. Assist cardiac nurses with their assignments

A male client, who is 24 hours postoperative for an exploratory laparotomy, complains that he is "starving" because he has had no "real food" since before the surgery. Prior to advancing his diet, which intervention should the nurse implement? a. Discontinue intravenous therapy b. Obtain a prescription for a diet change c. Assess for abdominal distention and tenderness. d. Auscultate bowel sounds in all four quadrants

d. Auscultate bowel sounds in all four quadrants

The nurse identifies an electrolyte imbalance, an elevated pulse rate, and elevated BP for a client with chronic kidney disease. Which is the most important action for the nurse to take? a. Monitor daily sodium intake. b. Record usual eating patterns. c. Measure ankle circumference. d. Auscultate for irregular heart rate.

d. Auscultate for irregular heart rate.

A client with angina pectoris is being discharge from the hospital. What instruction should the nurse plan to include in this discharge teaching? a. Engage in physical exercise immediately after eating to help decrease cholesterol levels. b. Walk briskly in cold weather to increase cardiac output c. Keep nitroglycerin in a light-colored plastic bottle and readily available. d. Avoid all isometric exercises but walk regularly. Avoid all isometric exercises, but walk regularly

d. Avoid all isometric exercises but walk regularly.

The nurse is preparing a discharge teaching plan for a client who had a liver transplant. Which instruction is most important to include in this plan? a. Limit intake fatty foods for one month after surgery. b. Notify the healthcare provider if edema occurs. c. Increase activity and exercise gradually, as tolerated. d. Avoid crowds for first two months after surgery.

d. Avoid crowds for first two months after surgery.

A client with atrial fibrillation receives a new prescription for dabigatran. What instruction should the nurse include in this client's teaching plan? a. Keep an antidote available in the event of hemorrhage b. Continue obtaining scheduled laboratory bleeding test c. Eliminate spinach and other green vegetable in the diet. d. Avoid use of nonsteroidal ant-inflammatory drugs (NSAID).

d. Avoid use of nonsteroidal ant-inflammatory drugs (NSAID).

Which breakfast selection indicates that the client understands the nurse's instructions about the dietary management of osteoporosis? a. Egg whites, toast and coffee. b. Bran muffin, mixed fruits, and orange juice. c. Granola and grapefruit juice d. Bagel with jelly and skim milk.

d. Bagel with jelly and skim milk.

A client receiving chemotherapy has severe neutropenia. Which snack is best for the nurse to recommend to the client? a. Plain yogurt with sweetened with raw honey b. Peanuts in the shell, roasted or un-roasted. c. Aged farmer's cheese with celery sticks d. Baked apples topped with dried raisins

d. Baked apples topped with dried raisins

A 13 years-old client with non-union of a comminuted fracture of the tibia is admitted with osteomyelitis. The healthcare provider collects home aspirate specimens for culture and sensitivity and applies a cast to the adolescent's lower leg. What action should the nurse implement next? a. Administer antiemetic agents b. Bivalve the cast for distal compromise c. Provide high- calorie, high-protein diet d. Begin parenteral antibiotic therapy

d. Begin parenteral antibiotic therapy

The nurse is planning care for a client who admits having suicidal thoughts. Which client behavior indicates the highest risk for the client acting on these suicidal thoughts? a. Express feelings of sadness and loneliness b. Neglects personal hygiene and has no appetite c. Lacks interest in the activity of the family and friends d. Begin to show signs of improvement in affect

d. Begin to show signs of improvement in affect

After a routine physical examination, the healthcare admits a woman with a history of Systemic Lupus Erythematous (SLE) to the hospital because she has 3+ pitting ankle edema and blood in her urine. Which assessment finding warrants immediate intervention by the nurse? a. Dark, rust-colored urine b. Urine output 300 ml/hr c. Joint and muscle aches d. Blood pressure 170/98

d. Blood pressure 170/98

A client is receiving mesalamine 800 mg PO TID. Which assessment is most important for the nurse to perform to assess the effectiveness of the medication? a. Pupillary response b. Oxygen saturation c. Peripheral pulses d. Bowel patterns

d. Bowel patterns

An older male client with type 2 diabetes mellitus reports that has experiences legs pain when walking short distances, and that the pain is relieved by rest. Which client behavior indicates an understanding of healthcare teaching to promote more effective arterial circulation? a. Consistently applies TED hose before getting dressed in the morning. b. Frequently elevated legs thorough the day. c. Inspect the leg frequently for any irritation or skin breakdown d. Completely stop cigarette/ cigar smoking.

d. Completely stop cigarette/ cigar smoking.

The nurse is caring for a client receiving continuous IV fluids through a single lumen central venous catheter (CVC). Based on the CVC care bundle, which action should be completed daily to reduce the risk for infection? a. Remind staff to follow protective environment precautions b. Gently flush the catheter lumen with sterile saline solution c. Cleanse the site and change the transparent dressing. d. Confirm the necessity for continued use of the CVC.

d. Confirm the necessity for continued use of the CVC.

Based on the information provided in this client's medical record during labor, which should the nurse implement? (Click on each chart tab for additional information. Please be sure to scroll to the bottom right corner of each tab to view all information contained in the client's medical record.) a. Apply oxygen 10 l/mask b. Stop the oxytocin infusion c. Turn the client to the right lateral position. d. Continue to monitor the progress of labor.

d. Continue to monitor the progress of labor.

A client with a chronic health problem has difficulty ambulating short distance due to generalized weakness but can bear weight on both legs. To assist with ambulation and provide the greatest stability, what assistive device is best for this client? a. A quad cane b. Crutches with 2-point gait. c. Crutches with 3-point gait. d. Crutches with 4-point gait.

d. Crutches with 4-point gait.

The nurse is preparing to administer an oral antibiotic to a client with unilateral weakness, ptosis, mouth drooping and, aspiration pneumonia. What is the priority nursing assessment that should be done before administering this medication a. Ask the client about soft foods preferences b. Auscultate the client's breath sounds c. Obtain and record the client's vital signs d. Determine which side of the body is weak.

d. Determine which side of the body is weak.

Before preparing a client for the first surgical case of the day, a part-time scrub nurse asks the circulating nurse if a 3 minute surgical hand scrub is adequate preparation for this client. Which response should the circulating nurse provide? a. Ask a more experience nurse to perform that scrub since it is the first time of the day b. Validate the nurse is implementing the OR policy for surgical hand scrub c. Inform the nurse that hand scrubs should be 3 minutes between cases. d. Direct the nurse to continue the surgical hand scrub for a 5-minute duration.

d. Direct the nurse to continue the surgical hand scrub for a 5-minute duration.

The nurse assigned unlicensed assistive personnel (UAP) to apply antiembolism stockings to a client. The nurse and UAP enters the room, the nurse observes the stockings that were applying by the UAP. The UAP states that the client requested application of the stockings as seen on the picture, for increased comfort. What action should the nurse take? a. Ask the client if the stocking feel comfortable. b. Supervise the UAP in the removal of the stockings. c. Place a cover over the client's toes to keep them warm. d. Discussed effective use of the stockings with the client and UAP

d. Discussed effective use of the stockings with the client and UAP

In assessing an adult client with a partial rebreather mask, the nurse notes that the oxygen reservoir bag does not deflate completely during inspiration and the client's respiratory rate is 14 breaths / minute. What action should the nurse implement a. Encourage the client to take deep breaths b. Remove the mask to deflate the bag c. Increase the liter flow of oxygen d. Document the assessment data

d. Document the assessment data

A client with chronic alcoholism is admitted with a decreased serum magnesium level. Which snack option should the nurse recommend to this client? a. Cheddar cheese and crackers. b. Carrot and celery sticks. c. Beef bologna sausage slices. d. Dry roasted almonds.

d. Dry roasted almonds.

The nurse is planning a class for a group of clients with diabetes mellitus about blood glucose monitoring. In teaching the class as a whole, the nurse should emphasize the need to check glucose levels in which situation? a. Prior to exercising b. Immediately after meals c. Before going to bed d. During acute illness.

d. During acute illness.

Which instruction should the nurse provide a pregnant client who is complaining of heartburn? a. Limit fluids between meals to avoid over distension of the stomach b. Take an antacid at bedtime and whenever symptoms worsen c. Maintain a sitting position for two hours after eating. d. Eat small meal throughout the day to avoid a full stomach.

d. Eat small meal throughout the day to avoid a full stomach.

An elderly female is admitted because of a change in her level of sensorium. During the evening shift, the client attempts to get out bed and falls, breaking her left hip. Buck's skin traction is applied to the left leg while waiting for surgery. Which intervention is most important for the nurse to include in this client's plan care? a. Evaluate her response to narcotic analgesia b. Asses the skin under the traction moleskin c. Place a pillow under the involved lower left leg d. Ensure proper alignment of the leg in traction.

d. Ensure proper alignment of the leg in traction.

In assessing a client twelve hours following transurethral resection of the prostate (TURP), the nurse observes that the urinary drainage tubing contains a large amount of clear pale pink urine and the continuous bladder irrigation is infusing slowly. What action should the nurse implement? a. Increase the rate of the continuous bladder irrigation b. Manually irrigate the catheter with sterile normal saline c. Clam the catheter above the drainage. d. Ensure that no dependent loops are present in the tubing.

d. Ensure that no dependent loops are present in the tubing.

A nurse is planning to teach infant care and preventive measures for sudden infant death syndrome (SIDS) to a group of new parents. What information is most important for the nurse to include? a. Swaddle the infant in a blanket for sleeping b. Place the infant in a prone position whenever possible c. Prop that the infant's crib matter is firm d. Ensure that the infant's crib mattress is firm.

d. Ensure that the infant's crib mattress is firm.

A client admitted to the psychiatric unit diagnosed with major depression wants to sleep during the day, refuses to take a bath, and refuses to eat. Which nursing intervention should the nurse implement first? a. Assess the client's ability to communicate with the other staff members b. Arrange a meeting with the family to discuss the client's situation c. Administer the client's antidepressant medication as prescribed. d. Establish a structured routine for the client to follow.

d. Establish a structured routine for the client to follow.

A client who is admitted to the intensive care unit with syndrome of inappropriate antidiuretic hormone (SIADH) has developed osmotic demyelination. Which intervention should the nurse implement first? a. Patch one eye. b. Reorient often. c. Range of motion. d. Evaluate swallow

d. Evaluate swallow

Progressive kyphoscoliosis leading to respiratory distress is evident in a client with muscul...Which finding warrants immediate intervention by the nurse? a. Extremity muscle weakness b. Bilateral eyelid drooping c. Inability to swallow pills d. Evidence of hypoventilation

d. Evidence of hypoventilation

When performing postural drainage on a client with Chronic Obstructive Pulmonary Disease (COPD), which approach should the nurse use? a. Perform the drainage immediately after meals b. Instruct the client to breath shallow and fast c. Obtain arterial blood gases (ABG's) prior to procedure d. Explain that the client may be placed in five positions

d. Explain that the client may be placed in five positions

An adult male is admitted to the emergency department after falling from a ladder. While waiting to have a computed tomography (CT) scan, he requests something for a severe headache. When the nurse offers him a prescribed does of acetaminophen, he asks for something stronger. Which intervention should the nurse implement? a. Request that the CT scan be done immediately b. Review the client's history for use of illicit drugs c. Assess client's pupils for their reaction to light. d. Explain the reason for using only non-narcotics.

d. Explain the reason for using only non-narcotics.

When conducting diet teaching for a client who was diagnosed with nutritional anemia in pregnancy, which foods should the nurse encourage the client to eat? (Select all that apply) a. Seeds, spices, lettuce b. Consomme, celery, carrot c. Oranges, orange juice, bananas d. Fortified whole wheat cereals, whole-grain pasta, brown rice e. Spinach, kale, dried raisins and apricots

d. Fortified whole wheat cereals, whole-grain pasta, brown rice e. Spinach, kale, dried raisins and apricots

The nurse discovers that an elderly client with no history of cardiac or renal disease has an elevated serum magnesium level. To further investigate the cause of this electrolyte imbalance, what information is most important for the nurse to obtain from the client's medical history? a. Genetically inherited disorders of family members b. Length and frequency of the client's tobacco use. c. Ingestion of selfish or fish oil capsules daily. d. Frequency of laxative use for chronic constipation

d. Frequency of laxative use for chronic constipation

Oral antibiotics are prescribed for an 18-month-old toddler with severe otitis media. An antipyrine and benzocaine-otic also prescribed for pain and inflammation. What instruction should the nurse emphasize concerning the installation of the antipyrine/ benzocaine otic solution? a. Place the dropper on the upper outer ear canal and instill the medication slowly. b. Warm the medication in the microwave for 10 seconds before instilling. c. Keep the medication refrigerated between administrations. d. Have the child lie with the ear up for one to two minutes after installation.

d. Have the child lie with the ear up for one to two minutes after installation.

The nurse is teaching a mother of a newborn with a cleft lip how to bottle feed her baby using medela haberman feeder, which has a valve to control the release of milk and a slit nipple opening. The nurse discusses placing the nipple's elongated tip in the back of the oral cavity. What instructions should the nurse provide the mother about feedings? a. Squeeze the nipple base to introduce milk into the mouth b. Position the baby in the left lateral position after feeding c. Alternate milk with water during feeding d. Hold the newborn in an upright position

d. Hold the newborn in an upright position

Following a lumbar puncture, a client voices several complaints. What complaint indicated to the nurse that the client is experiencing a complication? a. "I am having pain in my lower back when I move my legs" b. "My throat hurts when I swallow" c. "I feel sick to my stomach and am going to throw up" d. I have a headache that gets worse when I sit up"

d. I have a headache that gets worse when I sit up"

A newly graduated female staff nurse approaches the nurse manager and request reassignment to another client because a male client is asking her for a date and making suggestive comments. Which response is best for the nurse manager to provide? a. I have to call the supervisor o get someone else to transfer to this unit to care for him. b. I know you are good nurse and can handle this client in a professional manner. c. I'll talked to the client about his sexual harassment and I'll insist that he stop it immediately. d. I'll change your assignment, but let's talk about you a nurse should respond to this kind of client.

d. I'll change your assignment, but let's talk about you a nurse should respond to this kind of client.

An older male client with history of diabetes mellitus, chronic gout, and osteoarthritis comes to the clinic with a bag of medication bottles. Which intervention should the nurse implement first? a. Obtain a medical history b. Record pain evaluation c. Assess blood glucose d. Identify pills in the bag.

d. Identify pills in the bag.

In caring for a client with a PCA infusion of morphine sulfate through the right cephalic vein, The nurse assesses that the client in lethargic with a blood pressure of 90/60, pulse rate of 118 beats per minute, and respiratory rate of 8 breaths per minutes. What assessment should the nurse perform next? a. Note the appearance and patency of the client's peripheral IV site. b. Palpate the volume of the client's right radial pulse c. Auscultate the client's breath sounds bilaterally. d. Observe the amount and dose of morphine in the PCA pump syringe.

d. Observe the amount and dose of morphine in the PCA pump syringe.

A client has an intravenous fluid infusing in the right forearm. To determine the client's distal pulse rate most accurately, which action should the nurse implement? a. Elevate the client's upper extremity before counting the pulse rate b. Auscultate directly below the IV site with a Doppler stethoscope c. Turn off the intravenous fluids that are infusing while counting the pulse. d. Palpate at the radial pulse site with the pads of two or three fingers.

d. Palpate at the radial pulse site with the pads of two or three fingers.

At 40 week gestation, a laboring client who is lying is a supine position tells the nurse that she has finally found a comfortable position. What action should the nurse take? a. Encourage the client to turn on her left side. b. Place a pillow under the client's head and knees. c. Explain to the client that her position is not safe. d. Place a wedge under the client's right hip.

d. Place a wedge under the client's right hip.

When entering a client's room, the nurse discovers that the client is unresponsive and pulseless. The nurse initiate CPR and Calls for assistance. Which action should the nurse take next? a. Prepare to administer atropine 0.4 mg IVP b. Gather emergency tracheostomy equipment c. Prepare to administer lidocaine at 100 mg IVP d. Place cardiac monitor leads on the client's chest.

d. Place cardiac monitor leads on the client's chest.

A client with cervical cancer is hospitalized for insertion of a sealed internal cervical radiation implant. While providing care, the nurse finds the radiation implant in the bed. What action should the nurse take? a. Call the radiology department b. Reinsert the implant into the vagina c. Apply double gloves to retrieve the implant for disposal. d. Place the implant in a lead container using long-handled forceps

d. Place the implant in a lead container using long-handled forceps

The nurse is planning to assess a client's oxygen saturation to determine if additional oxygen is needed via nasal cannula. The client has a bilateral below-the-knee amputation and pedal pulses that are weak and threaty. What action should the nurse take? a. Document that an accurate oxygen saturation reading cannot be obtained b. Elevate to client's hands for five minutes prior to obtaining a reading from the finger c. Increase the oxygen based on the clients breathing patterns and lung sounds d. Place the oximeter clip on the ear lobe to obtain the oxygen saturation reading

d. Place the oximeter clip on the ear lobe to obtain the oxygen saturation reading

Which intervention should the nurse implement for a client with a superficial (first degree) burn? a. Spray an anesthetic agent over the burn every 3 to 4 hours b. Position the burn victim in front of a cool fan to decrease discomfort c. Apply ice pack for 30 mints to lower surface temperature d. Place wet clothes on the burned areas for short periods of time.

d. Place wet clothes on the burned areas for short periods of time.

The nurse is teaching a postmenopausal client about osteoporosis prevention. The client reports that she smokes 2 packs of cigarettes a day and takes 750 mg calcium supplements daily. What information should the nurse include when teaching this client about osteoporosis prevention? a. Family history is more important than calcium intake in determining the occurrence of osteoporosis b. Calcium should be taken once a day, preferable at the same time of day c. Smoking cessation is more important than calcium intake in preventing osteoporosis. d. Postmenopausal women need an intake of at least 1,500 mg of calcium daily.

d. Postmenopausal women need an intake of at least 1,500 mg of calcium daily.

The nurse is planning preoperative teaching plan of a 12-years old child who is scheduled for surgery. To help reduce the child anxiety, which action is the best for the nurse to implement? a. Give the child syringes or hospital mask to play it at home prior to hospitalization. b. Include the child in pay therapy with children who are hospitalized for similar surgery. c. Provide a family tour of the preoperative unit one week before the surgery is scheduled. d. Provide doll an equipment to re-enact feeling associated with painful procedures

d. Provide doll an equipment to re-enact feeling associated with painful procedures

When assessing the surgical dressing of a client who had abdominal surgery the previous day, the nurse observes that a small amount of drainage is present on the dressing and the wound's Hemovac suction device is empty with the plug open. How should the nurse respond? a. Replace the dressing and remove the drainage device b. Reposition the drainage device and keep the plug open c. Notify the healthcare provider that the drain is not working d. Recompress the wound suction device and secure to plug

d. Recompress the wound suction device and secure to plug

When gathering for a group therapy session at 1400 hours, a female client complains to the nurse that a smoking break has not been allowed all day. The nurse responds that 15 minute breaks were called over the unit intercom after breakfast and after lunch. The nurse is using what communication technique in responding to the client? a. Doubt b. Observation c. Confrontation d. Reflection

d. Reflection

Which instruction is most important for the nurse to provide a client who receives a new plan of care to treat osteoporosis? a. Begin a weight-bearing exercise plan b. Increase intake of foods rich in calcium c. Schedule a bone density tests every year. d. Remain upright after taking the medication.

d. Remain upright after taking the medication.

Following an outbreak of measles involving 5 students in an elementary school, which action is most important for the school nurse to take? a. Review the immunization records of all children in the elementary school b. Report the measles outbreak to all community health organizations c. Schedule a mobile public health vehicle to offer measles inoculations to unvaccinated children. d. Restrict unvaccinated children from attending school until measles outbreak is resolved.

d. Restrict unvaccinated children from attending school until measles outbreak is resolved.

An Insulin infusion for a client with diabetes mellitus who is experiencing hyperglycemic hyperosmolar...in addition to the client's glucose, which laboratory value is most important for the nurse to monitor? a. Urine ketones b. Urine albumin c. Serum protein d. Serum potassium

d. Serum potassium

When assessing a 6-month old infant, the nurse determines that the anterior fontanel is bulging. In which situation would this finding be most significant? a. Crying b. Straining on stool c. Vomiting d. Sitting upright.

d. Sitting upright.

A nurse-manager is preparing the curricula for a class for charge nurses. A staffing formula based on what data ensures quality client care and is most cost-effective? a. Client geographic location and age b. Number of staff and number of clients c. Weekend and weekday staff availability d. Skills of staff and client acuity

d. Skills of staff and client acuity

An older male client with a history of type 1 diabetes has not felt well the past few days and arrives at the clinic with abdominal cramping and vomiting. He is lethargic, moderately, confused, and cannot remember when he took his last dose of insulin or ate last. What action should the nurse implement first? a. obtain a serum potassium level b. administer the client's usual dose of insulin c. assess pupillary response to light d. Start an intravenous (IV) infusion of normal saline

d. Start an intravenous (IV) infusion of normal saline

The mother of an adolescent tells the clinic nurse, "My son has athlete's foot, I have been applying triple antibiotic ointment for two days, but there has been no improvement." What instruction should the nurse provide? a. Antibiotics take two weeks to become effective against infections such as athlete's foot. b. Continue using the ointment for a full week, even after the symptoms disappear. c. Applying too much ointment can deter its effectiveness. Apply a thin layer to prevent maceration. d. Stop using the ointment and encourage complete drying of the feet and wearing clean socks.

d. Stop using the ointment and encourage complete drying of the feet and wearing clean socks.

A client with a history of diabetes and coronary artery disease is admitted with shortness of breath, anxiety, and confusion. The client's blood pressure is 80/60 mmHg, heart rate 120 beats/minute with audible third and fourth heart sounds, and bibasilar crackles. The client's average urinary output is 5 ml/hour. Normal saline is infusing at 124 ml/hour with a secondary infusion of dopamine at mcg/kg/minute per infusion pump. With intervention should the nurse implement? a. Irrigate the indwelling urinary catheter. b. Prepare the client for external pacing. c. Obtain capillary blood glucose measurement. d. Titrate the dopamine infusion to raise the BP.

d. Titrate the dopamine infusion to raise the BP.

A client who is admitted to the intensive care unit with a right chest tube attached to a THORA-SEAL chest drainage unit becomes increasingly anxious and complain of difficulty breathing. The nurse determine the client is tachypneic with absent breath sounds in the client's right lungs fields. Which additional finding indicates that the client has developed a tension pneumothorax? a. Continuous bubbling in the water seal chamber b. Decrease bright red blood drainage c. Tachypnea and difficulty breathing d. Tracheal deviation toward the left lung.

d. Tracheal deviation toward the left lung.

A client in the postanesthesia care unit (PACU) has an eight (8) on the Aldrete postanesthesia scoring system. What intervention should nurse implement a. The client should be kept in the recovery room b. Assess the client's respiratory status immediately c. Notify the client's surgeon immediately d. Transfer the client to the surgical floor.

d. Transfer the client to the surgical floor.

A male adult is admitted because of an acetaminophen overdose. After transfer to the mental health unit, the client is told he has liver damage. Which information is most important for the nurse to include in the client's discharge plan?

do not take any over the counter meds

The nurse needs to add a medication to a liter of 5% Dextrose in Water (D5W) that is already infusing into a client. At what location should the nurse inject the medication?

medication port

A male client is admitted with a bowel obstruction and intractable vomiting for the last several hours despite the use of antiemetics. Which intervention should the nurse implement first? A. pH 7.50; PaCo2 42; HCO3 33; pO2 92. Infuse 0.9 % sodium chloride 500 ml bolus. Metabolic alkalosis. B. Insert nasogastric tube to intermittent suction. C. Maintain head of bed at 45 degrees D. Document strict intake and output

pH 7.50; PaCo2 42; HCO3 33; pO2 92. Infuse 0.9 % sodium chloride 500 ml bolus. Metabolic alkalosis.

A nurse who works in the nursery is attending the vaginal delivery of a term infant. What action should the nurse complete prior to leaving the delivery room?

place the id bands on the infant and mother

The development of atherosclerosis is a process of sequential events. Arrange the pathophysiological events in orders of occurrence. (Place the first event on top and the last on the bottom) 1. Arterial endothelium injury causes inflammation 2. Macrophages consume low density lipoprotein (LDL), creating foam cells 3. Foam cells release growth factors for smooth muscle cells 4. Smooth muscle grows over fatty streaks creating fibrous plaques 5. Vessel narrowing results in ischemia.

1. Arterial endothelium injury causes inflammation 2. Macrophages consume low density lipoprotein (LDL), creating foam cells 3. Foam cells release growth factors for smooth muscle cells 4. Smooth muscle grows over fatty streaks creating fibrous plaques 5. Vessel narrowing results in ischemia

During change of shift, the nurse reports that a male client who had abdominal surgery yesterday increasingly confused and disoriented during the night. He wandered into other clients rooms, saying that there are men in his room trying to hurt him. Because of continuing disorientation and the client's multiple attempts to get of bed, soft restrains were applied at 0400. In what order should the nurse who is receiving report implement these interventions? (Arrange from first action on top to last on the bottom) 1. Assess the client's skin and circulation for impairment related to the restrains 2. Evaluate the client's mentation to determine need to continue the restrains 3. Assign unlicensed assistive personnel to remove restrains and remain with client 4. Contact the client's surgeon and primary healthcare provider

1. Assess the client's skin and circulation for impairment related to the restrains 2. Evaluate the client's mentation to determine need to continue the restrains 3. Assign unlicensed assistive personnel to remove restrains and remain with client 4. Contact the client's surgeon and primary healthcare provide

A client admitted to the telemetry unit is having unrelieved chest pain after receiving 3 sublingual nitroglycerin tablets and morphine 8 mg IV. The electrocardiogram reveals sinus bradycardia with ST elevation. In what order should the nurse implement the nursing actions? (Arrange first to last) Move the crash cart to the client room Call the rapid response team to assist Inform the family of the critical situation Notify the client's healthcare provider

1. Call the rapid response team to assist 2. Move the crash cart to the client room 3. Notify the client's healthcare provider 4. Inform the family of the critical situation

A client is complaining of intermittent, left, lower abdominal pain that began two days ago...implement the following interventions?

1. Determine when the client had last bowel movement 2. Position client supine with knees bent 3. Inspect abdominal contour 4. Auscultate all four abdominal quadrants

A client diagnosed with bipolar disorder is going home on a week-end pass. Which suggestions should give the client's family to help them prepare for the visit? 1. Discuss the importance of continuing the usual at-home activities 2. Encourage the family to plan daily activities to keep the client busy 3. Have friends and family visit the client at a welcome party. 4. Instruct family to monitor the client's choice of television programs.

1. Discuss the importance of continuing the usual at-home activities

The nurse is collecting a sterile urine specimen using a straight catheter tray for culture.... (Arrange from first action to last). Use forceps and swaps to clean the urinary meatus Drape the client in a recumbent position for privacy Open the urinary catheterization tray Don sterile gloves using aseptic technique

1. Drape the client in a recumbent position for privacy 2. Open the urinary catheterization tray 3. Don sterile gloves using aseptic technique 4. Use forceps and swaps to clean the urinary meatus

An unlicensed assistive personnel UAP leaves the unit without notifying the staff. In what order should the unit manager implement this intervention to address the UAPs behavior? (Place the action in order from first on top to last on bottom.) 1. Note date and time of the behavior. 2. Discuss the issue privately with the UAP. 3. Plan for scheduled break times. 4. Evaluate the UAP for signs of improvement.

1. Note date and time of the behavior. 2. Discuss the issue privately with the UAP. 3. Plan for scheduled break times. 4. Evaluate the UAP for signs of improvement.

The nurse is using a straight urinary catheter kit to collect a sterile urine specimen from a female client. After positioning am prepping this client, rank the actions in the sequence they should be implemented. (Place to first action on the top on the last action on the bottom.) 1. Open the sterile catheter kit close to the client's perineum. 2. Don sterile gloves and prepare to sterile field 3. Cleanse the urinary meatus using the solution, swabs, and forceps provided 4. Place distal end of the catheter in sterile specimen cup and insert catheter into meatus

1. Open the sterile catheter kit close to the client's perineum. 2. Don sterile gloves and prepare to sterile field 3. Cleanse the urinary meatus using the solution, swabs, and forceps provided 4. Place distal end of the catheter in sterile specimen cup and insert catheter into meatus

Which action should the nurse implement with auscultating anterior breath sounds? (Place the first action on top and last action on the bottom) Auscultate bronchovesicular sounds from side to side the first and second intercostal spaces Place stethoscope in suprasternal area to auscultate for bronchial sounds Document normal breath sounds and location of adventitious breath sounds Displace female breast tissue and apply stethoscope directly on chest wall to hear vesicular sounds

1. Place stethoscope in suprasternal area to auscultate for bronchial sounds 2. Auscultate bronchovesicular sounds from side to side the first and second intercostal spaces 3. Displace female breast tissue and apply stethoscope directly on chest wall to hear vesicular sounds 4. Document normal breath sounds and location of adventitious breath sounds

Which actions should the nurse implement with auscultating anterior breath sounds? (Place the first action on top and last action on the bottom.) 1. Place stethoscope in suprasternal area to auscultate from bronchial sounds 2. Auscultate bronchovesicular sounds from side to side of the first and second intercostal spaces 3. Displace female breast tissue and apply stethoscope directly on chest wall to hear vesicular sounds 4. Document normal breath sounds and location of adventitious breath sounds

1. Place stethoscope in suprasternal area to auscultate from bronchial sounds 2. Auscultate bronchovesicular sounds from side to side of the first and second intercostal spaces 3. Displace female breast tissue and apply stethoscope directly on chest wall to hear vesicular sounds 4. Document normal breath sounds and location of adventitious breath sounds

When washing soiled hands, the nurse first wets the hands and applies soap. The nurse should complete additional actions in which sequence? (Arrange from first action on top last action on bottom.) 1. Rub hands palm to palm. 2. Interlace the fingers, 3. Dry hands with paper towel. 4. Turn off the water faucet.

1. Rub hands palm to palm. 2. Interlace the fingers, 3. Dry hands with paper towel. 4. Turn off the water faucet.

The nurse is caring for a 4-year-old male child who becomes unresponsive as his heart rate decreases to 40 beats/minute. His blood pressure is 88/70 mmHg, and his oxygen saturation is 70% while receiving 100% oxygen by non-rebreather face mask. In what sequence, from first to last, should the nurse implement these actions? (Place the first action on top and last action on the bottom.) 1. Start chest compressions with assisted manual ventilations 2. Administer epinephrine 0.01 mg/kg intraosseous (IO) 3. Apply pads and prepare for transthoracic pacing 4. Review the possible underlying causes for bradycardia.

1. Start chest compressions with assisted manual ventilations 2. Administer epinephrine 0.01 mg/kg intraosseous (IO) 3. Apply pads and prepare for transthoracic pacing 4. Review the possible underlying causes for bradycardia

The HCP prescribes methotrexate 7.5 mg PO weekly, in 3 divides doses for a child with rheumatoid arthritis whose body surface area (BSA) is 0.6 m2. The therapeutic dosage of methotrexate PO is 5 to 15 mg/m2/week. How many mg should the nurse administer in each of the three doses given weekly? (Enter the numeric value only. If round is required, round to the nearest tenth.)

1.5

A client is receiving and oral antibiotic suspension labeled 250 mg/2ml. The healthcare provider prescribes 200mg every 6 hours. How many ml should the nurse administer at each dose? (Enter numerical value only. If rounding is required, round to the nearest tenth)

1.6

The healthcare provider prescribes Morphine Sulfate Oral Solution 38 mg PO q4 hours for a client who is opioid-tolerant. The available 30 mL bottle is labeled, 100 mg/5 mL (20mg/mL), and is packaged with a calibrated oral syringe to provide to provide accurate dose measurements. How many mL should the nurse administer? (Enter the numerical value only. If rounding is required, round to the nearest tenth.)

1.9

A client is receiving an IV solution labeled Heparin Sodium 20,000 Units in 5% dextrose injection 500 ml at 25 ml/hour. How many units of heparin is the client receiving each hour?

1000

An IV antibiotic is prescribed for a client with a postoperative infection. The medication is to be administered in 4 divided doses. What schedule is best for administering this prescription

1000, 1600, 2200, 0400

The healthcare provider prescribes heparin protocol at 18 units/kg/hr for a client with a possible pulmonary embolism. This client weighs 144 pounds. The available solution is labeled, heparin sodium 25,000 units in 5% dextrose 250 ml. the nurse should program the pump to deliver how many ml/hr? (Enter numeric value only. If rounding is require round to the nearest whole number.)

12$ A client is receiving an IV of heparin sodium 25000 units in 5% dextrose injection 500 ml at 14 ml/hour...verify that the client is receiving the prescribed amount of heparin. How many units is the client receiving @700

The nurse uses the parkland formula (4ml x kg x total body surface area = 24 hours fluid replacement) to calculate the 24-hours IV fluid replacement for a client with 40% burns who weighs 76kg. How many ml should the client receive? (Enter numeric value only.)

12,160

The healthcare provider prescribes potassium chloride 25 mEq in 500 ml D_5W to infuse over 6 hours. The available 20 ml vial of potassium chloride is labeled, "10 mEq/5ml." how many ml of potassium chloride should the nurse add the IV fluid? (Enter numeric value only. If is rounding is required, round to the nearest tenth.

12.5

A client currently receiving an infusion labeled Heparin Sodium 25,000 Units in 5% Dextrose Injection 500 mL at 14 mL/hour. A prescription is received to change the rate of the infusion to 900 units of Heparin per hour. The nurse should set the infusion pump to deliver how many mL/hour? (Enter numeric value only)

18

The nurse notes the client receiving heparin infusion labeled, Heparin Na 25,000 Units in 5% Dextrose injection 500 ml at 50ml/hr. What dose of Heparin is the client receiving per hour?

2500

A client is receiving an IV solution of nitroglycerin 100mg/500ml D5W at 10 mcg/ minute. The nurse should program the infusion pump to deliver how many ml/hour? ( Enter numeric value only)

3 ml/hour

An adult client experiences a gasoline tank fire when riding a motorcycle and is admitted to the emergency department (ED) with full thickness burns to all surfaces of both lower extremities. What percentage of body surface area should the nurse document in the electronic medical record (EMR)?

36%

During the admission assessment, the nurse auscultates heart sounds for a client with no history of cardiovascular disease. Where should the nurse listen when assessing the client's point of maximal impulse (PMI) (Click the chosen location. To change, click on a new location)

4-5th intercostal space midclavicular

The nurse mixes 250 mg of debutamine in 250 ml of D5W and plans to administer the solution at rate client weighing 110 pounds. The nurse should set the infusion pump to administer how many ml per hour only. If rounding is required, round the nearest whole number.)

45

Dopamine protocol is prescribed for a male client who weigh 198 pounds to maintain the mean arterial pressure (MAP) greater than 65 mmHg. His current MAP is 50 mmHg, so the nurse increases the infusion to 7 mcg/kg/minute. The infusion is labeled dextrose 5% in water (D5W) 500 ml with dopamine 400 mg. The nurse should program the infusion pump to deliver how many ml/hour?

47

A 154 pound client with diabetic ketoacidosis is receiving an IV of normal saline 100 ML with regular insulin 100 units. The healthcare provider prescribes a rate of 0.1 units/kg/hour. To deliver the correct dosage, the nurse should set the infusion pump to Infuse how many ml/hour? enter numeric value only

7

The healthcare provider prescribes an IV solution of isoproterenol (Isuprel) 1 mg in 250 ml of D5W at 300 mcg/hour. The nurse should program the infusion pump to deliver how many ml/hour? (Enter numeric value only.)

75

A female client reports that she drank a liter of a solution to cleanse her intestines... immediately. How many ml of fluid intake should the nurse document? Whole number

760

The nurse is preparing a heparin bolus dose of 80 units/kg for a client who weighs 220 pounds. Heparin sodium injection, USP is available in a 3o ml multidose vial with the concentration of 1,000 USP units/ml. how many ml of heparin should the nurse administer? (Enter numeric value only)

8

A 16-year-old adolescent with meningococcal meningitis is receiving a continuous IV infusion of penicillin G, which is prescribed as 20 million units in a total volume of 2 liters of normal saline every 24 hr. The pharmacy delivers 10 million units/ liters of normal saline. How many ml/hr should the nurse program the infusion pump? (Enter numeric value only. If rounding is required, round to the nearest whole number.)

83

The nurse suspect may be hemorrhaging internally. Which findings of an orthostatic test may indicate to the nurse of major bleed?

A decrease in the systolic b/p of 10mm/hg with a corresponding increase of heart rate of 20.

A young adult female client with recurrent pelvic pain for 3 year returns to the clinic for relief of severe dysmenorrhea. The nurse reviews her medical record which indicates that the client has endometriosis. Based on this finding, what information should the nurse provide this client? A) Oral contraceptives increase the symptoms of endometriosis. B) The symptoms of endometriosis can increase with menopause. C) An option to diagnose disease extent and provide therapeutic treatment is laparoscopy. D) Infertile is successfully treated with removal of intra-abdominal endometrial lesions.

A) Oral contraceptives increase the symptoms of endometriosis.

A 59-year-old male client comes to the clinic and reports his concern over a lump that, "just popped up on my neck about a week ago." In performing an examination of the lump, the nurse palpates a large, nontender, hardened left subclavian lymph node. There is not overlying tissue inflammation. What do these findings suggest A.)Malignancy B.)Bacterial infection C.)Viral infection D.)Lymphangitis

A.) A.)Malignancy

The nurse is caring for four clients...postoperative hemoglobin of 8.7 mg/dl; client C, newly admitted with potassium...an appendectomy who has a white blood cell count of 15,000mm3. What intervention... A.)Determine the availability of two units of packed cells in the blood bank for client B B.)Increase the oxygen flow rate to 4 liters/minute per face mask for client A C.)Remove any foods, such as banana or orange juice, for the breakfast tray for client C D.)Inform client D that surgery is likely to be delayed until the infection responds to antibiotics

A.)Determine the availability of two units of packed cells in the blood bank for client B

A male client is admitted with a severe asthma attack. For the last 3 hours he has experienced increased shortness of breath. His arterial blood gas results are: pH 7.22 PaCO2 55 mmHg; HCO3 25 mEq/L or mmol/L (SI). Which intervention should the nurse implement?

Administer PRN dose of albuterol

A client arrives in the emergency center with a blood alcohol level of 500 mg/dl. When transferred to the observation unit, the client becomes demanding, aggressive, and shouts at the staff. Which assessments finding is most important for the nurse to identify in the first 24 hours?

Agitation and threats to harms staff

The nurse is triaging several children as they present to the emergency room after an accident. Which child requires the most immediate intervention by the nurse?

An 11-year-old with a headache, nausea, and projectile vomiting

An antacid is prescribed for a client with gastroesophageal (GERD). The client asks the nurse, "How does this help my GERD?" What is the best response by the nurse?

Antacids will neutralize the acid in your stomach

Which class of drugs is the only source of a cure for septic shock?

Antiinfectives

The healthcare provider explains through an interpreter the risks and benefits of a scheduled surgical procedure to a non-English speaking female client. The client gives verbal consent and the healthcare provider leaves, instructing the nurse to witness the signature on the consent form. The client and the interpreter then speak together in the foreign language for an additional 2 minutes until the interpreter concludes, "She says it is OK." What action should the nurse take next?

Ask for a full explanation from the interpreter of the witnessed discussion

After administering a proton pump inhibitor (PPI), which action should the nurse take to evaluate the effectiveness of the medication?

Ask the client about gastrointestinal pain

A female client is admitted with end stage pulmonary disease is alert, oriented, and complaining of shortness of breath. The client tells the nurse that she wants "no heroic measures" taken if she stops breathing, and she asks the nurse to document this in her medical record. What action should the nurse implement

Ask the client to discuss "do not resuscitate" with her healthcare provider

A male client notifies the nurse that he feels short of breath and has chest pressure radiating down his left arm. A STAT 12-lead electrocardiogram (ECG) is obtained and shows ST segment elevation in leads II, II, aVF and V4R. The nurse collects blood samples and gives a normal saline bolus. What action is most important for the nurse to implement?

Asses for contraindications for thrombolytic therapy

A preoperative client states he is not allergic to any medications. What is the most important nursing action for the nurse to implement next?

Assess client's knowledge of an allergy response

The nurse is arranging home care for an older client who has a new colostomy following a large bowel resection three day. The clients plan to live with a family member. Which action should the nurse implement? Select all that apply

Assess the client for self-care ability Provide pain medication instructions Teach care of ostomy to care provider

A male client with rheumatoid arthritis is schedule for a procedure in the morning. The... unable to complete the procedure because of early morning stiffness. Which intervention... implement?

Assign a UAP to assist the client with a warm shower early in the morning

While removing staples from a male client's postoperative wound site, the nurse observes that the client's eyes are closed and his face and hands are clenched. The client states, "I just hate having staples removed." After acknowledging the client's anxiety, what action should the nurse implement?

Attempt to distract the client with general conversation

A client in the intensive care unit is being mechanically ventilated, has an indwelling urinary catheter in place, an exhibiting signs of restlessness. Which action should the nurse take fist? Administer PRN dose of lorazepam Auscultate bilateral breath sounds Check urinary catheter for obstruction Review the heart rhythms on cardiac monitor.

Auscultate bilateral breath sounds

After changing to a new brand of laundry detergent, an adult male reports that he has a fine itchy rash. Which assessment finding warrants immediate intervention by the nurse?

Bilateral Wheezing.

A female client who is admitted to the mental health unit for opiate dependency is receiving clonidine 0.1 mg PO for withdrawal symptoms. The client begins to complain of feeling nervous and tells the nurse that her bones are itching. Which finding should the nurse identify as a contraindication for administering the medication?

Blood pressure 90/76 mm Hg

A child is diagnosed with acquired aplastic anemia. The nurse knows that this child has the best prognosis with which treatment regimen?

Bone marrow transplantation

The healthcare provider prescribes carboprost tromethamine (Hemabate) 250 mcg IM for a multigravida postpartum client who is experiencing heavy, bright red vaginal bleeding. Prior to administering this medication, which interventions should the RN implement? A.)Obtain a second IV access. B).Decrease the room temperature. C.)Give the prescribed antiemetic. D.)Insert an indwelling catheter.

C).Give the prescribed antiemetic.

The mother of a child recently diagnosed with asthma asks the nurse how to help protect her child from having asthmatic attacks. To avoid triggers for asthmatic attacks, which instructions should the nurse provide the mother? (Select all that apply)

Close car windows and use air conditioner Avoid sudden changes in temperature Keep away from pets with long hair Stay indoors when grass is being cut

Which assessment finding of a postmenopausal woman necessitates a referral by the nurse to the healthcare provider for evaluation of thyroid functioning

Cold sensitivity

A client's subjective data includes dysuria, urgency, and urinary frequency. What action should the nurse implement next?

Collect a clean-catch specimen

The nurse is caring for a client following a myelogram. Which assessment finding should the nurse report to the healthcare provider immediately?

Complain of headaches and stiff neck

A nurse plans to call the healthcare provider to report an 0600 serum potassium level of 2 mEq/L or mmol/L (SI), but the charge nurse tells the nurse that the healthcare provider does not like to receive early morning calls and will make rounds later in the morning. What action should the nurse make?

Contact the healthcare provider immediately to report the laboratory value regardless of the advice

A client is admitted with a wound on the right hand and associated cellulitis. In assessing the client's hand, which finding required most immediate follow-up by the nurse?

Cyanotic nailbeds

An elderly female client with osteoarthritis reports increasing pain and stiffness in her right knee and asks how to reduce these symptoms. In responding to the client, the nurse recognizes what pathology as the cause of her symptoms?

Destruction of joint cartilage.

During a clinic visit, a client with a kidney transplant ask, "What will happen if chronic rejection develops?" which response is best for the nurse to provide

Dialysis would need to be resumed if chronic rejection becomes a reality

A female client reports she has not had a bowel movement for 3 days, but now is defecating frequent small amount of liquid stool. Which action should the nurse implement?

Digitally check the client for a fecal impaction

While the nurse is preparing a scheduled intravenous (IV) medication, the client states that the IV site hurts and refuses to allow the nurse to administer a flush to assess the site. Which intervention should the nurse implement?

Discontinue the painful IV after a new IV is inserted

In assessing a client 48 hours following a fracture, the nurse observes ecchymosis at the fracture site, and recognizes that hematoma formation at the bone fragment site has occurred. What action should the nurse implement?

Document the extend of the bruising in the medical record

While completing an admission assessment for a client with unstable angina, which closed questions should the nurse ask about the client's pain?

Does your pain occur when walking short distances?

The nurse is caring for a client who is experiencing a tonic-clonic seizure. Which actions should the nurse implement? (Select all that apply)

Ease the client to the floor Loosen restrictive clothing Note the duration of the seizure

While a child is hospitalized with acute glomerulonephritis, the parents ask why blood pressure readings are taken so often. Which response by the nurse is most accurate?

Elevated blood pressure must be anticipated and identified quickly

A male client tells the nurse that he is concerned that he may have a stomach ulcer, because he is experiencing heartburn and a dull growing pain that is relieved when he eats. What is the best response by the nurse?

Encourage the client to obtain a complete physical exam since these symptoms are consistent with an ulcer

After administering an antipyretic medication. Which intervention should the nurse implement?

Encouraging liberal fluid intake

A client with rapid respirations and audible rhonchi is admitted to the intensive care unit because of a pulmonary embolism (PE). Low-flow oxygen by nasal cannula and weight based heparin protocol is initiated. Which intervention is most important for the nurse to include in this client's plan of care?

Evaluate daily blood clotting factors.

The nurse enters a client's room to administer scheduled daily medications and observes the client leaning forward and using pursed lip breathing. Which action is most important for the nurse to implement first?

Evaluate the oxygen saturation

A female client on the mental health unit frequently asks the nurse when she can be discharged. Then, becoming more anxious, she begins to pace the hallway. What intervention should the nurse implement first?

Explore the client's reasons for wanting to be discharged.

The nurse is assessing a primigravida a 39-weeks gestation during a weekly prenatal visit. Which finding is most important for the nurse to report to the healthcare provider?

Fetal heart rate of 200 beats/minute

The nurse discovers that an elderly client with no history of cardiac or renal disease has an elevated serum magnesium level. To further investigate the cause of this electrolyte imbalance, what information is most important for the nurse to obtain from the client's medical history

Frequency of laxative use for chronic constipation

When conducting diet teaching for a client who was diagnosed with a myocardial infarction, which snack foods should the nurse encourage the client to eat? (Select all that apply).

Fresh turkey slices and berries raw unsalted almonds and apples

A young adult female with chronic kidney disease (CKD) due to recurring pyelonephritis is hospitalized with basilar crackles and peripheral edema. She is complaining of severe nausea and the cardiac monitor indicates sinus tachycardia with frequent premature ventricular contraction. Her blood pressure is 200 /110 mm Hg, and her temperature is 101 F which PRN medication should the nurse administers first?

Furosemide

A client's telemetry monitor indicates ventricular fibrillation (VF). After delivering one counter shock, the nurse resumes chest compression, after another minute of compression , the client's rhythm converts to supraventricular tachycardia (SVT) on the monitor, at this point , what is the priority intervention for the nurse?

Give IV dose of adenosine rapidly over 1-2 seconds.

During a staff meeting, a nurse verbally attacks the nurse manager conducting the meeting, stating, "you always let your favorites have holidays off give then easier assignments. You are unfair and prejudiced" how should the nurse-manager respond?

Give me specific examples to support your statements.

A middle-aged woman, diagnosed with Graves' disease, asks the nurse about this condition. Which etiological pathology should the nurse include in the teaching plan about hyperthyroidism? (Select all that apply.) Graves' disease, an autoimmune condition, affects thyroid stimulating hormone receptors. T3 and T4 hormone levels are increased Large protruding eyeballs are a sign of hyperthyroid function Weight gain is a common complaint in hyperthyroidism Early treatment includes levothyroxine (Synthroid).

Graves' disease, an autoimmune condition, affects thyroid stimulating hormone receptors. T3 and T4 hormone levels are increased Large protruding eyeballs are a sign of hyperthyroid function

A 60-year-old female client asks the nurse about hormones replacement therapy (HRT) as a means preventing osteoporosis. Which factor in the client's history is a possible contraindication for the use of HRT?

Her mother and sister have a history of breast cancer

A 46-year-old male client who had a myocardial infarction 24-hours ago comes to the nurse's station fully dressed and wanting to go home. He tells the nurse that he is feeling much better at this time. Based on this behavior, which nursing problem should the nurse formulate?

Ineffective coping related to denial

The nurse should teach the parents of a 6 year-old recently diagnosed with asthma that the symptom of acute episode of asthma are due to which physiological response?

Inflammation of the mucous membrane & bronchospasm

Sublingual nitroglycerin is administered to a male client with unstable angina who complains of crushing chest pain. Five minutes later the client becomes nauseated and his bloods pressure drops to 60/40. Which intervention should the nurse implement?

Infuse a rapid IV normal saline bolus

A mother brings her 3-week-old son to the clinic because he is vomiting "all the time." In performing a physical assessment, the nurse notes that the infant has poor skin turgor, has lost 20% of his birth weight, and has a small palpable oval-shaped mass in his abdomen. What intervention should the nurse implement first?

Initiate a prescribed IV for parental fluid

An adult is admitted to the emergency department following ingestion of a bottle of antidepressants secondary to chronic paint. A nasogastric tube and a left subclavian venous catheter are placed. The nurse auscultates audible breath sounds on the right side, faint sounds procedure should the nurse prepare for first?

Insertion of a left- sided chest tube.

A client with pneumonia has arterial blood gases levels at: PH 7.33; PaCO2 49 mm/hg; HCO3 25 mEq/L; PaO2 95. What intervention should the nurse implement based on these results

Institute coughing and deep breathing protocols

During a left femoral artery aortogram, the healthcare provider inserts an arterial sheath and initiate...through the sheath to dissolve an occluded artery. Which interventions should the nurse implement?

Instruct the client to keep the left leg straight Observe the insertion site for a hematoma Circle first noted drainage on the dressing

A client with a new diagnosis of Raynaud's disease lives alone. Which instruction should the nurse include in the client's discharged teaching plan?

Keep room temperature 80

A client is receiving continuous bladder irrigation via a triple-lumen suprapubic catheter that was placed during prostatectomy. Which report by the unlicensed assistive personnel (UAP) requires intervention by the nurse?

Leakage around catheter insertion site

When entering a client's room to administer an 0900 IV antibiotic, the nurse finds that the client is engaged in sexual activity with a visitor. Which actions should the nurse implement?

Leave the room and close the door quietly

A client is admitted for cellulitis surrounding an insect bite on the lower, right arm and intravenous (IV) antibiotic therapy is prescribed. Which action should the nurse implement before performing venipuncture?

Lower the left arm below the level of the heart

A male client who had a small bowel resection acquired methicillin- resistant Staphylococcus aureus (MRSA) while hospitalized. He was treated and released, but is readmitted today because of diarrhea and dehydration. It is most important for the nurse to implement which intervention

Maintain contact transmission precautions

A client with a history of using illicit drugs intravenously is admitted with Kaposi's sarcoma. Which intervention should the nurse include in this client's admission plan of care?

Monitor for secondary infections.

A client is admitted to isolation with the diagnosis of active tuberculosis. Which infection control measures should the nurse implement?

Negative pressure environment

When caring for a client with traumatic brain injury (TBI) who had a craniotomy for increased intracranial pressure (ICP), the nurse assesses the client using the Glasgow coma scale (GCS) every two hours. For the past 8 hours the client's GCS score has been 14. What does this GCS finding indicate about the client?

Neurologically stable without indications of an increased IC

A client with hyperthyroidism who has not been responsive to medications is admitted for evaluation. What action should the nurse implement? (Click on each chart tab for additional information. Please scroll to the bottom right corner of each tab to view all information contained in the client's medical record.)

Notify the healthcare provider

One day after abdominal surgery, an obese client complains of pain and heaviness in the right calf. What action should the nurse implement?

Observe for unilateral swelling

The nurses observes that a postoperative client with a continuous bladder irrigation has a large blood clot in the urinary drainage tubing. What actions should the nurse perform first?

Observe the amount of urine in the client's urinary drainage bag.

A client with a peripherally inserted central catheter (PICC) line has a fever. What client assessment is most important for the nurse to perform?

Observe the antecubital fossa for inflammation.

A-12-years old boy has a body mass index (BMI) of 28, a systolic pressure and a glycosylated hemoglobin (HBA1C) of 7.8%. Which selection indicated that his mother understands the management of his diet

One whole-wheat bagel with cream cheese, two strips of bacon, six ounces of orange juice.

Which needle should the nurse use to administer intravenous fluids (IV) via a client's implanted port?

One with the clamp or The one that does not contain a needle

Following an open reduction of the tibia, the nurse notes bleeding on the client's cast. Which action should the nurse implement?

Outline the area with ink and check it every 15 minutes to see if the area has increased

While assisting a male client who has muscular dystrophy (MD) to the bathroom, the nurse observes that he is awkward and clumsy. When he expresses his frustration and complains of hip discomfort, which intervention should the nurse implement?

Place a portable toilet next to the bed

The nurse enters a client's room and observe the unlicensed assistive personnel (UAP) making an occupied bed as seen in the picture. What action should the nurse take first?

Place the side rails in an up position

While taking vital signs, a critically ill male client grabs the nurse's hand and ask the nurse not to leave. What action is best for the nurse to take?

Pull up a chair and sit beside the client's bed

The nurse is preparing a community education program on osteoporosis. Which instruction is helpful in preventing bone loss and promoting bone formation

Recommend weigh bearing physical activity

When checking a third grader's height and weight the school nurse notes that these measurements have not changed in the last year. The child is currently taking daily vitamins, albuterol, and methylphenidate for attention deficit hyperactivity disorder (ADHD). Which intervention should the nurse implement? A. Report findings to the parents. B. Document findings in the child's school file. C. Refer child to the family healthcare provider. D. Encourage child to get more sleep.

Refer child to the family healthcare provider

An 18-year-old female client is seen at the health department for treatment of condylomata acuminate (perineal warts) caused by the human papillomavirus (HPV). Which intervention should the nurse implement? a. Tell the client that the vaccine for HPV is not indicated b. Inform the client that warts do not return following cryotherapy c. Recommended the use of latex condoms to prevent HPV transmission. d. Reinforce the importance of annual papanicolaou (Pap) smears.

Reinforce the importance of annual papanicolaou (Pap) smears.

A female client with breast cancer who completed her first chemotherapy treatment today at an out-patient center is preparing for discharge. Which behavior indicates that the client understands her care needs

Rented movies and borrowed books to use while passing time at home

A gravida 2 para 1, at 38-weeks gestation, scheduled for a repeat cesarean section in one week, is brought to the labor and delivery unit complaining of contractions every 10 minutes. While assessing the client, the client's mothers enter the labor suite and says in a loud voice, "I've had 8 children and I know she's in labor. I want her to have her cesarean section right now!" what action should the nurse take? A.)Request the mother to leave the room B.)Tell the mother to stop speaking for the client C.)Request security to remove her from the room D.)Notify the charge nurse of the situation

A).Request the mother to leave the room

During the infusion of a second unit of packed red blood cells, the client's temperature increases from 99 to 101.6 f. which intervention should the nurse implement? A.)Stop the transfusion start a saline B.)Observe for a maculopapular rash C.)Report the fever to the blood bank D.)Give a PRN dose of acetaminophen

A).Stop the transfusion start a saline

A client is being treated for syndrome of inappropriate antidiuretic hormone (SIADH). On examination, the client has a weight gain of 4.4 lbs (2 kg) in 24 hours and an elev ated blood pressure. Which intervention should the nurse implement first? A. Ensure client takes a diuretic q AM B. Obtain serum creatinine levels daily C. Measure ankle circumference D. Monitor daily sodium intake

A. Ensure client takes a diuretic q AM

An adult male with schizophrenia who has been noncompliant in taking oral antipsychotic medications refuses a prescribed IM medication. Which action should the nurse take? A. Notify the healthcare provider of the client's refusal B. Administer an oral PRN medication for agitation C. Ask for staff assistance with administering the injection D. explain that oral medications will no longer be required

A. Notify the healthcare provider of the client's refusal

The home health nurse is preparing to make daily visits to a group of clients. Which client should the nurse visit first? A. A client with congestive heart failure who reports a 3 pound weight gain in the last two days B. An immobile client with a stage 3 pressure ulcer on the coccyx who is having low back pain C. A client diagnosed with chronic obstructive pulmonary disease (COPD) who is short of breath D. A terminally ill older adult who has refused to eat or drink anything for the last 48 hours

A. A client with congestive heart failure who reports a 3 pound weight gain in the last two days

The nurse is caring for a group of clients with the help of a practical nurse (PN). Which nursing actions should the nurse assign to the PN? (Select all that apply.) A. Administer a dose of insulin per sliding scale for a client with type 2 diabetes mellitus (DM). B. Obtain postoperative vital signs for a client one day following unilateral knee arthroplasty C. Perform daily surgical dressing change for a client who had an abdominal hysterectomy D. Initiate patient controlled analgesia (PCA) pumps for two clients immediately postoperative E. Start the second blood transfusion for a client twelve hours following a below knee amputation

A. Administer a dose of insulin per sliding scale for a client with type 2 diabetes mellitus (DM). B. Obtain postoperative vital signs for a client one day following unilateral knee arthroplasty C. Perform daily surgical dressing change for a client who had an abdominal hysterectomy

A client with a traumatic brain injury becomes progressively less responsive to stimuli. The client has a "Do Not Resuscitate" prescription, and the nurse observes that the unlicensed assistive personnel (UAP) has stopped turning the client from side to side as previously schedules. What action should the nurse take? A. Advise the UAP to resume positioning the client on schedule B. Encourage the UAP to provide comfort care measures only C. Assume total care of the client to monitor neurologic function D. Assign a practical nurse to assist the UAP in turning the client

A. Advise the UAP to resume positioning the client on schedule

A client is admitted to the surgical unit with symptoms of a possible intestinal obstruction. When preparing to insert a nasogastric (NG) tube, which intervention should the nurse implement? A. Elevate the head of the bed 60 to 90 degrees B. Measure from corner of mouth to angle of jaw C. Administer a PRN analgesic D. Assess for a gag reflex

A. Elevate the head of the bed 60 to 90 degrees

The husband of an older woman, diagnosed with pernicious anemia, calls the clinic to report that his wife still has memory loss and some confusion since she received the first dose of nasal cyanocobalamin two days ago. He tells the nurse that he is worried that she might be getting Alzheimer's disease. What action should the nurse take? A. Explain that memory loss and confusion are common with vitamin B12 deficiency B. Ask if the client is experiencing any change in bowl habits C. determine if the client is taking iron and folic acid supplements D. Encourage the husband to bring the client to the clinic for a complete blood count

A. Explain that memory loss and confusion are common with vitamin B12 deficiency

A client with multiple sclerosis is receiving beta-1b interferon every other day. To assess for possible bone marrow suppression caused by the medication, which serum laboratory test findings should the nurse monitor? (Select all that apply) A. Platelet count B. Red blood cell count (RBC) C. White blood cell count (WBC). D. Albumin and protein E. Sodium and potassium

A. Platelet count B. Red blood cell count (RBC) C. White blood cell count (WBC).

The nurse is assessing and elderly bedridden client. Which finding indicates that the turning and positioning schedule is effective in protecting the client's skin? A. Reddened skin areas disappear within 15 minutes of being turned and positioned. B. No complaints of pressure or pain are verbalized by the client after being turned C. Only small areas of redness remain longer than 30 min after the client

A. Reddened skin areas disappear within 15 minutes of being turned and positioned.

The nurse is caring for a 17-year-old male who fell 20 feet 5 months ago while climbing the side of a cliff and has been in a sustained vegetative state since the accident. Which intervention should the nurse implement? A. Talk directly to the adolescent while providing care. B. Monitor vital signs and neuro status every 2 hours. C. Inquire about food allergies and food likes and dislikes. D. Initiate open communication with the teen's parents.

A. Talk directly to the adolescent while providing care

An older client is admitted for repair of a broken hip. To reduce the risk for infection in the postoperative period, which nursing care interventions should the nurse include in the client's plan of care? (Select all that apply) A. Teach client to use incentive spirometer q2 hours while awake. B. Remove urinary catheter as soon as possible and encourage voiding. C. Maintain sequential compression devices while in bed. D. Administer low molecular weight heparin as prescribed E. Assess pain level and medicate PRN as prescribed.

A. Teach client to use incentive spirometer q2 hours while awake. B. Remove urinary catheter as soon as possible and encourage voiding.

In caring for a client who is receiving linezolid IV for nosocomial pneumonia, which assessment finding is most important for the nurse to report to the healthcare provider? A. Watery diarrhea B. Yellow-tinged sputum C. Increased fatigue D. Nausea and headache

A. Watery diarrhea

During orientation, a newly hired nurse demonstrates suctioning of a tracheostomy in a skills class, as seen in the video. After the demonstration, the supervising nurse expresses concern that the demonstrated procedure increased the client's risk for which problem? A. Infection B. Ineffective airway clearance C. Altered comfort D. Impaired gas exchange

A. infection

A client with Addison's crisis is admitted for treatment with adrenal cortical supplementation. Based on the client's admitting diagnosis, which findings require immediate action by the nurse? (Select all that apply) A.)Headache and tremors B.)Postural hypotension C.)Pallor and diaphoresis D.)Irregular heart beat E.)Plus 4 pitting edema

A.)Headache and tremors B).Postural hypotension C).Pallor and diaphoresis D).Irregular heart beat

The nurse reviews the laboratory findings of a client with an open fracture of the tibia. The white blood cell (WBC) count and erythrocyte sedimentation rate (ESR) are elevated. Before reporting this information to the healthcare provider, what assessment should the nurse obtain? A. Degree of skin elasticity B. Appearance of wound C. Bilateral pedal pulse force D. Onset of any bleeding

B. Appearance of wound

A male client with diabetes mellitus type 2, who is taking pioglitazone PO daily, reports to the nurse the recent onset of nausea, accompanied by dark-colored urine, and a yellowish cast to his skin. What instructions should the nurse provide? A. "You have become dehydrated from the nausea. You will need to rest and increase fluid intake" B. "you need to seek immediate medical assistance to evaluate the cause of these symptoms" C. A urine specimen will be needed to determine what kind of infection you have developed" D. use insulin per sliding scale until the nausea resolves, and then resume your oral medication

B. "you need to seek immediate medical assistance to evaluate the cause of these symptoms"

A male client with ulcerative colitis received a prescription for a corticosteroid last month, but because of the side effect he stopped taking the medication 6 year ago. Which finding warrants immediate intervention by the nurse? A. Hypotension and fever B. Anxiety and restlessness. C. Fluid retention D. Increased blood glucose.

B. Anxiety and restlessness.

Oxygen at 5l/min per nasal cannula is being administered to a 10 year old child with pneumonia. When planning care for this child, what principle of oxygen administration should the nurse consider? A. Taking a sedative at bedtime slows respiratory rate, which decreases oxygen? B. Avoid administration of oxygen at high levels for extended periods. C. Increase oxygen rate during sleep to compensate for slower respiratory rate. D. Oxygen is less toxic when it is humidified with a hydration source.

B. Avoid administration of oxygen at high levels for extended periods.

An unconscious client is admitted to the intensive care unit and is placed on a ventilator. The ventilator alarms continuously and the client's oxygen saturation level is 62%. What action should the nurse take first? A. Call respiratory therapy. B. Begin manual ventilation immediately. C. Monitor oxygen saturation levels q5 minutes. D. Silence the alarm and call the technician.

B. Begin manual ventilation immediately.

The unit clerk reports to the charge nurse that a healthcare provider has written several prescriptions that are illegible and it appears the healthcare provider used several unapproved abbreviations in the prescriptions. What actions should the charge nurse take? A. Complete and file an incident (variance) report B. Call the healthcare provider who wrote the prescription C. Contact the healthcare provider review board for instructions D. Report the situation to the house supervisor

B. Call the healthcare provider who wrote the prescription

A client who is at 10-weeks gestation calls the clinic because she has been vomiting for the past 24 hours. The nurse determines that the client has no fever. Which instructions should the nurse give to this client? A. Remain on clear liquids until the vomiting subsides B. Come to the clinic to be seen by a healthcare provider C. Make an appointment at the clinic if a fever occurs D. Take nothing by mouth until there is no more nausea

B. Come to the clinic to be seen by a healthcare provider

A 75-year-old female client is admitted to the orthopedic unit following an open reduction and internal fixation of a hip fracture. On the second postoperative day, the client becomes confused and repeatedly asks the nurse she is. What information for the nurse to obtain? A. Use of sleeping medications. B. History of alcohol use, C. Use of antianxiety medications, D. History of this behavior.

B. History of alcohol use,

A client with Addison's disease becomes weak, confused, and dehydrated following the onset of an acute viral infection. The client's laboratory values include; sodium 129 mEq/l (129mmol/l SI), glucose 54 mg/dl (2.97mmol/l SI) and potassium 5.3 mmol/l SI). When reporting the findings to the HCP, the nurse anticipates a prescription for which intravenous medications? A. Regular insulin. B. Hydrocortisone C. Broad spectrum antibiotic D. Potassium chloride

B. Hydrocortisone

A 17-year -old male is brought to the emergency department by his parents because he has been coughing and running a fever with flu-like symptoms for the past 24 hours. Which intervention should the nurse implement first? A. Obtain a chest X-ray per protocol. B. Place a mask on the client's face. C. Assess the client's temperature. D. Determine the client's blood pressure

B. Place a mask on the client's face.

During discharge teaching, an overweight client heart failure (HF) is asked to make a grocery list for the nurse to review. Which food choices included on the client's list should the nurse encourage? (Select all that apply) A. Canned fruit in heavy syrup. B. Plain, air-popped popcorn. C. Cheddar cheese cubes. D. Natural whole almonds. E. Lightly salted potato chips

B. Plain, air-popped popcorn. D. Natural whole almonds.

The nurse is collecting sterile sample for culture and sensitivity from a disposable three chamber water-seal drainage system connected to a pleural chest tube. The nurse should obtain the sample from which site on the drainage system? A. Stopper port located above the water-seal level B. Plastic tubing located at the chest insertion site C. Rubberized port at the bottom of collection chamber D. Tubbing located on the top of the suction chamber

B. Plastic tubing located at the chest insertion site

A mother calls the nurse to report that at 0900 she administered a PO dose of digoxin to her 4-month-old infant, but at 0920 the baby vomited the medicine, what instruction should the nurse provide to this mother A. Give another dose. B. Withhold this dose. C. Administer a half dose now. D. Mix the next dose with food.

B. Withhold this dose

A client is admitted with an exacerbation of heart failure secondary to COPD. Which observations by the nurse require immediate intervention to reduce the likelihood of harm to this client? (Select all that apply). A. A bedside commode is positioned near the bed B. A saline lock is present in the right forearm C. A full pitcher of water is on the bedside table D. The client is lying in a supine position in bed E. A low sodium diet tray was brought to the room

C. A full pitcher of water is on the bedside table D. The client is lying in a supine position in bed

A male client who was admitted with an acute myocardial infarction receives a cardiac diet with sodium restriction and complains that his hamburger is flavorless. Which condiment should the nurse offer? A. Pickle relish. B. Steak sauce. C. Fresh horseradish. D. Tomato ketchup.

C. Fresh horseradish

The nurse is ready to insert an indwelling urinary catheter as seen in the picture. At this point in the procedure, what actions should the nurse take before inserting the catheter? (Select all that apply)

C. Gently palpate the client's bladder for distention D. Hold the catheter 3 - 4 inches (7.5 - 10 cm) from its tip E. Secure the urinary drainage bag to the bed frame

An adolescent, whose mother recently died, comes to the school nurse complain headache. Which statement made by the students should warrant further explanation nurse? A. "I've had dreams about Mon since she died." B. "I've been very sad and cry a lot at night." C. "I miss Mon and would like to go see her'". D. " it's hard to concentrate on my homework"

C. I miss Mom and would like to go see her'".

An older male client with a history of diabetes mellitus, chronic gout, and osteoarthritis comes to the clinic with a bag of medication bottles. Which intervention should the nurse implement first? A. Record pain evaluation B. Assess blood glucose C. Identify pills in the bag D. Obtain a medical history

C. Identify pills in the bag

The nursing staff on a medical unit includes a registered nurse (RN), practical nurse (LPN), and unlicensed assistant personnel (UAP). Which task should the charge nurse assign to the RN?

Supervised a newly hired graduate nurse during an admission assessment

An adult client comes to the clinic and reports his concern over a lump that "just popped up on my neck about a week ago." In performing an examination of the lump, the nurse palpates a large, non-tender, hardened left subclavian lymph node. There is no overlying tissue inflammation. What do these finding suggest? A. Bacterial infection B. Lymphangitis C. Malignancy D. Viral infection

C. Malignancy

A male client with an antisocial personality disorder is admitted to an in-patient mental health unit for multiple substance dependency. When providing a history, the client justifies to the nurse his use of illicit drugs. Based on this pattern of behavior this client's history is most likely to include which finding? A. Phobias and panic attacks when confronted by authority figures. B. Suicidal ideations and multiple attempts C. Multiple convictions for misdemeanors and class B felonies. D. Delusions of grandiosity and persecution

C. Multiple convictions for misdemeanors and class B felonies.

The nurse is preparing to administer an IV dose of ciprofloxacin to a client with urinary tract infection. Which client data requires the most immediate intervention by the nurse? A. Urine culture positive for MRSA B. Serum sodium of 145 mEq/L (145 mmol/L SI) C. Serum creatinine of 4.5 mg/dl (398 mcmol/L SI) D. White blood cell count of of 12,000 mm3 (12 x 109/L SI)

C. Serum creatinine of 4.5 mg/dl (398 mcmol/L SI)

Immediately after extubation, a client who has been mechanically ventilated is placed on a 50% non-rebreather. The client is hoarse and complaining of a sore throat. Which assessment finding should the nurse report to the healthcare provider immediately? A. Blood tinged sputum B. Expiratory wheezing C. Upper airway stridor D. Oxygen saturations 90%

C. Upper airway stridor

After multiple attempts to stop drinking, an adult male is admitted to the medical intensive care unit (MICU) with delirium tremens. He is tachycardic, diaphoretic, restless, and disoriented. Which finding indicates a life- threatening condition? A.CIWA-Ar for alcohol withdrawal score of 30 B. Acute onset of unrelenting chest pain C. Widening QRS complexes and flat waves D. Intense tremor and involuntary muscle activity

C. Widening QRS complexes and flat waves

A preschooler with constipation needs to increase fiber intake. Which snack suggestion should the nurse provide? A. soft pretzels B. fruit-flavored yogurt C. oatmeal cookies D. low fat cheese sticks

C. oatmeal cookies

The nurse is preparing to administer an infusion of amino acid-dextrose total parenteral nutrition (TPN) through a central venous catheter (CVC) line. Which action should the nurse implement first? a- Attached de IV tubing to the central line. b- Check the TPN solution for cloudiness c- Set the infusion PUMP at the prescribed rate. d- Prime the IV tubbing with the TPN solution.

Check the TPN solution for cloudiness

A client with bleeding esophageal varices receives vasopressin (Pitressin) IV. What should the nurse monitor for during the IV infusion of this medication?

Chest pain and dysrhythmia

After diagnosis and initial treatment of a 3 year old with Cystic fibrosis, the nurse provides home care instructions to the mother, which statement by the child's mother indicates that she understands home care treatment to promote pulmonary functions?

Chest physiotherapy should be performed twice a day before a meal.

An older woman who was recently diagnosed with end stage metastatic breast cancer is admitted because she is experiencing shortness of breath and confusion. The client refuses to eat and continuously asks to go home. Arterial blood gases indicate hypoxia. Which intervention is most important for the nurse to implement?

Clarify end of life desires

While caring for a toddler receiving oxygen (02) via face mask, the nurse observes that the child's lips and nares are dry and cracked. Which intervention should the nurse implement A.)Ask the mother what she usually uses on the child's lips and nose B.)Apply a petroleum jelly (Vaseline) to the child's nose and lips C.)Use a topical lidocaine (Zylocaine viscous) analgesic for cracked lips D.)Use a water soluble lubricant on affected oral and nasal mucosa

D).Use a water soluble lubricant on affected oral and nasal mucosa

While the school nurse is teaching a group of 14-year-olds, one of the participants remarks, "You are too young to be our teacher! You're not much older than we are!" How should the nurse respond? A. "I think I am qualified to teach this group." B. "How old do you think I am?" C. "Do you think you can teach it any better?" D. "We need to stay focused on the topic."

D. "We need to stay focused on the topic."

To reduce the risk of being named in malpractice lawsuit, which action is most important for the nurse to take? A. Establish a trusting nurse-client relationship. B. Complete an incident report following a client injury. C. Maintain current professional malpractice insurance, D. Adhere consistently to standards of care.

D. Adhere consistently to standards of care.

The nurse is caring for a client with acute kidney injury (AKI) secondary to gentamicin therapy the client's serum blood potassium is elevated, which finding requires immediate action by the nurse? A. Tall peak T waves on the cardiac monitor B. Peripheral pitting edema at 2 + indentation C. Serum creatinine above 0.5 mg/dl or 44.2 micro-mmol/dl D. Anuria for the last 12 hours.

D. Anuria for the last 12 hours.

A confused, older client with Alzheimer's disease becomes incontinent of urine when attempting to find the bathroom. Which action should the nurse implement? A. Instruct the client to use the call button when a bedpan is needed B. Apply adult diapers after each attempt to void C. Check residual urine volume using an indwelling urinary catheter D. Assist the client's to a bedside commode every two hours

D. Assist the client's to a bedside commode every two hours

A female client is admitted for diabetic crisis resulting from inadequate dietary practices. After stabilization, the nurse talks to the client about her prescribed diet. What client characteristic is most import for successful adherence to the diabetic diet? A. Knows that insulin must be given 30 min before eating B. Frequently eats fruits and vegetables at meals and between meals/ C. Has someone available who can prepare and oversee the diet D. Demonstrates willingness to adhere to the diet consistently

D. Demonstrates willingness to adhere to the diet consistently

The nurse and an unlicensed assistive personnel (UAP) are providing care for a client with a nasogastric tube (NGT) when the client begins to vomit. How should the nurse manage this situation? A. Determine the presence of hematemesis as the UAP irrigates the NGT B. Instruct the UAP to bring an antiemetic to the nurse at the bedside C. Assess the appearance of the emesis while the UAP checks bowel sounds D. Direct the UAP to measure the emesis while the nurse irrigates the NGT

D. Direct the UAP to measure the emesis while the nurse irrigates the NGT

The charge nurse observes a new nurse preparing to insert an intravenous (IV) catheter. The new nurse has gathered supplies, including intravenous catheters, an intravenous insertion kit, and a 4x4 sterile gauze dressing to cover and secure the insertion site. What action should the charge nurse take? A. Plan to observe the secured IV site after the insertion procedure B. Confirm that the nurse has gathered the necessary supplies C. Remind the nurse to tape the gauze dressing securely in place D. Instruct the nurse to use a transparent dressing over the site

D. Instruct the nurse to use a transparent dressing over the site

A client who has a suspected brain tumor is schedules for a computed (CT) scan. When preparing the client for the client for the CT scan, which intervention should the nurse implement? A. Determine if the client has had a knee or hip replacement B. Immobilize the client's neck before moving onto stretcher C. Give an antiemetic to control nausea D. Obtain the client's food allergy history

D. Obtain the client's food allergy history

A client with a liver abscess develops septic shock. A sepsis resuscitation bundle protocol is initiated and the client receives a bolus of IV fluids. Which parameter should the nurse monitor to assess effectiveness of the fluid bolus? A. Mean arterial pressure (MAP) B. White blood cell count C. Blood culture D. Oxygen saturation

D. Oxygen saturation

An 11-year-old client is admitted to the mental health unit after trying to run away from home and threatening self-harm. The nurse establishes a goal to promote effective coping, and plans to ask the client to verbalize three ways to deal with stress. Which activity is best to establish rapport and accomplish this therapeutic goal? A. Bring the client to the team meeting to discuss the treatment plan B. Ask the client to write feeling in a journal and then review it together C. Explain the purpose of each medication the client is currently taking D. Play a board game with the client and begin taking about stressors

D. Play a board game with the client and begin taking about stressors

The nurse is preparing to gavage feed a premature infant through an orogastric tube. During insertion of the tube, the infant's heart rate drops to 60 beats / minute. Which action should the nurse take? A. Continue the insertion since this is a typical response B. pause and monitor for a continues drop of the heart rate C. Insert the feeding tube into the infant's nasal passage D. Postpone the feeding until the infant's vital signs and stable

D. Postpone the feeding until the infant's vital signs and stable

An unlicensed assistive personnel (UAP) informs the nurse who is giving medications that a female client is crying. The client was just informed that she has a malignant tumor. What action should the nurse implement first? A. Provide the client with a PRN antianxiety medication and allow privacy for her to grieve. B. Instruct the UAP to notify the client's spiritual advisor of her need for counseling. C. Ask another nurse to finish giving medications and attend to the client immediately. D. Tell the client that the nurse will be back to talk to her after medications are given.

D. Tell the client that the nurse will be back to talk to her after medications are given

An adult male who fell from a roof and fractures his left femur is admitted for surgical stabilization after having a soft cast applied in the emergency department. Which assessment finding warrants immediate intervention by the nurse? A. Onset of mild confusion B. Pain score 8 out of 10 C. Pale, diaphoretic skin D. Weak palpable distal pulses

D. Weak palpable distal pulses

When assessing a multigravida the first postpartum day, the nurse finds a moderate amount of lochia rubra, with the uterus firm, and three fingerbreadths above the umbilicus. What action should the nurse implement first A.)Massage the uterus to decrease atony B.)Review the hemoglobin to determine hemorrhage C.)Increase intravenous infusion D.)Check for a distended bladder

D.)Check for a distended bladder

Which assessment is more important for the nurse to include in the daily plan of care for a client with a burned extremity?

Distal pulse intensity

While attempting to stablish risk reduction strategies in a community, the nurse notes that the regional studies have indicated....persons with irreversible mental deficiencies due to hypothyroidism. The nurse should seek funding to implement which screening measure?

T4 levels in newborns

A client presents to the labor and delivery unit, screaming "THE BABY IS COMING" which action should the nurse implement first.

Observe the perineum

The husband of a client with advanced ovarian cancer wants his wife to have every treatment available. When the husband leaves, the client tells the nurse that she has had enough chemotherapy and wants to stop all treatments but knows her husband will sign the consent form for more treatment. The nurse's response should include which information? a- The husband cannot sign the consent for the client, her signature is required b- The client's specific wishes should be discussed with her healthcare provider c- Counseling should be sought to resolve the husband's desire to control his wife d- The healthcare team will formulate a plan of care to keep the client comfortable e- The client should seek a second medical opinion before deciding to stop treatment.

a- The husband cannot sign the consent for the client, her signature is required b- The client's specific wishes should be discussed with her healthcare provider d- The healthcare team will formulate a plan of care to keep the client comfortable

A male client is discharged from the intensive care unit following a myocardial infarction, and the healthcare provider low-sodium diet. Which lunch selection indicates to the nurse that this client understands the dietary restrictions? a- Turkey salad sandwich. b- Clam chowder c- Macaroni and cheese d- Bacon, lettuce, and tomato sandwich

a- Turkey salad sandwich.

A woman who takes pyridostigmine for myasthenia gravis (MG) arrives at the emergency department complaining of extreme muscle weakness. Her adult daughter tells the nurse that since yesterday her mother has been unable to smile, which assessment finding warrants immediate intervention by the nurse? a- Uncontrollable drooling b- Inability to raise voice c- Tingling of extremities d- Eyelid drooling

a- Uncontrollable drooling

A client with HIV and pulmonary coccidioidomycosis is receiving amphotericin B. which assessment finding should the nurse report to the healthcare provider? a- Urinary output of 25mL per hour b- Hemoglobin level of 10 g/dL or 100 g/L (S1) c- Hyperactive bowel sounds d- Oral temperature of 100.4 F (38 C)

a- Urinary output of 25mL per hour

In conducting a health assessment, the nurse determines that both parents of a child with asthma smoke cigarettes. What recommendation is best to the nurse to recommend to the parents? a- avoid smoking in the house b- stop smoking immediately c- decrease the number of cigarettes smoke daily d- obtain nicotine patches to assist in smoking sensation

a- avoid smoking in the house

A client with a serum sodium level of 125 meq/mL should benefit most from the administration of which intravenous solution? a. 0.9% sodium chloride solution (normal saline) b. 0.45% sodium chloride solution (half normal saline) c. 10% Dextrose in 0.45% sodium chloride d. 5% dextrose in 0.2% sodium chloride

a. 0.9% sodium chloride solution (normal saline)

Which location should the nurse choose as the best for beginning a screening program for hypothyroidism? a. A business and professional women's group. b. An African-American senior citizens center c. A daycare center in a Hispanic neighborhood d. An after-school center for Native-American teens

a. A business and professional women's group.

When organizing home visits for the day, which older client should the home health nurse plan to visit first? a. A woman who takes naproxen (Naprosyn) and reports a recent onset of dark, tarry stools. b. A man who receives weekly injections of epoetin (Procrit) for a low serum iron level c. A man with emphysema who smokes and is complaining of white patches in his mouth d. A frail woman with heart failure who reported a 2 pounds' weight gain in the last week.

a. A woman who takes naproxen (Naprosyn) and reports a recent onset of dark, tarry stools

A client is receiving a full strength continuous enteral tube feeding at 50 ml/hour and has developed diarrhea. The client has a new prescription to change the feeding to half strength. What intervention should the nurse implement? a. Add equal amounts of water and feeding to a feeding bag and infuse at 50ml/hour b. Continue the full strength feeding after decreasing the rate of infusion to 25 ml/hr. c. Maintain the present feeding until diarrhea subsides and the begin the next new prescription. d. Withhold any further feeding until clarifying the prescription with healthcare provides.

a. Add equal amounts of water and feeding to a feeding bag and infuse at 50ml/hour

A primigravida client is 36 weeks gestation is admitted to labor and delivery unit because her membranes ruptured 30minutes ago. Initial assessment indicates 2cm dilation, 50% effaced, -2 station, vertex presentation greenish colored amniotic fluid, and contractions occurring 3-5 minutes with a low FHR after the last contraction peaks: a. Administer Oxygen via face mask b. Apply an internal fetal heart monitor c. Notify the healthcare provider d. Use a vibroacoustic stimulator

a. Administer Oxygen via face mask

An older adult resident of a long-term care facility has a 5-year history of hypertension. The client has a headache and rate the pain 5 on a pain scale 0 to 10. The client's blood pressure is currently 142/89. Which interventions should the nurse implement? (Select all that apply) a. Administer a daily dose of lisinopril as scheduled. b. Assess the client for postural hypotension. c. Notify the healthcare provider immediately d. Provide a PRN dose of acetaminophen for headache e. Withhold the next scheduled daily dose of warfarin.

a. Administer a daily dose of lisinopril as scheduled. d. Provide a PRN dose of acetaminophen for headache

The nurse caring for a 3-month-old boy one day after a pylorotomy notices that the infant is restless, is exhibiting facial grimaces, and is drawing his knees to his chest. What action should the nurse take? a. Administer a prescribed analgesia for pain. b. Increase IV infusion rate for rehydration c. Provide additional blankets to increase body temperature d. Feed one ounce of formula to correct hypoglycemia.

a. Administer a prescribed analgesia for pain.

A client diagnosed with calcium kidney stones has a history of gout. A new prescription for aluminum hydroxide (Amphogel) is scheduled to begin at 0730. Which client medication should the nurse bring to the healthcare provider's attention? a. Allopurinol (Zyloprim) b. Aspirin, low dose c. Furosemide (lasix) d. Enalapril (vasote)

a. Allopurinol (Zyloprim)

The nurse observes a newly hired unlicensed assistive personnel (UAP) performing a fingestick to obtain a client's blood glucose. Prior to sticking the client's finger, the UAP explains the procedure and tell the client that it I painless. What action should the nurse take? a. Allow the UAP to complete the procedure, then discuss the painless comment privately with the UAP. b. Stop the UAP before the procedure and explain to the client that some discomfort may be felt c. Interject that while the procedure is not extremely painful, the client will feel a prick on the finger. d. Report the incident to the education director and request additional instruction for the UAP.

a. Allow the UAP to complete the procedure, then discuss the painless comment privately with the UAP.

A young adult who is hit with a baseball bat on the temporal area of the left skull is conscious when admitted to the ED and is transferred to the Neurological Unit to be monitored for signs of closed head injury. Which assessment finding is indicative of a developing epidural hematoma? a. Altered consciousness within the first 24 hours after injury. b. Cushing reflex and cerebral edema after 24 hours c. Fever, nuchal rigidity and opisthotonos within hours d. Headache and pupillary changes 48 hours after a head injury

a. Altered consciousness within the first 24 hours after injury.

Which client is at the greatest risk for developing delirium? a. An adult client who cannot sleep due to constant pain. b. an older client who attempted 1 month ago c. a young adult who takes antipsychotic medications twice a day d. a middle-aged woman who uses a tank for supplemental oxygen

a. An adult client who cannot sleep due to constant pain.

In making client care assignment, which client is best to assign to the practical nurse (PN) working on the unit with the nurse? a. An immobile client receiving low molecular weight heparin q12 h. b. A client who is receiving a continuous infusion of heparin and gets out of bed BID c. A client who is being titrated off heparin infusion and started on PO warfarin (Coumadin) d. An ambulatory client receiving warfarin (Coumadin) with INR of 5 second.

a. An immobile client receiving low molecular weight heparin q12 h.

The nurse assesses a client with new onset diarrhea. It is most important for the nurse to question the client about recent use of which type of medication? a. Antibiotics b. Anticoagulants c. Antihypertensive d. Anticholinergics

a. Antibiotics

A male client with cancer is admired to the oncology unit and tells the nurse that he is in the hospital for palliative care measures. The nurse notes that the client's admission prescription include radiation therapy. What action should the nurse implement? a. Ask the client about his expected goals for the hospitalization b. Explain the palliative care measures can be provided at home c. Notify do radiation department to withhold the treatment for now d. Determine if the client wishes to cancel further radiation treatment

a. Ask the client about his expected goals for the hospitalization

A young adult male who is being seen at the employee health care clinic for an annual assessment tell the nurse that his mother was diagnosed with schizophrenia when she was his age and that life with a schizophrenic mother was difficulty indeed. Which response is best for the nurse to provide? a. Ask the client if he is worried about becoming schizophrenic at the age his mother was diagnosed. b. Encourage the client to seek genetic counseling to determine his risk for mental illness. c. Informed the client that his mother schizophrenic has affected his psychological development. d. Tell the client that mental illness has a familial predisposition, so he should see a psychiatrist.

a. Ask the client if he is worried about becoming schizophrenic at the age his mother was diagnosed.

While the nurse is conducting a daily assessment of an older woman who resides in a long-term facility, the client begins to cry and tells the nurse that her family has stopped calling and visiting. What action should the nurse take first? a. Ask the client when a family member last visited her. b. Determine the client's orientation to time and space c. Review the client's record regarding social interactions d. Reassure the client of her family's love for her

a. Ask the client when a family member last visited her.

A preschool-aged boy is admitted to the pediatric unit following successful resuscitation from a near-drowning incident. While providing care to child, the nurse begins talking with his preadolescent brother who rescued the child from the swimming pool and initiated resuscitation. The nurse notices the older boy becomes withdrawn when asked about what happened. What action should the nurse take a. Ask the older brother how he felt during the incident. b. Commend the older brother for his heroic actions c. Tell the older brother that he seems depressed d. Develop a water safety teaching plan for the family.

a. Ask the older brother how he felt during the incident.

An adult client with severe depression was admitted to the psychiatric unit yesterday evening. Although the client ran one year ago, his spouse states that the client no longer runs, bur sits and watches television most of the day. Which is most important for the nurse to include in this client's plan of care for today? a. Assist client in identifying goals for the day. b. Encourage client to participate for one hour in a team sport. c. Schedule client for a group that focuses on self-esteem. d. Help client to develop a list of daily affirmations.

a. Assist client in identifying goals for the day.

A client who had an open cholecystectomy two weeks ago comes to the emergency department with complaints of nausea, abdominal distention, and pain. Which assessment should the nurse implement? a. Auscultate all quadrant of the abdomen. b. Perform a digital rectal exam c. Palpate the liver and spleen d. Obtain a hemoccult of the client's stool

a. Auscultate all quadrant of the abdomen.

The nurse notes that a client has been receiving hydromorphone (Dilaudid) every six hours for four days. What assessment is most important for the nurse to complete? a. Auscultate the client's bowel sounds b. Observe for edema around the ankles c. Measure the client's capillary glucose level d. Count the apical and radial pulses simultaneously

a. Auscultate the client's bowel sounds

Which instruction is most important for the nurse to provide a client who is being discharge following treatment for Guillain-Barre syndrome? a. Avoid exposure to respiratory infections. b. Use relaxation exercise when anxious c. Continue physical therapy at home d. Plan short, frequent rest periods.

a. Avoid exposure to respiratory infections.

The home care nurse provide self-care instruction for a client chronic venous insufficiency cause by deep vein thrombosis. Which instructions should the nurse include in the client's discharge teaching plan? Select all that apply a. Avoid prolonged standing or sitting b. Use recliner for long period of sitting c. continue wearing elastic stocking d. Maintain the bed flat while sleeping e. Cross legs at knee but not at ankle

a. Avoid prolonged standing or sitting b. Use recliner for long period of sitting c. continue wearing elastic stocking

An adult male reports the last time he received penicillin he developed a severe maculopapular rash all over his chest. What information should the nurse provide the client about future antibiotic prescriptions? a. Be alert for possible cross-sensitivity to cephalosporin agents. b. Monitor peak ad trough levels whenever taking any antibiotic c. Watch daily urine output and weight gain while taking antibiotics d. Wear sun block and protective clothing to avoid exposure to sun.

a. Be alert for possible cross-sensitivity to cephalosporin agents.

A client has a prescription for lorazepam 2mg for alcohol withdrawal symptoms. Which finding... the client? a. Blood pressure 149/101 b. Irregular pulse rate of 80 c. Oral temperature is 98.9 F (37.1 C) d. Pain rated 7 on scale 1-10

a. Blood pressure 149/101

The mother of a child with cerebral palsy (CP) ask the nurse if her child's impaired movements will worsen as the child grows. Which response provides the best explanation? a. Brain damage with CP is not progressive but does have a variable course b. CP is one of the most common permanent physical disability in children c. Severe motor dysfunction determines the extent of successful habilitation d. Continued development of the brain lesion determines the child's outcome.

a. Brain damage with CP is not progressive but does have a variable course

Which statement is accurate regarding the pathological changes in the pulmonary system associated with acute (adult) respiratory distress syndrome (ARDS)? a. Capillary hydrostatic pressure exceeds colloid osmotic pressure, producing interstitial edema b. A high ventilation-to-perfusion ratio is characteristic of affected lung fields in ARDS c. Functional residual capacity and lung compliance increase as the disease progresses d. Interstitial edema that occurs due to capillary fluid shifts is usually more serious than alveolar edema.

a. Capillary hydrostatic pressure exceeds colloid osmotic pressure, producing interstitial edema

A child with heart failure is receiving the diuretic furosemide (Lasix) and has serum potassium level 3.0 mEq/L. Which assessment is most important for the nurse to obtain? a. Cardiac rhythm and heart rate. b. Daily intake of foods rich in potassium. c. Hourly urinary output d. Thirst ad skin turgor.

a. Cardiac rhythm and heart rate.

The nurse is administering a 750 ml cleansing enema to an adult client. After approximately150 ml of enema has informed, the client states, 'stop I can't hold anymore." What action should the nurse take? a. Clamp the tubing and instruct the client to breathe deeply before continuing. b. Discontinue infusing the enema and record the client's response. c. Slow infusion of the enema and instruct the client to use paint breathing d. Place the client on the bedpan and continue infusion of the enema.

a. Clamp the tubing and instruct the client to breathe deeply before continuing.

A mother brings her 6-year-old child, who has just stepped on a rusty nail, to the pediatrician's office. Upon inspection, the nurse notes that the nail went through the shoe and pierced the bottom of the child's foot. Which action should the nurse implement first? a. Cleanse the foot with soap and water and apply an antibiotic ointment b. Provide teaching about the need for a tetanus booster within the next 72 hours. c. have the mother check the child's temperature q4h for the next 24 hours d. transfer the child to the emergency department to receive a gamma globulin injection

a. Cleanse the foot with soap and water and apply an antibiotic ointment

A male client is admitted for the removal of an internal fixation that was inserted for the fracture ankle. During the admission history, he tells the nurse he recently received vancomycin (vancomycin) for a methicillin-resistant Staphylococcus aureus (MRSA) wound infection. Which action should the nurse take? (Select all that apply.) a. Collect multiple site screening culture for MRSA b. Call healthcare provider for a prescription for linezolid (Zyrovix) c. Place the client on contact transmission precautions d. Obtain sputum specimen for culture and sensitivity e. Continue to monitor for client sign of infection.

a. Collect multiple site screening culture for MRSA c. Place the client on contact transmission precautions e. Continue to monitor for client sign of infection.

A client is admitted with acute pancreatitis. The client admits to drinking a pint of bourbon daily. The nurse medicates the client for pain and monitors vital signs q2 hours. Which finding should the nurse report immediately to the healthcare provider? a. Confusion and tremors b. Yellowing and itching of skin. c. Abdominal pain and vomiting d. Anorexia and abdominal distention

a. Confusion and tremors

A 6-year-old child with acute infectious diarrhea is placed on a rehydration therapy... Which action should the nurse instruct the parents to take if the child begins to vomit? a. Continue giving ORS frequently in small amounts b. Withhold all oral intake c. Supplement ORS with gelatin or chicken broth d. Provide only bottle water.

a. Continue giving ORS frequently in small amounts

A primigravida a 40-weeks gestation with preeclampsia is admitted after having a seizure in the hot tub at a midwife's birthing center. Based on documentation in the medical record, which action should the nurse implement? (Click on each chart tab for additional information. Please be sure to scroll to the bottom right corner of each tab to view all information contained in the client's medical record.) a. Continue to monitor the client's blood pressure hourly. b. Inform the healthcare provider of CBC results c. Update the nursery staff on the client's status d. Give a dose of calcium gluconate per preeclampsia protocol.

a. Continue to monitor the client's blood pressure hourly.

A client is receiving oxytocin (Pitocin) to augment early labor. Which assessment is most important time the infusion rate is increases? a. Contraction pattern b. Blood pressure c. Infusion site d. Pain level

a. Contraction pattern

The nurse is caring for a client who is entering the second stage of labor. Which action should the nurse implement first? a. Convey to the client that birth is imminent. b. Prepare the client for spinal anesthesia c. Empty the client's bladder using a straight catheter d. Prepare the coach to accompany the client to delivery

a. Convey to the client that birth is imminent.

An elderly male client is admitted to the mental health unit with a sudden onset of global disorientation and is continuously conversing with his mother, who died 50 years ago. The nurse reviews the multiple prescriptions he is currently taking and assesses his urine specimen, which is cloudy, dark yellow, and has foul odor. These findings suggest that his client is experiencing which condition? a. Delirium b. Depression c. Dementia d. Psychotic episode

a. Delirium

An adolescent with major depressive disorder has been taking duloxetine (Cymbalta) for the past 12 days. Which assessment finding requires immediate follow-up a. Describes life without purpose b. Complains of nausea and loss of appetite c. States is often fatigued and drowsy d. Exhibits an increase in sweating.

a. Describes life without purpose

To prevent infection by auto contamination during the acute phase of recovery from multiple burns, which intervention is most important for the nurse to implement? a. Dress each wound separately. b. Avoid sharing equipment between multiple clients. c. Use gown, mask and gloves with dressing change. d. Implement protective isolation.

a. Dress each wound separately.

A client is being discharged with a prescription for warfarin (Coumadin). What instruction should the nurse provide this client regarding diet? a. Eat approximated the same amount of leafy green vegetables daily so the amount of vitamin K consumed is consistent. b. Avoid eating all foods that contain any vitamin K because it is an antagonist of Coumadin. c. Increase the intake of dark green leafy vegetables while taking Coumadin d. Eat two servings of raw dark green leafy vegetables daily and continue for 30 days after Coumadin therapy is completed.

a. Eat approximated the same amount of leafy green vegetables daily so the amount of vitamin K consumed is consistent.

A male client with cancer, who is receiving antineoplastic drugs, is admitted to the... what findings is most often manifest this condition? a. Ecchymosis and hematemesis b. Weight loss and alopecia c. Weakness and activity intolerance d. Sore throat and fever

a. Ecchymosis and hematemesis

Before leaving the room of a confused client, the nurse notes that a half bow knot was used to attach the client's wrist restraints to the movable portion of the client's bed frame. What action should the nurse take before leaving the room? a. Ensure that the knot can be quickly released. b. Tie the knot with a double turn or square knot. c. Move the ties so the restraints are secured to the side rails. d. Ensure that the restraints are snug against the client's wrist.

a. Ensure that the knot can be quickly released.

A client with type 2 diabetes mellitus is admitted for frequent hyperglycemic episodes and a glycosylated hemoglobin (HbA1c) of 10%. Insulin glargine 10 units subcutaneously once a day at bedtime and a sliding scale with insulin aspart q6h are prescribed. What action should the nurse include in this client's plan of care? a. Fingerstick glucose assessment q6h with meals b. Mix bedtime dose of insulin glargine with insulin aspart sliding scale dose c. Review with the client proper foot care and prevention of injury d. Do not contaminate the insulin aspart so that it is available for iv use e. Coordinate carbohydrate controlled meals at consistent times and intervals f. Teach subcutaneous injection technique, site rotation and insulin management

a. Fingerstick glucose assessment q6h with meals c. Review with the client proper foot care and prevention of injury e. Coordinate carbohydrate controlled meals at consistent times and intervals f. Teach subcutaneous injection technique, site rotation and insulin management

A 60-year-old female client with a positive family history of ovarian cancer has developed an abdominal mass and is being evaluated for possible ovarian cancer. Her Papanicolau (Pap) smear results are negative. What information should the nurse include in the client's teaching plan a. Further evaluation involving surgery may be needed b. A pelvic exam is also needed before cancer is ruled out c. Pap smear evaluation should be continued every six month d. One additional negative pap smear in six months is needed.

a. Further evaluation involving surgery may be needed

A neonate with a congenital heart defect (CHD) is demonstrating symptoms of heart failure (HF). Which interventions should the nurse include in the infant's plan of care? a. Give O2 at 6 L/nasal canula for 3 repeated oximetry screens below 90% b. Administer diuretics via secondary infusion in the morning only c. Evaluate heart rate for effectiveness of cardio tonic medications d. Use high energy formula 30 calories/ounce at Q3 hours feeding via soft nipples e. Ensure Interrupted and frequent rest periods between procedures.

a. Give O2 at 6 L/nasal canula for 3 repeated oximetry screens below 90% c. Evaluate heart rate for effectiveness of cardio tonic medications d. Use high energy formula 30 calories/ounce at Q3 hours feeding via soft nipples e. Ensure Interrupted and frequent rest periods between procedures.

The nurse is auscultating a client's lung sounds. Which description should the nurse use to document this sound? a. High pitched or fine crackles. b. Rhonchi c. High pitched wheeze d. Stridor

a. High pitched or fine crackles.

The nurse is developing the plan of care for a client with pneumonia and includes the nursing diagnosis of "Ineffective airway clearance related to thick pulmonary secretions." Which intervention is most important for the nurse to include in the client's plan of care? a. Increase fluid intake to 3,000 ml/daily b. Administer O2 at 5L/mint per nasal cannula c. Maintain the client in a semi Fowler's position d. Provide frequent rest period.

a. Increase fluid intake to 3,000 ml/daily

In planning strategies to reduce a client's risk for complications following orthopedic surgery, the nurse recognizes which pathology as the underlying cause of osteomyelitis? a. Infectious process b. Metastatic process c. Autoimmune disorder d. Inflammatory disorder

a. Infectious process

After placement of a left subclavian central venous catheter (CVC), the nurse receives report of the x-ray findings that indicate the CVC tip is in the client's superior vena cava. Which action should the nurse implement? a. Initiate intravenous fluid as prescribed b. Notify the HCP of the need to reposition the catheter c. Remove the catheter and apply direct pressure for 5 minutes. d. Secure the catheter using aseptic technique

a. Initiate intravenous fluid as prescribed

The nurse is preparing a client who had a below-the-knee (BKA) amputation for discharge to home. Which recommendations should the nurse provide this client? (Select all that apply) a. Inspect skin for redness b. Use a residual limb shrinker c. Apply alcohol to the stump after bathing d. Wash the stump with soap and water e. Avoid range of motion exercises

a. Inspect skin for redness b. Use a residual limb shrinker d. Wash the stump with soap and water

The health care provider prescribes atenolol 50 mg daily for a client with angina pectoris...to the health care provider before administering this medication? a. Irregular pulse b. Tachycardia c. Chest pain d. Urinary frequency

a. Irregular pulse

A client is admitted with metastatic carcinoma of the liver, ascites, and bilateral 4+ pitting edema of both lower extremities. When the client complains that the antiembolic stocking are too constricting, which intervention should the nurse implement? a. Maintain both lower extremities elevated on pillows. b. Remove the contracting antiembolic stocking c. Administer diuretics in the morning hours d. Restrict PO fluid intake to 500 ml per shift

a. Maintain both lower extremities elevated on pillows.

A client who had a small bowel resection acquired methicillin resistant staphylococcus aureus (MRSA) while hospitalized. He treated and released, but is readmitted today because of diarrhea and dehydration. It is most important for the nurse to implement which intervention. a. Maintain contact transmission precaution b. Review white blood cell (WBC) count daily c. Instruct visitors to gown and wash hands d. Collect serial stool specimens for culture

a. Maintain contact transmission precaution

A female client has been taking a high dose of prednisone, a corticosteroid, for several months. After stopping the medication abruptly, the client reports feeling "very tired". Which nursing intervention is most important for the nurse to implement? a. Measure vital signs b. Auscultate breath sounds c. Palpate the abdomen d. Observe the skin for bruising

a. Measure vital signs

A client with cirrhosis of the liver is admitted with complications related to end stage liver disease. Which intervention should the nurse implement? (Select all that apply.) a. Monitor abdominal girth. b. Increase oral fluid intake to 1500 ml daily. c. Report serum albumin and globulin levels. d. Provide diet low in phosphorous. e. Note signs of swelling and edema.

a. Monitor abdominal girth. c. Report serum albumin and globulin levels. e. Note signs of swelling and edema.

Two days after admission a male client remembers that he is allergic to eggs, and informs the nurse of the allergy. Which actions should the nurse implement? (Select all that apply) a. Notify the food services department of the allergy. b. Enter the allergy information in the client's record. c. Document the statement in the nurse's notes d. Note the allergy on the diet intake flow sheet e. Add egg allergy to the client's allergy arm band.

a. Notify the food services department of the allergy. b. Enter the allergy information in the client's record. e. Add egg allergy to the client's allergy arm band.

To evaluate the effectiveness of male client's new prescription for ezetimibe, which action should the clinic nurse implement? a. Remind the client to keep his appointments to have his cholesterol level checked. b. Teach the client to weigh himself weekly and keep a log of the measurements c. Assess the elasticity of the client's skin at the next scheduled clinic appointment d. Encourage the client to keep a diary of his food intake until his next visit to the clinic.

a. Remind the client to keep his appointments to have his cholesterol level checked.

The nursing staff on a medical unit includes a registered nurse (RN), practical nurse (PN), and an unlicensed assistive personnel (UAP). Which task should the charge nurse assign to the RN? a. Supervise a newly hired graduate nurse during an admission assessment. b. Transport a client who is receiving IV fluids to the radiology department. c. Administer PRN oral analgesics to a client with a history of chronic pain d. Complete ongoing focused assessments of a client with wrist restrain.

a. Supervise a newly hired graduate nurse during an admission assessment.

The nurse is assisting a new mother with infant feeding. Which information should the nurse provide that is most likely to result in a decrease milk supply for the mother who is breastfeeding? a. Supplemental feedings with formula b. Maternal diet high in protein c. Maternal intake of increased oral fluid d. Breastfeeding every 2 or 3 hours.

a. Supplemental feedings with formula

A client with a large pleural effusion undergoes a thoracentesis. Following the procedure, which assessment finding warrants immediate intervention by the nurse? a. The client has asymmetrical chest wall expansion b. The clients complain of pain at the insertion site c. The client chest's x-ray indicates decreased pleural effusion d. The client's arterial blood gases are pH 7.35, PaO2 85, Pa CO2 35, HCO3 26

a. The client has asymmetrical chest wall expansion

A client in septic shock has a double lumen central venous catheter with one liter of 0.9% Normal Saline Solution infusing at 1 ml/hour through one lumen and TPN infusing at 50 ml/ hr. through one port. The nurse prepared newly prescribed IV antibiotic that should take 45 mints to infuse. What intervention should the nurse implement? a. Use a secondary port of the Normal Saline solution to administer the antibiotic. b. Add the antibiotic to the TPN solution, and continue the normal saline solution. c. Stop the TPN infusion for the time needed to administer the prescribed antibiotic. d. Add the antibiotic to the Normal Saline solution and continue both infusions.

a. Use a secondary port of the Normal Saline solution to administer the antibiotic.

A client is discharged with automated peritoneal dialysis (PD) to be used nightly...which instructions should the nurse include? a. Wash hands before cleaning exit site b. Keep the head of the bed flat at night c. Feel for a thrill and a distal pulse nightly d. Do not get up if fluid is left in the abdomen

a. Wash hands before cleaning exit site

A client is being discharged home after being treated for heart failure (HF). What instruction should the nurse include in this client's discharge teaching plan? a. Weigh every morning b. Eat a high protein diet c. Perform range of motion exercises d. Limit fluid intake to 1,500 ml daily

a. Weigh every morning

When planning care for a client with acute pancreatitis, which nursing intervention has the highest priority? a. Withhold food and fluid intake. b. Initiate IV fluid replacement. c. Administer antiemetic as needed. d. Evaluate intake and output ratio.

a. Withhold food and fluid intake.

An older male who is admitted for end stage of chronic obstructive pulmonary disease (COPD) tells the nurse .... The client provides the nurse with a living will and DNR. What action should the nurse implement? a- Inform the family of the client whishes b- Obtain a prescription for DNR c- Clergy consultation d- Ask the patient why he made this choice

b- Obtain a prescription for DNR

The charge nurse observes the practical nurse (PN) apply sterile gloves in preparation for performing a sterile dressing change. Which action by the PN requires correction by the charge nurse? a- Opening the package b- Picking up the second glove c- Picking up the first glove d- Positioning of the table

b- Picking up the second glove

The nurse delegates to an unlicensed assistive personnel (UAP) denture care for a client with...daily leaving. When making this assignment, which instruction is most important for the nurse to do? a- Do not remove the dentures, but instead brush them within the mouth b- Place a washcloth in the sink while cleaning the dentures. c- Use tepid, not hot, water to clean the dentures d- Avoid damaging the dentures using a soft-bristled toothbrush.

b- Place a washcloth in the sink while cleaning the dentures.

A health care provider continuously dismisses the nursing care suggestions made by staff nurses. As a result...dealing with the healthcare provider. What action should the nurse manager implement? a- Confront the health care provider about the perceived lack of respect for the staff nurses. b- Plan an interdisciplinary staff meeting to develop strategies to enhance client care c- Request an investigation about the perceived incivility of the healthcare provider interaction. d- Remind the staff that avoidance behavior is not a professional way to handle the problem.

b- Plan an interdisciplinary staff meeting to develop strategies to enhance client care

A 3-year-old boy with a congenital heart defect is brought to the clinic by his mother... During the assessment, the mother asks the nurse why her child is at the 5th percent...response is best for the nurse to provide? a. Does your child seem mentally slower than his peers also? b. "His smaller size is probably due to the heart disease" c. Haven't you been feeding him according to recommended daily allowances for children? d. You should not worry about the growth tables. They are only averages for children

b. "His smaller size is probably due to the heart disease"

The nurse is conducting health assessments. Which assessment finding increases a 56 year-old woman's risk for developing osteoporosis? a. Body mass index of (BMI) of 31 b. 20 pack-year history of cigarette smoking c. Birth control pill usage until age 45 d. Diabetes mellitus in family history

b. 20 pack-year history of cigarette smoking

What is the primary goal when planning nursing care for a client with degenerative joint disease (DJD)? a. Obtain adequate rest and sleep b. Achieve satisfactory pain control. c. Improve stress management skills d. Reduce risk for infection.

b. Achieve satisfactory pain control.

After checking the fingerstick glucose at 1630, what action should the nurse implement? a. Notify the healthcare provider b. Administer 8 units of insulin aspart SubQ c. Gives an IV bolus of Dextrose 50% 50 ml d. Perform quality control on the glucometer.

b. Administer 8 units of insulin aspart SubQ

After receiving the Braden scale findings of residents at a long-term facility, the charge nurse should to tell the unlicensed assistive personnel (UAP) to prioritize the skin care for which client? a. An older adult who is unable to communicate elimination needs. b. An older man whose sheets are damped each time he is turned. c. A woman with osteoporosis who is unable to bear weight. d. A poorly nourished client who requires liquid supplement. An older man whose sheets are damped each time he is turned.

b. An older man whose sheets are damped each time he is turned.

At the end of a preoperative teaching session on pain management techniques, a client starts to cry and states, "I just know I can't handle all the pain." What is the priority nursing diagnosis for this client? a. Knowledge deficit b. Anxiety c. Anticipatory grieving d. Pain (acute)

b. Anxiety

A multigravida, full-term, laboring client complains of "back labor". Vaginal examination reveals that the client's 3 cm with 50% effacement and the fetal head is at -1 station. What should the nurse implement? a. Turn the client to a lateral position b. Apply counter-pressure to the sacral area c. Notify the scrub nurse to prepare the OR d. Ambulate the client between contractions

b. Apply counter-pressure to the sacral area

A new member joins the nursing team spreads books on the table, puts items on two chairs, and sits on a third chair. The members of the group are forced to move closer and remove their possessions from the table what action should the nurse leader take? a. Move to welcome and accommodate a new person b. Ask the new person to move belonging to accommodate others c. Tell the new person to move belongings because of limited space d. Bring in additional chairs so that all staff members can be seated

b. Ask the new person to move belonging to accommodate others

Which intervention should the nurse implement during the administration of vesicant chemotherapeutic agent via an IV site in the client's arm? a. Explain the temporary burning of the IV site may occur. b. Assess IV site frequently for signs of extravasation c. Apply a topical anesthetic of the infusion site for burning d. Monitor capillary refill distal to the infusion site.

b. Assess IV site frequently for signs of extravasation

The nurse should teach the client to observe which precaution while taking dronedarone? a. Stay out of direct sunlight b. Avoid grapefruits and its juice c. Reduce the use of herbal supplements d. Minimize sodium intake.

b. Avoid grapefruits and its juice

The nurse is preparing an older client for discharge following cataract extraction. Which instruction should be include in the discharge teaching? a. Do not read without direct lighting for 6 weeks. b. Avoid straining at stool, bending, or lifting heavy objects. c. Irrigate conjunctiva with ophthalmic saline prior to installing antibiotic ointment. d. Limit exposure to sunlight during the first 2 weeks when the cornea is healing.

b. Avoid straining at stool, bending, or lifting heavy objects.

The nurse assesses a female client with obstructive sleep apnea syndrome (OSAS) who is 5 feet tall (152 cm) and weighs 155 pounds (70 kg), the client's 24 hour diet history includes: no breakfast, cheeseburger and fries for lunch; lasagna, chocolate ice cream and a cola drink for dinner, and 2 glasses of wine in the evening before going to bed for a total caloric intake of 3500 calories. What instructions should the nurse provide? (Select all that apply) a. Maintain current caloric intake b. Avoid use of alcohol as a sleep aide at bedtime c. Reduce intake of dairy products d. Start a weight loss program e. Set a goal of increasing BMI (Body Mass Index)

b. Avoid use of alcohol as a sleep aide at bedtime d. Start a weight loss program

The nurse is evaluating the diet teaching of a client with hypertension. What dinner selection indicates that the client understands the dietary recommendation for hypertension? a. Tomato soup, grilled cheese sandwich, pickles, skim milk, and lemon meringue pie. b. Baked pork chop, applesauce, corn on the cob, 2% milk, and key-lime pie. c. Grilled steak, baked potato with sour cream, green beans, coffee and raisin cream pie. d. Beed stir fry, fried rice, egg drop soup, diet coke and pumpkin pie. Baked pork chop, applesauce, corn on the cob, 2% milk, and key-lime pie

b. Baked pork chop, applesauce, corn on the cob, 2% milk, and key-lime pie.

595. A client admitted to the emergency center had inspiratory and expiratory wheezing, nasal flaring, and thick, tenacious sputum secretions observed during the physical examination. Based on these assessment findings, what classification of pharmacologic agents should the nurse anticipate administering? a. Beta blockers b. Bronchodilators c. Corticosteroids d. Beta-adrenergic

b. Bronchodilators

A client with Alzheimer's disease (AD) is receiving trazodone (Desyrel), a recently prescribed atypical antidepressant. The caregiver tells the home health nurse that the client's mood and sleep patterns are improved, but there is no change in cognitive ability. How should the nurse respond to this information? a. Explain that it may take several weeks for the medication to be effective b. Confirm the desired effect of the medication has been achieved. c. Notify the health care provider than a change may be needed. d. Evaluate when and how the medication is being administered to the client.

b. Confirm the desired effect of the medication has been achieved.

The nurse is developing an educational program for older clients who are being discharged with new antihypertensive medications. The nurse should ensure that the educational materials include which characteristics? Select all that apply a. Written at a twelfth-grade reading level b. Contains a list with definitions of unfamiliar terms c. Uses common words with few Syllables d. Printed using a 12-point type font e. Uses pictures to help illustrate complex ideas

b. Contains a list with definitions of unfamiliar terms c. Uses common words with few Syllables e. Uses pictures to help illustrate complex ideas

Which assessment is more important for the nurse to include in the daily plan of care for a client with a burned extremity a. Range of Motion b. Distal pulse intensity c. Extremity sensation d. Presence of exudate

b. Distal pulse intensity

An Unna boot is applied to a client with a venous stasis ulcer. One week later, when the Unna boot is removed during a follow-up appointment, the nurse observes that the ulcer site contains bright red tissue. What action should the nurse take in response to this finding? a. Immediately apply a pressure dressing b. Document the ongoing wound healing. c. Irrigate the wound with sterile saline d. Obtain a capillary INR, measurement

b. Document the ongoing wound healing.

A client in her first trimester of pregnancy complains of nausea. Which complementary therapy should the nurse recommend? a. Eat food high in garlic with the evening meal b. Drink chamomile tea at breakfast and in the evening. c. Increase cocoa in the diet and drink before bedtime d. Join a yoga class that meets at least weekly

b. Drink chamomile tea at breakfast and in the evening.

A woman just learned that she was infected with Heliobacter pylori. Based on this finding, which health promotion practice should the nurse suggest? a. Schedule a colonoscopy within the next month. b. Encourage screening for a peptic ulcer. c. Screen all family member for hepatitis A d. Eat small, frequent meals thought the day.

b. Encourage screening for a peptic ulcer.

In assessing a client at 34-weeks' gestation, the nurse notes that she has a slightly elevated total T4 with a slightly enlarged thyroid, a hematocrit of 28%, a heart rate of 92 beats per minute, and a systolic murmur. Which finding requires follow-up? a. Elevated thyroid hormone level. b. Hematocrit of 28%. c. Heart rate of 92 beats per minute. d. Systolic murmur.

b. Hematocrit of 28%.

A client with arthritis has been receiving treatment with naproxen and now reports ongoing stomach pain, increasing weakness, and fatigue. Which laboratory test should the nurse monitor? a. Sed rate (ESR) b. Hemoglobin c. Calcium d. Osmolality.

b. Hemoglobin

During a well-baby, 6-month visit, a mother tells the nurse that her infant has had fewer ear infections than her 10-year-old daughter. The nurse should explain that which vaccine is likely to have made the difference in the siblings' incidence of otitis media? a. Varicella Virus Vaccine Live b. Hemophilic Influenza Type B (HiB) vaccine c. Pneumococcal vaccine d. Palivizumab vaccine for RSV

b. Hemophilic Influenza Type B (HiB) vaccine

A male client is admitted with burns to his face and neck. Which position should the nurse place the client to prevent contract? a. Flexed with the chin toward the chest. b. Hyperextended with neck supported by a rolled towel. c. Side-lying with the head on a pillow d. Prone with face supported by an inflated rubber ring.

b. Hyperextended with neck supported by a rolled towel.

When implementing a disaster intervention plan, which intervention should the nurse implement first? a. Initiate the discharge of stable clients from hospital units b. Identify a command center where activities are coordinated c. Assess community safety needs impacted by the disaster d. Instruct all essential off-duty personnel to report to the facility.

b. Identify a command center where activities are coordinated

An adult client is exhibit the maniac stage of bipolar disorder is admitted to the psychiatric unit. The client has lost 10 pounds in the last two weeks and has no bathed in a week "I'm trying to start a new business and "I'm too busy to eat". The client is oriented to time, place, person but not situation. Which nursing problem has the greatest priority? a. Hygiene-self-care deficit b. Imbalance nutrition c. Disturbed sleep pattern d. Self-neglect

b. Imbalance nutrition

The psychiatric nurse is talking to a newly admitted client when a male client diagnosed with antisocial behavior intrudes on the conversation and tells the nurse, "I have to talk to you right now! It is very important!" how should the nurse respond to this client? a. Put his behavior on extinction and continue talking with the newly admitted. b. Inform him that the nurse is busy admitting a new client and will talk to him later. c. Encourage him to go to the nurse's station and talk with another nurse. d. Introduce him to the newly admitted client and ask him to him to join in the conversation.

b. Inform him that the nurse is busy admitting a new client and will talk to him later.

A client experiencing withdrawal from the benzodiazepines alprazolam (Xanax) is demonstrating severe agitation and tremors. What is the best initial nursing action? a. Administer naloxone (Narcan) per PNR protocol b. Initiate seizure precautions c. Obtain a serum drug screen d. Instruct the family about withdrawal symptoms.

b. Initiate seizure precautions

What is the priority nursing action when initiating morphine therapy via an intravenous patient-controlled analgesia (PCA) pump? a. Assess the client's ability to use a numeric pain scale b. Initiate the dosage lockout mechanism on the PCA pump c. Instruct the client to use the medication before the pain become severe d. Assess the abdomen for bowel sounds

b. Initiate the dosage lockout mechanism on the PCA pump

An adolescent's mother calls the clinic because the teen is having recurrent vomiting and...Combative in the last 2 days. The mother states that the teen takes vitamins, calcium, mag...With aspirin. Which nursing intervention has highest priority? a. Advise the mother to withhold all medications by mouth. b. Instruct the mother to take the teen to the emergency room c. Recommend that the teen withhold food and fluids for 2 hours d. Suggest that the adolescent breath slowly and deeply.

b. Instruct the mother to take the teen to the emergency room

An older male adult resident of long-term care facility is hospitalized for a cardiac catheterization that occurred yesterday. Since the procedure was conducted, the client has become increasingly disoriented. The night shift nurse reports that he attempted to remove the sandbag from his femoral artery multiple times during the night. What actions should the nurse take? (Select all that apply.) a. Recommend a 24-hour caregiver on discharge to the long-term facility. b. Notify the healthcare provider of the client's change in mental status. c. Include q2 hour's reorientation in the client's plan of care. d. Request immediate evaluation by Rapid Response Team e. Apply soft wrist restraints so that the operative site is protected.

b. Notify the healthcare provider of the client's change in mental status. c. Include q2 hour's reorientation in the client's plan of care.

An older male client arrives at the clinic complaining that his bladder always feels full. He complains of weak urine flow, frequent dribbling after voiding, and increasing nocturia with difficulty initiating his urine stream. Which action should the nurse implement? a. Obtain a urine specimen for culture and sensitivity b. Palpate the client's suprapubic area for distention c. Advise the client to maintain a voiding diary for one week d. Instruct in effective technique to cleanse the glans penis

b. Palpate the client's suprapubic area for distention

The nurse has received funding to design a health promotion project for AfricanAmerican women who are at risk for developing breast cancer. Which resource is most important in designing this program? a. A listing of African-American women so live in the community b. Participation of community leaders in planning the program c. Morbidity data for breast cancer in women of all races d. Technical assistance to produce a video on breast self-examination.

b. Participation of community leaders in planning the program

In early septic shock states, what is the primary cause of hypotension? a. Peripheral vasoconstriction b. Peripheral vasodilation c. Cardiac failure d. A vagal response

b. Peripheral vasodilation

A client is admitted to the hospital after experiencing a brain attack, commonly referred to as a stroke or cerebral vascular accident (CVA). The nurse should request a referral for speech therapy if the client exhibits which finding? a. Abnormal responses for cranial nerves I and II b. Persistent coughing while drinking c. Unilateral facial drooping d. Inappropriate or exaggerated mood swings

b. Persistent coughing while drinking

In assessing a pressure ulcer on a client's hip, which action should the nurse include? a. Determine the degree of elasticity surrounding the lesion b. Photograph the lesion with a ruler placed next to the lesion c. Stage the depth of the ulcer using the Braden numeric scale d. Use a gloved finger to palpate for tunneling around the lesion

b. Photograph the lesion with a ruler placed next to the lesion

A client who is newly diagnosed with type 2 diabetes mellitus (DM) receives a prescription for metformin (Glucophage) 500 mg PO twice daily. What information should the nurse include in this client's teaching plan? (Select all that apply.) a. Take an additional dose for signs of hyperglycemia b. Recognize signs and symptoms of hypoglycemia. c. Report persist polyuria to the healthcare provider. d. Use sliding scale insulin for finger stick glucose elevation. e. Take Glucophage with the morning and evening meal.

b. Recognize signs and symptoms of hypoglycemia. c. Report persist polyuria to the healthcare provider. e. Take Glucophage with the morning and evening meal.

A client with intestinal obstructions has a nasogastric tube to low intermittent suction and is receiving an IV of lactated ringer's at 100 ml/H. which finding is most important for the nurse to report to the healthcare provider? a. Gastric output of 900 mL in the last 24 hours b. Serum potassium level of 3.1 mEq/L or mmol/L (SI) c. Increased blood urea nitrogen (BUN) d. 24-hour intake at the current infusion rate.

b. Serum potassium level of 3.1 mEq/L or mmol/L (SI)

The fire alarm goes off while the charge nurse is receiving the shift report. What action should the charge nurse implement first? a. Instruct the client's family member to stay in the visitor waiting area until further notice b. Tell the staff to keep all clients and visitors in the client rooms with the doors closed. c. Direct the nursing staff to evacuate the clients using the stairs in a calm and orderly manner. d. Call the hospital operator to determine if the is indeed a real emergency or a fire drill.

b. Tell the staff to keep all clients and visitors in the client rooms with the doors closed.

An adult male is brought to the emergency department by ambulance following a motorcycle accident. He was not wearing a helmet and presents with periorbital bruising and bloody drainage from both ears. Which assessment finding warrants immediate intervention by the nurse? a. Rebound abdominal tenderness b. nausea and projectile vomit c. rib pain with deep inspiration d. diminished bilateral breath sounds

b. nausea and projectile vomit

A female client who was mechanically ventilated for 7 days is extubated. Two hours later...productive cough, and her respirations are rapids and shallow. Which intervention is most important? a- Review record of recent analgesia b- Provide frequent pulmonary toilet c- Prepare the client for intubation d- Obtain STAT arterial blood gases

c- Prepare the client for intubation

While performing a skin inspection for a female adult client, the nurse observes a rash that is well circumscribed, has silvery scales and plaques, and is located on the elbows and knees. These assessment findings are likely to indicate which condition? a- Tinea corporis b- Herpes zoster c- Psoriasis d- Drug reaction

c- Psoriasis

The nurse is preparing a 50 ml dose of 50% dextrose IV for a client with insulin SHOCK... medication? a- Dilute the Dextrose in one liter of 0.9% Normal Saline solution. b- Mix the dextrose in a 50 ml piggyback for a total volume of 100 ml. c- Push the undiluted Dextrose slowly through the currently infusion IV. d- Ask the pharmacist to add the Dextrose to a TPN solution.

c- Push the undiluted Dextrose slowly through the currently infusion IV.

A female client is taking alendronate, a bisphosphate, for postmenopausal osteoporosis. The client tells the nurse that she is experiencing jaw pain. How should the nurse respond? a- Determine how the client is administering the medication b- Confirm that this is a common symptom of osteoporosis c- Report the client's jaw pain to the healthcare provider. d- Advise the client to gargle with warm salt water twice daily. Report the client's jaw pain to the healthcare provider.

c- Report the client's jaw pain to the healthcare provider.

A 35 years old female client has just been admitted to the post anesthesia recovery unit following a partial thyroidectomy. Which statement reflects the nurse's accurate understanding of the expected outcome for the client following this surgery? a- Supplemental hormonal therapy will probably be unnecessary b- The thyroid will regenerate to a normal size within a few years. c- The client will be restricted from eating seafood d- The remainder of the thyroid will be removed at a later date.

c- The client will be restricted from eating seafood

A client with gestational diabetes is undergoing a non-stress test (NST) at 34-week gestation... is 144 beats/minute. The client is instructed to mark the fetal monitor by pressing a button each time the baby moves. After 20 minutes, the nurse evaluates the fetal monitor strip what? a- The mother perceives and marks at least four fetal movements b- Fetal movements must be elicited with vibroacoustic stimulator c- Two FHR accelerations of 15 beats/minute x 15 seconds are recorded. d- No FHR late deceleration occur in response to fetal movement

c- Two FHR accelerations of 15 beats/minute x 15 seconds are recorded.

When administering an immunization in an adult client, the nurse palpates and administer the injection one inch below the acromion process into the center of the muscle mass. The nurse should document that the vaccine was administered at what site? a. Rectus femenis b. Ventrogluteous c. Vastus lateralis d. Deltoid

d. Deltoid

A toddler presents to the clinic with a barking cough, strider, refractions with respiration, the child's skin is pink with capillary refill of 2 seconds. Which intervention should the nurse implement? a. Encourage the child to cough b. Obtain a throat specimen for culture c. Administer nebulized epinephrine d. Collect blood for arterial blood gasses

c. Administer nebulized epinephrine

A 10 year old who has terminal brain cancer asks the nurse, "What will happen to my body when I die?" How should the nurse respond? a. "Your mother & father will be here soon. Talk to them about that." b. "Why do you want to know about what will happen to your body when you die?" c. "The heart will stop beating & you will stop breathing." d. "Are you concerned about where your spirit will go?"

c. "The heart will stop beating & you will stop breathing."

The nurse is assigned to care for clients on a medical unit. Based on the notes taken during the shift report, which client situation warrants the nurse's immediate attention? a. A young adult with Crohn's disease who reports having diarrheal stools b. An older adult with type 2 diabetes whose breakfast tray arrives 20 minutes late. c. A 10-year-old who is receiving chemotherapy and the infusion pump is beeping. d. A teenager who reports continued pain 30 minutes after receiving an oral analgesic.

c. A 10-year-old who is receiving chemotherapy and the infusion pump is beeping.

A family member of a frail elderly adult asks the nurse about eligibility requirements for hospice care. What information should the nurse provide? (Select all that apply.) a. All family must agree about the need for hospice care. b. Hospice services are covered under Medicare Part B. c. A client must be willing to accept palliative care, not curative care. d. The healthcare provider must project that the client has 6 months or less to live. e. All medications except pain treatment will be stopped during hospice care.

c. A client must be willing to accept palliative care, not curative care. d. The healthcare provider must project that the client has 6 months or less to live.

A nurse with 10 years experience working in the emergency room is reassigned to the perinatal unit to work an 8 hour shift. Which client is best to assign to this nurse? a. A client who is leaking clear fluid b. A mother who just delivered a 9 pounds boy c. A mother with an infected episiotomy. d. A client at 28- weeks' gestation in pre-term labor.

c. A mother with an infected episiotomy.

The charge nurse is making assignment on a psychiatric unit for a practical nurse (PN) and newly license register nurse (RN). Which client should be assigned to the RN? a. An adult female who has been depress for the past several months and denies suicidal ideations. b. A middle-age male who is in depressive phase on bipolar disease and is receiving Lithium. c. A young male with schizophrenia who said voices is telling him to kill his psychiatric. d. An elderly male who tell the staff and other client that he is superman and he can fly.

c. A young male with schizophrenia who said voices is telling him to kill his psychiatric.

The healthcare provider changes a client's medication prescription from IV to PO administration and double the dose. The nurse notes in the drug guide that the prescribed medication, when given orally, has a high first-pass effect and reduce bioavailability. What action should the nurse implement? a. Continue to administer the medication via the IV route b. Give half the prescribed oral dose until the provider is consulted. c. Administer the medication via the oral route as prescribed. d. Consult with the pharmacist regarding the error in prescription.

c. Administer the medication via the oral route as prescribed.

A female nurse who took drugs from the unit for personal use was temporarily released from duty. After completion of mandatory counseling, the nurse has asked administration to allow her to return to work. When the nurse administrator approaches the charge nurse with the impaired nurse request, which action is best for the charge nurse to take? a. Since treatment is completed, assign the nurse to the route RN responsibilities b. Ask to meet with impaired nurse's therapist before allowing her back on the unit. c. Allow the impaired nurse to return to work and monitor medication administration d. Meet with staff to assess their feelings about the impaired nurse's return to the unit.

c. Allow the impaired nurse to return to work and monitor medication administration

An older male comes to the clinic with a family member. When the nurse attempts to take the client's health history, he does not respond to questions in a clear manner. What action should the nurse implement first a. Ask the family member to answer the questions. b. Provide a printed health care assessment form c. Assess the surroundings for noise and distractions. d. Defer the health history until the client is less anxious.

c. Assess the surroundings for noise and distractions.

Which problem, noted in the client's history, is important for the nurse to be aware of prior to administration of a newly prescribed selective serotonin reuptake inhibitor (SSRI)? a. Bulimia nervosa b. Obsessive compulsive disorder c. Aural migraine headaches. d. Erectile dysfunction.

c. Aural migraine headaches.

A client in the intensive care unit is being mechanically ventilated, has an indwelling urinary catheter in place, and is exhibiting signs of restlessness. Which action should the nurse take first? a. Review the heart rhythm on cardiac monitors b. Check urinary catheter for obstruction c. Auscultated bilateral breath sounds d. Give PRN dose of lorazepam (Ativan)

c. Auscultated bilateral breath sounds

A client with a recent colostomy expresses concern about the ability to control flatus. Which intervention is most important for the nurse to include in the client's plan of care? a. Adhere to a bland diet whenever planning to eat out b. Decrease fluid intake at meal times c. Avoid foods that caused gas before the colostomy d. Eliminate foods high in cellulose

c. Avoid foods that caused gas before the colostomy

During a home visit, the nurse observed an elderly client with diabetes slip and fall. What action should the nurse take first? a. Give the client 4 ounces of orange juice b. Call 911 to summon emergency assistance c. Check the client for lacerations or fractures d. Asses clients blood sugar level

c. Check the client for lacerations or fractures

A female client is extremely anxious after being informed that her mammogram was abnormal and needs to be repeated. Client is tearful and tells the nurse her mother died of breast cancer. What action should the nurse take? a. Provide the client with information about treatment options for breast cancer. b. Reassure the client that the final diagnosis has not been made. c. Encourage the client to continue expressing her fears and concerns. d. Suggest to the client that she seek a second opinion.

c. Encourage the client to continue expressing her fears and concerns.

A vacuum-assistive closure (VAC) device is being use to provide wound care for a client who has stage III pressure ulcer on a below-the- knee (BKA) residual limb. Which intervention should the nurse implement to ensure maximum effectiveness of the device? a. Empty the device every 8 hours and change the dressing daily ensure sterility b. Extended the transparent film dressing only to edge of wound to prevent tension. c. Ensure the transparent dressing has no tears that might create vacuum leaks d. Use an adhesive remover when changing the dressing to promote comfort.

c. Ensure the transparent dressing has no tears that might create vacuum leaks

Which type of Leukocyte is involved with allergic responses and the destruction of parasitic worms? a. Neutrophils b. Lymphocytes c. Eosinophils d. Monocytes

c. Eosinophils

The nurse receives a newborn within the first minutes after a vaginal delivery and intervenes to establish adequate respirations. What priority issue should the nurse address to ensure the newborn's survival? a. Hypoglycemia b. Fluid balance c. Heat loss d. Bleeding tendencies

c. Heat loss

The healthcare provider prescribes acarbose (Precose), an alpha-glucosidase inhibitor, for a client with Type 2 diabetes mellitus. Which information provides the best indicator of the drug's effectiveness? a. Body max index (BMI) between 20 and 24 b. Blood pressure reading less than 120/80 mm Hg c. Hemoglobin A1C (HbA1C) reading less than 7% d. Self-reported glucose levels of 120-150 mg/dl.

c. Hemoglobin A1C (HbA1C) reading less than 7%

After six days on a mechanical ventilator, a male client is extubated and place on 40% oxygen via face mask. He is awake and cooperative, but complaining of a severe sore throat. While sipping water to swallow a medication, the client begins coughing, as if strangled. What intervention is most important for the nurse to implement? a. Administer PRN medication b. Titrate the oxygen to keep saturation above 92% c. Hold oral intake until swallow evaluation is done. d. Elevate the head of his bed at least 45 degrees.

c. Hold oral intake until swallow evaluation is done.

A client with severe full-thickness burns is scheduled for an allografting procedure. Which information should the nurse provide the client? a. The donor site will be painless a few days after the surgery b. Allografts are made from human and nonhuman material sources. c. Human sources graft require monitor for signs of graft injection d. Human source grafts require monitoring for signs of graft rejection.

c. Human sources graft require monitor for signs of graft injection

The nurse is evaluating the health teaching of a female client with condyloma acuminate. Which statement by the client indicates that teaching has been effective? a. These warts are caused by a fungus b. Early treatment is very effective c. I need to have regular pap smears d. I will clean my hot tub better

c. I need to have regular pap smears

When assessing a multigravida the first postpartum day, the nurse finds a moderate amount of lochia rubra, with the uterus firm, and three fingerbreadths above the umbilicus. What action should the nurse implement first? a. Massage the uterus to decrease atony b. Check for a distended bladder c. Increase intravenous infusion d. Review the hemoglobin to determined hemorrhage

c. Increase intravenous infusion

The nurse is assessing a first day postpartum client. Which finding is most indicative of a postpartum infection? a. White blood count of 19,000 mm3 b. Oral temperature of 100.2 F (37.9 C) c. Moderate amount of foul-smelling lochia. d. Blood pressure 122/74 mm Hg

c. Moderate amount of foul-smelling lochia.

A woman with an anxiety disorder calls her obstetrician's office and tells the nurse of increased anxiety since the normal vaginal delivery of her son three weeks ago. Since she is breastfeeding, she stopped taking her antianxiety medications, but thinks she may need to start taking them again because of her increased anxiety. What response is best for the nurse to provide this woman? a. Describe the transmission of drugs to the infant through breast milk b. Encourage her to use stress relieving alternatives, such as deep breathing exercises c. Inform her that some antianxiety medications are safe to take while breastfeeding d. Explain that anxiety is a normal response for the mother of a 3-week-old.

c. Inform her that some antianxiety medications are safe to take while breastfeeding

At 0600 while admitting a woman for a schedule repeat cesarean section (C-Section), the client tells the nurse that she drank a cup a coffee at 0400 because she wanted to avoid getting a headache. Which action should the nurse take first? a. Ensure preoperative lab results are available b. Start prescribed IV with lactated Ringer's c. Inform the anesthesia care provider d. Contact the client's obstetrician.

c. Inform the anesthesia care provider

What action should the school nurse implement to provide secondary prevention to a school-age children? a. Collaborate with a science teacher to prepare a health lesson b. Prepare a presentation on how to prevent the spread of lice c. Initiate a hearing and vision screening program for first-graders d. Observe a person with type 1 diabetes self-administer a dose of insulin

c. Initiate a hearing and vision screening program for first-graders

Which action should the school nurse take first when conducting a screening for scoliosis? a. Compare dorsal measurement of trunk b. Extend arms over head for visualization c. Inspect for symmetrical shoulder height. d. Observe weight-bearing on each leg.

c. Inspect for symmetrical shoulder height.

The nurse teaches an adolescent male client how to use a metered dose inhaler. Seen in the picture. What instruction should the nurse provide? a. Secure the mouthpiece under the tongue. b. Press down on the device after breathing in fully c. Move the device one to two inches away from the mouth d. Breathe out slowly and deeply while compressing the device

c. Move the device one to two inches away from the mouth

The nurse is auscultating a client's heart sounds. Which description should the nurse use to document this sound? (Please listen to the audio first to select the option that applies) a. S1 S2 b. S1 S2 S3 c. Murmur d. Pericardial friction rub.

c. Murmur

Which problem reported by a client taking lovastatin requires the most immediate fallow up by the nurse? a. Diarrhea and flatulence b. Abdominal cramps c. Muscle pain d. Altered taste

c. Muscle pain

A male client who was diagnosed with viral hepatitis A 4 weeks ago returns to the clinic complaining of weakness and fatigue. Which finding is most important for the nurse to report to the healthcare provider? a. Dark yellow-brown colored urine b. Nonspecific muscle and joint pain c. New onset of purple skin lesions. d. Weakness when getting up to walk.

c. New onset of purple skin lesions.

After receiving report, the nurse can most safely plan to assess which client last? The client with... a. A rectal tube draining clear, pale red liquid drainage b. A distended abdomen and no drainage from the nasogastric tube c. No postoperative drainage in the Jackson-Pratt drain with the bulb compressed d. Dark red drainage on a postoperative dressing, but no drainage in the Hemovac®.

c. No postoperative drainage in the Jackson-Pratt drain with the bulb compressed

Four hours after surgery, a client reports nausea and begins to vomit. The nurse notes that the client has a scopolamine transdermal patch applied behind the ear. What action should the nurse take? a. Reposition the transdermal patch to the client's trunk. b. Remove the transdermal patch until the vomiting subsides. c. Notify the healthcare provider of the vomiting. d. Explain that this is a side effect of the medication in the patch.

c. Notify the healthcare provider of the vomiting.

A client with possible acute kidney injury (AKI) is admitted to the hospital and mannitol is prescribed as a fluid challenge. Prior to carrying out this prescription, what intervention should the nurse implement? a. Collect a clean catch urine specimen. b. Instruct the client to empty the bladder. c. Obtain vital signs and breath sounds. d. No specific nursing action is required

c. Obtain vital signs and breath sounds.

The nurse instructs an unlicensed assistive personnel (UAP) to turn an immobilized elderly client with an indwelling urinary catheter every two hours. What additional action should the nurse instruct the UAP to take each time the client is turned? a. Empty the urinary drainage bag b. Feed the client a snack c. Offer the client oral fluids d. Assess the breath sounds

c. Offer the client oral fluids

A native-American male client diagnosed with pneumonia, states that in addition to his prescribed medical treatment of IV antibiotics he wishes to have a spiritual cleaning performed. Which outcome statement indicates that the best plan of care was followed? a. Identifies his ethnocentric values and behaviors b. States an understanding of the medical treatment c. Participated actively in all treatments regimens d. Expresses a desire for cultural assimilation

c. Participated actively in all treatments regimens

A client with rheumatoid arthritis (RA) starts a new prescription of etanercept (Enbrel) subcutaneously once weekly. The nurse should emphasize the importance of reporting problem to the healthcare provider? a. Headache b. Joint stiffness c. Persistent fever d. Increase hunger and thirst

c. Persistent fever

The nurse is demonstrating correct transfer procedures to the unlicensed assisted personnel (UAP) working on a rehabilitation unit. The UAPs ask the nurse how to safely move a physically disabled client from the wheelchair to a bed. What action should the nurse recommended? a. Hold the client at arm's length while transferring to better distribute the body weight. b. Apply the gait belt around the client's waits once standing position has been assumed. c. Place a client's locked wheelchair on the client's strong side next to the bed. d. Pull the client into position by reaching from the opposite side of the bed.

c. Place a client's locked wheelchair on the client's strong side next to the bed.

A male client with cirrhosis has ascites and reports feeling short of breath. The client is in semi Fowler position with his arms at his side. What action should the nurse implement? a. Reposition the client in a side-lying position and support his abdomen with pillows. b. Elevate the client's feet on a pillow while keeping the head of the bed elevated. c. Raise the head of the bed to a Fowler's position and support his arms with a pillow Place the client in a shock position and monitor his vital signs at frequent intervals

c. Raise the head of the bed to a Fowler's position and support his arms with a pillow

A client is admitted with an epidural hematoma that resulted from a skateboarding accident. To differentiate the vascular source of the intracranial bleeding, which finding should the nurse monitor? a. Slow increasing intracranial pressure (ICP) b. Decerebrate posturing c. Rapid onset of decreased level of consciousness. d. Coup contrecoup signs

c. Rapid onset of decreased level of consciousness.

An unlicensed assistive personnel (UAP) reports that a client's right hand and fingers spasms when taking the blood pressure using the same arm. After confirming the presence of spams what action should the nurse take? a. Ask the UAP to take the blood pressure in the other arm b. Tell the UAP to use a different sphygmomanometer. c. Review the client's serum calcium level d. Administer PRN antianxiety medication.

c. Review the client's serum calcium level

A male client reports the onset of numbness and tingling in his fingers and around his mouth. Which lab is important for the nurse to review before contacting the health care provider? a. capillary glucose b. urine specific gravity c. Serum calcium d. white blood cell count

c. Serum calcium

A client who received multiple antihypertensive medications experiences syncope due to a drop in blood pressure to 70/40. What is the rationale for the nurse's decision to hold the client's scheduled antihypertensive medication? a. Increased urinary clearance of the multiple medications has produced diuresis and lowered the blood pressure b. The antagonistic interaction among the various blood pressure medications has reduced their effectiveness c. The additive effect of multiple medications has caused the blood pressure to drop too low. d. The synergistic effect of the multiple medications has resulted in drug toxicity and resulting hypotension.

c. The additive effect of multiple medications has caused the blood pressure to drop too low.

The nurse performs a prescribed neurological check at the beginning of the shift on a client who was admitted to the hospital with a subarachnoid brain attack (stroke). The client's Glasgow Coma Scale (GCS) score is 9. What information is most important for the nurse to determine? a. When the client's stroke symptoms started b. If the client is oriented to time c. The client's previous GCS score. d. The client's blood pressure and respiration rate.

c. The client's previous GCS score.

A client with bipolar disorder began taking valproic acid (Depakote) 250 mg PO three times daily two months ago. Which finding provides the best indication that the medication regimen is effective? a. The nurse note that no pills remain in the prescription bottle. b. The client serum Depakote level is 125 mcg/ml c. The family reports a great reduction in client's maniac behavior d. The client denies any occurrence of suicidal ideation.

c. The family reports a great reduction in client's maniac behavior

When assessing a mildly obese 35-year-old female client, the nurse is unable to locate the gallbladder when palpating below the liver margin at the lateral border of the rectus abdominal muscle. What is the most likely explanation for failure to locate the gallbladder by palpation? a. The client is too obese b. Palpating in the wrong abdominal quadrant c. The gallbladder is normal d. Deeper palpation technique is needed

c. The gallbladder is normal

The nurse is assessing an older adult with type 2 diabetes mellitus. Which assessment finding indicates that the client understands long- term control of diabetes? a. The fasting blood sugar was 120 mg/dl this morning. b. Urine ketones have been negative for the past 6 months c. The hemoglobin A1C was 6.5g/100 ml last week d. No diabetic ketoacidosis has occurred in 6 months.

c. The hemoglobin A1C was 6.5g/100 ml last week

A client is admitted to a medical unit with the diagnosis of gastritis and chronic heavy alcohol abuse. What should the nurse administered to prevent the development of Wernicke's syndrome? a. Lorazepam (Ativan) b. Famotidine (Pepcid) c. Thiamine (Vitamin B1) d. Atenolol (Tenormin)

c. Thiamine (Vitamin B1)

The public nurse health received funding to initiate primary prevention program in the community. Which program the best fits the nurse's proposal? a. Case management and screening for clients with HIV. b. Regional relocation center for earthquake victims c. Vitamin supplements for high-risk pregnant women. d. Lead screening for children in low-income housing.

c. Vitamin supplements for high-risk pregnant women.

When teaching a group of school-age children how to reduce the risk of Lyme disease which instruction should the camp nurse include a. Wash hands frequently b. Avoid drinking lake water c. Wear long sleeves and pants d. Do not share personal products

c. Wear long sleeves and pants

In preparing assignments for the shift, which client is best for the charge nurse to assign to a practical nurse (PN)? a- An older client who fell yesterday and is now complaining of diplopia b- An adult newly diagnosed with type 1 diabetes and high cholesterol c- A client with pancreatic cancer who is experience intractable pain. d- An older client post-stroke who is aphasic with right-sided hemiplegia

d- An older client post-stroke who is aphasic with right-sided hemiplegia

A male client with COPD smokes two packs of cigarettes per day and is admitted to the hospital for a respiratory infection. He complains that he has trouble controlling respiratory distress at home when using his rescue inhaler. Which comment from the client indicates to the nurse that he is not using his inhaler properly? a. "I have a hard time inhaling and holding my breath after I squeeze the inhaler, but I do my best" b. "I never use the inhaler unless I am feeling really short of breath" c. I always shake the inhaler several times before I start" d. "After I squeeze the inhaler and swallow, I always feel a slight wave of nausea, bit it goes away"

d. "After I squeeze the inhaler and swallow, I always feel a slight wave of nausea, bit it goes away"

After a third hospitalization 6 months ago, a client is admitted to the hospital with ascites and malnutrition. The client is drowsy but responding to verbal stimuli and reports recently spitting up blood. What assessment finding warrants immediate intervention by the nurse? a. Bruises on arms and legs b. Round and tight abdomen c. Pitting edema in lower legs d. Capillary refill of 8 seconds

d. Capillary refill of 8 seconds

The charge nurse in a critical care unit is reviewing clients' conditions to determine who is stable enough to be transferred. Which client status report indicates readiness for transfer from the critical care unit to a medical unit? a. Pulmonary embolus with an intravenous heparin infusion and new onset hematuria b. Myocardial infarction with sinus bradycardia and multiple ectopic beats c. Adult respiratory distress syndrome with pulse oximetry of 85% saturation. d. Chronic liver failure with a hemoglobin of 10.1 and slight bilirubin elevation

d. Chronic liver failure with a hemoglobin of 10.1 and slight bilirubin elevation

The healthcare provider prescribes the antibiotic cephradine 500mg PO every 6 hours for a client with a postoperative wound infection. Which foods should the nurse encourage this client to eat a. Vanilla-flavored yogurt b. Low fat chocolate milk. c. Calcium fortified juice d. Cinnamon applesauce

d. Cinnamon applesauce

The home health nurse is assessing a male client who has started peritoneal dialysis (PD) 5 days ago. Which assessment finding warrants immediate intervention by the nurse? a. Finger stick blood glucose 120 mg/dL post exchange b. Arteriovenous (AV) graft surgical site pulsations. c. Anorexia and poor intake of adequate dietary protein d. Cloudy dialysate output and rebound abdominal pain

d. Cloudy dialysate output and rebound abdominal pain

When evaluating a client's rectal bleeding, which findings should the nurse document? a. Number of blood clots expelled with each stool. b. Unique odor noted with GI bleeding c. Evidence of internal hemorrhoids. d. Color characteristics of each stool.

d. Color characteristics of each stool.

A nurse is conducting a physical assessment of a young adult. Which information provides the best indication of the individual nutritional status? a. A 24-hour diet history b. History of a recent weight loss c. Status of current petite d. Condition of hair, nails, and skin

d. Condition of hair, nails, and skin

A client with hyperthyroidism is being treated with radioactive iodine (I-131). Which explanation should be included in preparing this client for this treatment? a. Explain the need for using lead shields for 2 to 3 weeks after the treatment b. Describe the signs of goiter because this is a common side effect of radioactive iodine c. Explain that relief of the signs/symptoms of hyperthyroidism will occur immediately d. Describe radioactive iodine as a tasteless, colorless medication administered by the healthcare provider

d. Describe radioactive iodine as a tasteless, colorless medication administered by the healthcare provider

The nurse is preparing an intravenous (IV) fluid infusion using an IV pump. Within 30 seconds of turning on the machine, the pump's alarm beeps "occlusion". What action should the nurse implement first? a. Flush the vein with 3 ml of sterile normal saline. b. Assess the IV catheter insertion site for infiltration. c. Verify the threading of the tubing through the IV pump. d. Determine if the clamp on the IV tubing is released

d. Determine if the clamp on the IV tubing is released

An infant is placed in a radiant warmer immediately after birth. At one hour of age, the nurse finds the infant tachypneic, and hypotonic. What is the first action that the nurse should take? a. Notify the healthcare provider immediately b. Increase the temperature of the radiant warmer c. Assess the infant's heart rate. d. Determine the infant's blood sugar level.

d. Determine the infant's blood sugar level

An infant who is admitted for surgical repair of a ventricular septal defect (VSD) is irritable and diaphoretic with jugular vein distention. Which prescription should the nurse administer first? a. Spironolactone b. Potassium c. Ampicillin sodium parental d. Digoxin.

d. Digoxin.

An adult woman who is seen in the clinic with possible neuropathic pain of the right leg rates her pain as a 7 on a 10 point scale. What action should the nurse take? a. Elevate the foot and leg on two pillows b. Measure the client's capillary glucose c. Ask the client to dorsiflex the right foot. d. Encourage the client to describe the pain.

d. Encourage the client to describe the pain.

A resident of a long-term care facility, who has moderate dementia, is having difficulty eating in the dining room. The client becomes frustrated when dropping utensils on the floor and then refuses to eat. What action should the nurse implement? a. Allow client to choose foods from a menu b. Assign a staff member to feed the client c. Have meals brought to the client's room d. Encourage the client to eat finger foods.

d. Encourage the client to eat finger foods.

The nurse note a depressed female client has been more withdrawn and noncommunicative during the past two weeks. Which intervention is most important to include in the updated plan of care for this client? a. Encourage the client's family to visit more often b. Schedule a daily conference with the social worker c. Encourage the client to participate in group activities d. Engage the client in a non-threatening conversation.

d. Engage the client in a non-threatening conversation.

An adult female client tells the nurse that though she is afraid her abusive boyfriend might one day kill her, she keeps hoping that he will change. What action should the nurse take first? a. Report the finding to the police department b. Discuss treatment options for abusive partners c. Determine the frequency and type of client's abuse d. Explore client's readiness to discuss the situation.

d. Explore client's readiness to discuss the situation.

A clinical trial is recommended for a client with metastatic breast cancer, but she refuses to participate and tells her family that she does not wish to have further treatments. The client's son and daughter ask the nurse to try and convince their mother to reconsider this decision. How should the nurse respond? a. Ask the client with her children present if she fully understands the decision she has made. b. Discuss success of clinical trials and ask the client to consider participating for one month. c. Explain to the family that they must accept their mother's decision. d. Explore the client's decision to refuse treatment and offer support

d. Explore the client's decision to refuse treatment and offer support

The nurse caring for a client with dysphagia is attempting to insert an NG tube, but the client will not swallow and is not gagging. What action should the nurse implement to facilitate the NGT passage into the esophagus? a. Push the NGT beyond the oropharynx gently yet swiftly. b. Offer the client sips of water or ice and coax to swallow c. Elevate the bed 90 degree and hyperextend the head. d. Flex the client's head with chin to the chest and insert.

d. Flex the client's head with chin to the chest and insert.

The nurse walks into a client's room and notices bright red blood on the sheets and on the floor by the IV pole. Which action should the nurse take first? a. Clean up the spilled blood to reduce infection transmission. b. Notify the healthcare provider that the client appears to be bleeding. c. Apply direct pressure to the client's IV site. d. Identify the source and amount of bleeding.

d. Identify the source and amount of bleeding.

The nurse inserts an indwelling urinary catheter as seen in the video what action should the nurse take next? a. Remove the catheter and insert into urethral opening b. Observe for urine flow and then inflate the balloon. c. Insert the catheter further and observe for discomfort. d. Leave the catheter in place and obtain a sterile catheter.

d. Leave the catheter in place and obtain a sterile catheter.

The nurse is caring for a client who had gastric bypass surgery yesterday. Which intervention is most important for the nurse to implement during the first 24 postoperative hours? a. Insert an indwelling urinary catheter b. Monitor for the appearance of an incisional hernia c. Instruct the client to eat small frequent meals d. Measure hourly urinary output.

d. Measure hourly urinary output.

While assisting a client who recently had a hip replacement into a bed pan, the nurse notices that there is a small amount of bloody drainage on the surgical dressing, the client's skin is warm to the touch, and there is a strong odor from the urine. Which action should the nurse take? a. Obtain a urine sample from the bed pan b. Remove dressing and assess surgical site c. Insert an indwelling urinary catheter d. Measure the client's oral temperature

d. Measure the client's oral temperature

The nurse is interviewing a client with schizophrenia. Which client behavior requires immediate intervention? a. Lip smacking and frequent eye blinking b. Shuffling gait and stooped posture c. Rocks back and forth in the chair d. Muscle spasms of the back and neck

d. Muscle spasms of the back and neck

A client is admitted to the emergency department with a respiratory rate of 34 breaths per minute and high pitched wheezing on inspiration and expiration, the medical diagnosis is severe exacerbation of asthma. Which assessment finding, obtained 10 min after the admission assessment, should the nurse report immediately to the emergency department healthcare provider? a. An apical pulse of 120 beats per minute b. Extreme agitation with staff and family c. Client report being anxious d. No wheezing upon auscultation of the chest.

d. No wheezing upon auscultation of the chest.

While removing an IV infusion from the hand of a client who has AIDS, the nurse is struck with the needle. After washing the puncture site with soap & water, which action should the nurse take? a. Complete a usual incident report b. Start prophylactic treatment c. Seek psychological resources d. Notify the employee health nurse.

d. Notify the employee health nurse.

A client with myasthenia Gravis (MG) is receiving immunosuppressive therapy. Review recent laboratory test results show that the client's serum magnesium level has decreased below the normal range. In addition to contacting the healthcare provider, what nursing action is most important? a. Check the visual difficulties b. Note most recent hemoglobin level c. Assessed for he and Hand joint pain d. Observe rhythm on telemetry monitor

d. Observe rhythm on telemetry monitor

Suicide precautions are initiated for a child admitted to the mental health unit following an intentional narcotic overdose. After a visitor leaves, the nurse finds a package of cigarettes in the client's room. Which intervention is most important for the nurse to implement? a. Assign a sitter for constant observation b. Screen future visitors for contraband c. Document suicide monitoring frequently d. Remove cigarettes for the client's room.

d. Remove cigarettes for the client's room.

During an annual physical examination, an older woman's fasting blood sugar (FBS) is determined to be 140 mg/dl or 7.8 mmol/L (SI). Which additional finding obtained during a follow-up visit 2 weeks later is most indicative that the client has diabetes mellitus (DM)? a. An increased thirst with frequent urination b. Blood glucose range during past two weeks was 110 to 125 mg/dl or 6.1 to 7.0 mmol/L(SI) c. Two-hour postprandial glucose tolerance test (GTT) is 160 mg/dL or 8.9 mmol/L (SI) d. Repeated fasting blood sugar (FBS) is 132 mg/dl or 7.4 mmol/L (SI).

d. Repeated fasting blood sugar (FBS) is 132 mg/dl or 7.4 mmol/L (SI).

During discharge teaching, the nurse discusses the parameters for weight monitoring with a client who was recently diagnosed with heart failure (HF). Which information is most important for the client to acknowledge? a. Keep a daily weight record b. Obtain weight at the same time every day c. Limit intake of dietary salt. d. Report weight gain of 2 pounds (0.9kg) in 24 hours

d. Report weight gain of 2 pounds (0.9kg) in 24 hours

An older adult male is admitted with complications related to chronic obstructive pulmonary disease (COPD). He reports progressive dyspnea that worsens on exertion and his weakness has increased over the past month. The nurse notes that he has dependent edema in both lower legs. Based on these assessment findings, which dietary instruction should the nurse provide? a. Limit the intake of high calorie foods. b. Eat meals at the same time daily. c. Maintain a low protein diet. d. Restrict daily fluid intake. Restrict daily fluid intake.

d. Restrict daily fluid intake.

A male client reports to the clinic nurse that he has been feeling well and is often "dizzy" his blood pressure is elevated. Based on this findings, this client is at a greatest risk for which pathophysiological condition? a. Pulmonary hypertension b. Left ventricular hypertrophy c. Renal failure d. Stroke

d. Stroke

The nurse is making a home visit to a male client who is in the moderate stage of Alzheimer's diseases. The client's wife is exhausted and tells the nurse that the family plans to take turns caring for the client in their home, each keeping him for two weeks at a time. How should the nurse respond? a. Advise the client's spouse to consider inpatient hospice care as an alternative b. Suggest that each rotation last one week, rather than two, to prevent caregiver fatigue c. Use active listening to allow the client and spouse to express their feelings about the plan d. Suggest enrolling the client in adult daycare instead of rotating among family.

d. Suggest enrolling the client in adult daycare instead of rotating among family.

A 350-bed acute care hospital declares an internal disaster because the emergency generators malfunctioned during a city-wide power failure. The UAPs working on a general medical unit ask the charge nurse what they should do first. What instruction should the charge nurse provide to these UAPs? a. Go to the emergency department and complete assigned tasks b. Shut all doors to client rooms on the unit in case a fire erupts c. Offer to assist the ICY with ventilator-dependent clients d. Tell all their assigned clients to stay in their rooms.

d. Tell all their assigned clients to stay in their rooms.

For the past 24 hours, an antidiarrheal agent, diphenoxylate, has been administered to a bedridden, older client with infectious gastroenteritis. Which finding requires the nurse to take further action? a. Loss of appetite b. Serum K 4.0 mEq/or mmol/dl (SI) c. Loose, runny stool d. Tented skin turgor.

d. Tented skin turgor.

A client who is taking an oral dose of a tetracycline complains of gastrointestinal upset. What snack should the nurse instruct the client to take with the tetracycline? a. Fruit-flavored yogurt. b. Cheese and crackers. c. Cold cereal with skim milk. d. Toasted wheat bread and jelly

d. Toasted wheat bread and jelly

Azithromycin is prescribed for an adolescent female who has lower lobe pneumonia and recurrent chlamydia. What information is most important for the nurse to provide to this client a. Have partner screened for human immunodeficiency virus b. Report a sudden onset arthralgia to the healthcare provider c. Decrease intake of high-fat-foods, caffeine, and alcohol d. Use two forms of contraception while taking this drug.

d. Use two forms of contraception while taking this drug.

A client with history of bilateral adrenalectomy is admitted with a week, irregular pulse, and hypotension. Which assessment finding warrants immediate intervention by the nurse? a. Decrease urinary output b. Low blood glucose level c. Profound weight gain d. Ventricular arrhythmias.

d. Ventricular arrhythmias.


Ensembles d'études connexes

IB Computer Science: Topic 1 - System Fundamentals

View Set

1.1 Introduction to Business Management

View Set

Accounting 1 - Chapter 9 TRUE/FALSE

View Set

Determining Author's Purpose/Point of View in Informational Text

View Set

Leadership Test #1 (Ch 1-6 10th Edition + Kahoot)

View Set

CHP 5 - Personal Planning and Recruiting

View Set

Exam 2: MGT 3513: Introduction to Human Resource Management (Dr. Kincy Madison) Spring 2018: Mississippi State University

View Set

Тема 1. Поняття ризику, його класифікація та місце в страхуванні

View Set